Você está na página 1de 107

Polos Olmpicos de Treinamento

Curso de Teoria dos Nmeros - Nvel 2


Samuel Barbosa Feitosa
Aula
1
Divisibilidade I
Teorema 1. (Algoritmo da Divisao) Para quaisquer inteiros positivos a e b, existe um unico
par (q, r) de inteiros n ao negativos tais que b = aq + r e r < a. Os n umeros q e r sao
chamados de quociente e resto, respectivamente, da divisao de b por a.
Exemplo 2. Encontre um n umero natural N que, ao ser dividido por 10, deixa resto 9, ao
ser dividido por 9 deixa resto 8, e ao ser dividido por 8 deixa resto 7.
O que acontece ao somarmos 1 ao nosso n umero? Ele passa a deixar resto 0 na divisao por
10, 9 e 8. Assim, um possvel valor para N e 10 9 8 1.
Exemplo 3. a) Verique que a
n
1 = (a 1)(a
n1
+ a
n2
+ . . . + a + 1)
b) Calcule o resto da divisao de 4
2012
por 3.
Para o item a), usando a distributividade e efetuando os devidos cancelamentos no lado
direito, podemos escrever:
a
n
+ a
n1
+ . . . + a
2
+ a a
n1
a
n2
. . . a 1 = a
n
1.
Para o item b), veja que 3 = 41 e assim e natural substituir os valores dados na express ao
do primeiro item:
4
2012
1 = 3(4
2011
+ . . . + 4 + 1).
Isso signica que q = (4
2011
+ . . . + 4 + 1) e que r = 1.
Observacao 4. O teorema anterior admite um enunciado mais geral: Para quaisquer intei-
ros a e b, com a = 0, existe um unico par de inteiros (q, r) tais que b = aq +r, 0 r < |a|.
Por exemplo, o resto da divisao de 7 por 3 e 2 e o quociente e 3.
Iremos agora estudar propriedades a respeito das operacoes com restos.
Teorema 5. (Teorema dos Restos) Se b
1
e b
2
deixam restos r
1
e r
2
na divisao por a,respectivamente,
entao:
POT 2012 - Teoria dos N umeros - Nvel 2 - Aula 1 - Samuel Feitosa
b
1
+ b
2
deixa o mesmo resto que r
1
+ r
2
na divisao por a
b
1
b
2
deixa o mesmo resto que r
1
r
2
na divisao por a.
Demonstrac ao. Por hip otese, existem q
1
, q
2
e q tais que: b
1
= aq
1
+ r
1
, b
2
= aq
2
+ r
2
e
r
1
+ r
2
= aq + r, logo:
b
1
+ b
2
= a(q
1
+ q
2
+ q) + r.
Como 0 < r < |a|, b
1
+ b
2
deixa resto r quando dividido por a. A demonstra cao para o
produto e deixada ao cargo do leitor.
Observacao 6. Em alguns casos, e prefervel que o professor faca uma demonstrac ao do
resultado anterior para a = 3 ou a = 5 apenas com o intuito de deixar os alunos mais
confort aveis a respeito do resultado.

E prefervel que mais tempo seja gasto resolvendo
exemplos e problemas. Na se c ao de congruencias, os alunos terao um contato mais apro-
priado com o enunciado anterior.
Exemplo 7. Qual o resto que o n umero 1002 1003 1004 deixa quando dividido por 7?
Como 1002 deixa resto 1 por 7, o n umero acima deixa o mesmo resto que 1 2 3 = 6 por 7.
Exemplo 8. Qual o resto que o n umero 4
5000
deixa quando dividido por 3?
Como 4 deixa resto 1 por 3, 4
5000
deixa o mesmo resto que 1 1 . . . 1
. .
5000
= 1 por 3.
Exemplo 9. Qual o resto que o n umero 2
2k+1
deixa quando dividido por 3?
Note que 2
0
deixa resto 1 por 3, 2
1
deixa resto 2 por 3, 2
2
deixa resto 1 por 3, 2
3
deixa
resto 2 por 3, 2
4
deixa resto 1 por 3. Precebeu alguma coisa? Como 100 e par, o resto
devera ser 1. Como 2
2
deixa resto 1, entao 2
2k
= 2
2
2
2
. . . 2
2
. .
k
deixa o mesmo resto que
1 1 . . . 1
. .
k
= 1 e 2
2k+1
= 2
2k
2 deixa o mesmo resto que 1 2 = 2 por 3.
Exemplo 10. Qual o resto de n
3
+ 2n na divisao por 3?
Se o resto de n por 3 e r, o resto de n
3
+ 2n e o mesmo de r
3
+ 2r. Para r = 0, esse
resto seria 0. Para r = 1, seria o mesmo resto de 3 que e 0. Finalmente, para r = 2, o
resto seria o mesmo de 8 + 4 = 12 que tambem e 0. Assim, nao importa qual o resto de n
por 3, o n umero n
3
+ 2n sempre deixara resto 0. Uma ideia importante nessa solucao foi
divid-la em casos. Tambem poderamos ter resolvido esse exemplo apelando para alguma
fatora cao:
n
3
+ 2n = n
3
n + 3n = n(n
2
1) + 3n = n(n 1)(n + 1) + 3n.
Como n1, n e n+1 s ao consecutivos, um deles e m ultiplo de 3. Assim, o ultimo termo da
igualdade anterior e a soma de dois m ultiplos de 3 e consequentemente o resto procurado
e 0.
2
POT 2012 - Teoria dos N umeros - Nvel 2 - Aula 1 - Samuel Feitosa
Observacao 11. Fatorac oes podem ser muito uteis para encontrarmos os valores explcitos
de q e r.
Exemplo 12. Prove que, para cada n natural,
(n + 1)(n + 2) . . . (2n)
e divisvel por 2
n
.
Veja que
(n + 1)(n + 2) . . . (2n) =
1 2 2n
1 2 n
.
Para cada n umero natural k no produto escrito no denominador, temos uma aparicao de
2k no produto escrito no numerador. Basta efetuarmos os cancelamentos obtendo:
(n + 1)(n + 2) . . . (2n) = 2
n
1 3 (2n 1).
Exemplo 13. (Olimpada de Leningrado 1991) Cada um dos naturais a, b, c e d e divisvel
por ab cd, que tambem e um n umero natural. Prove que ab cd = 1.
Se chamarmos p = ab cd, teremos a = px, b = py, c = pz e d = pt onde x, y, z e t s ao
inteiros. Assim, p = p
2
(xy zt). Consequentemente 1 = p(xy zt) e conclumos que p = 1,
pois p e natural.
Exemplo 14. A soma digital D(n) de um inteiro positivo n e denida recursivamente como
segue:
D(n) =
_
n se 1 n 9,
D(a
0
+ a
1
+ . . . + a
m
) se n > 9,
onde a
0
, a
1
, . . . , a
m
sao todos os dgitos da expressao decimal de n na base 10, i.e.,
n = a
m
10
m
+ a
m1
10
m1
+ . . . + a
1
10 + a
0
Por exemplo, D(989) = D(26) = D(8) = 8. Prove que: D((1234)n) = D(n), para n =
1, 2, 3 . . .
Como 10
n
1
n
= (101)(10
n1
+10
n2
+. . . +1), podemos concluir que 10
n
sempre deixa
resto 1 na divisao por 9. Assim, n = a
m
10
m
+a
m1
10
m1
+. . . +a
1
10+a
0
, deixa o mesmo
resto que a
m
+ a
m1
+ . . . + a
0
na divisao por 9. Desse modo, D(n) nada mais e do que
o resto na divisao por 9 do n umero n. Como 1234 deixa resto 1 por 9, o n umero (1234)n
deixa o mesmo resto que 1 n por 9, ou seja, D((1234)n) = D(n).
Observacao 15. O exemplo anterior contem o criterio de divisibilidade por 9, i.e., n deixa
o mesmo resto que D(n) na divisao por 9. O criterio de divisibilidade por 3 e an alogo pois
10
n
tambem sempre deixa resto 1 por 3.
Exemplo 16. Encontre todos os pares de inteiros positivos a e b tais que 79 = ab +2a +3b.
3
POT 2012 - Teoria dos N umeros - Nvel 2 - Aula 1 - Samuel Feitosa
Fatoremos a expressao anterior. Somando 6 aos dois lados da equa cao, obtemos:
85 = 6 + ab + 2a + 3b
= (3 + a)(2 + b)
Assim, (3 + a) e (2 + b) s ao divisores positivos de 85 maiores que 1. Os unicos divisores
positivos de 85 s ao 1, 5, 19, 85. Logo, os possveis pares de valores para (3 + a, 2 + b) s ao
(5, 19) ou (19, 5) que produzem as solucoes (a, b) = (2, 17) e (16, 3).
Problema 17. (Olimpada Russa) Prove que se
2
n
2
n
e um inteiro, entao
2
2
n
1
2
2
n
1
tambem e um inteiro.
Se k =
2
n
2
n
, entao
2
2
n
1
2
2
n
1
=
2(2
2
n
2
1)
2
n
1
= 2
_
2
nk
1
2
n
1
_
= 2
_
(2
n
1)(2
n(k1)
+ 2
n(k2)
+ . . . + 2
n
+ 1)
2
n
1
_
= 2(2
n(k1)
+ 2
n(k2)
+ . . . + 2
n
+ 1),
e um n umero inteiro.
Problemas Propostos
Problema 18. Encontre os inteiros que, na divisao por 7, deixam um quociente igual ao
resto.
Problema 19. Determinar os n umeros que divididos por 17 dao um resto igual ao quadrado
do quociente correspondente.
Problema 20. (OCM 1985) Encontre o quociente da divisao de a
128
b
128
por
(a
64
+ b
64
)(a
32
+ b
32
)(a
16
+ b
16
)(a
8
+ b
8
)(a
4
+ b
4
)(a
2
+ b
2
)(a + b)
Problema 21. (OCM 1994) Seja A = 777 . . . 77 um n umero onde o dgito 7aparece 1001
vezes. Determinar o quociente e o resto da divisao de A por 1001.
Problema 22. Encontre um inteiro que deixa resto 4 na divisao por 5 e resto 7 na divisao
por 13
Problema 23. Encontre o menor inteiro que, dividido por 29 deixa resto 5, e dividido por
31 da resto 28.
4
POT 2012 - Teoria dos N umeros - Nvel 2 - Aula 1 - Samuel Feitosa
Problema 24. Prove que, para todo inteiro positivo n o n umero n
5
5n
3
+ 4n e divisvel
por 120.
Problema 25. (Fatorac oes Importantes)
a) Seja S = 1+z +z
2
+z
3
+. . . +z
n1
. Veja que S +z
n
= 1+zS entao S(z 1) = z
n
1.
Conclua que, para quaisquer x e y vale:
x
n
y
n
= (x y)(x
n1
+ x
n2
y + x
n3
y
2
+ . . . + x
2
y
n3
+ xy
n2
+ y
n1
)
b) Mostre que se n e mpar vale:
x
n
+ y
n
= (x + y)(x
n1
x
n2
y + x
n3
y
2
. . . + x
2
y
n3
xy
n2
+ y
n1
)
Problema 26. Prove que, o n umero 1
99
+ 2
99
+ 3
99
+ 4
99
+ 5
99
e m ultiplo de 5.
Problema 27. Mostre que o n umero 1
n
+ 8
n
3
n
6
n
e multiplo de 10 para todo natural
n.
Problema 28. Encontre o resto da divisao 37
10
1 por 11.
Problema 29. Prove que 2222
5555
+ 5555
2222
e divisvel por 7.
Problema 30. Encontre o ultimo dgito do n umero 1989
1989
.
Problema 31. Mostre que se n divide a entao 2
n
1 divide 2
a
1.
Problema 32. (Cone Sul 1996) Provar que o n umero
1995 1997
1996
1996 1997
1995
+ 1
1996
2
e um inteiro.
Problema 33. Mostre que para n mpar, n divide 1
n
+ 2
n
+ . . . + (n 1)
n
Problema 34. Existe um natural n tal que n
n
+ (n + 1)
n
e divisvel por 2011?
Problema 35. Quantos n umeros inteiros positivos n existem tais que n + 3 divide n
2
+ 7?
Problema 36. Encontre o n umero de inteiros n tais que
1. 1000 < n < 8000.
2. n
n+1
+ (n + 1)
n
e divisvel por 3.
Problema 37. Sejam m e n naturais tais que mn + 1 e m ultiplo de 24, mostre que m + n
tambem e m ultiplo de 24.
5
POT 2012 - Teoria dos N umeros - Nvel 2 - Aula 1 - Samuel Feitosa
Problema 38. (Irlanda 1997) Encontre todos os pares de inteiros (x, y) tais que 1+1996x+
1998y = xy.
Dicas e Solu coes
18. Os n umeros s ao {0, 8, 16, 24, . . . , 8 7}.
18. Escreva n = 17q + q
2
e note que 0 q
2
< 17. Assim, q = 0, 1, 2, 3, 4.
19. Use a diferen ca de quadrados sucessivas vezes para obter (a b) como quociente.
21. O n umero do problema e igual a
7(10
1001
1)
9
. Alem disso,
10
999
+1
10
3
+1
e inteiro e
10
1001
1
10
3
+1
=
100
10
999
+1
10
3
+1

100
10
3
+1
.
22. Os n umeros que satisfazem essa propriedade s ao os n umeros da forma 65k + 59.
24. Basta mostrar que n
5
5n
3
+ 4n e m ultiplo de 3, 8 e 5. Na divisao por 5, temos
quatro restos possveis: {0, 1, 2, 3, 4}. Assim, o n umero n
5
5n
3
+4n possui o mesmo
resto na divisao por 5 que um dos cinco n umeros: {0
5
5 0
3
+40, 1
5
5 1
3
+4, 2
5

5 2
3
+ 8, 3
5
5 3
3
+ 12, 4
5
5 4
3
+ 16}. Como todos esses n umeros s ao m ultiplos
de 5, segue que n
5
5n
3
+ 4n e m ultiplo de 5 para todo n inteiro. O procedimento
com 3 e 8 e semelhante.
25. Para o item a), troque z por
x
y
. Para o item b), substitua y por y no item anterior.
26. Pelo problema anterior, como 99 e mpar temos: 1
99
+ 4
99
= (1 + 4)(1
98
+ 1
97
4 +
. . . +1 4
97
+4
98
). Da, segue que 1
99
+4
99
e m ultiplo de 5. Analogamente podemos
mostrar que 2
99
+ 3
99
e m ultiplo de 5.
27. O n umero em quest ao e m utiplo de 2 pois e a soma de dois mpares e dois pares.
Para ver que tambem e m ultiplo de 5, basta notar que 5 divide 1
n
6
n
e 8
n
3
n
.
Isso pode ser facilmente mostrado usando a fatora cao do exerccio 25.
31. Se a = nk, temos (2
n
1)(2
n(k1)
+ 2
n(k2)
+ . . . + 2
n
+ 1) = 2
nk
1.
32. Veja que 19951997
1996
19961997
1995
+1 = 1995(1997
1996
1)1996(1997
1995
1).
Pela fatora cao de x
n
y
n
,
1996 (1997
1995
1)
1996
2
= (1997
1994
+ 1997
1993
+ . . . + 1),
e inteiro. Alem disso, pela mesma fatora cao,
1995 (1997
1996
1)
1996
2
= 1995
_
1997
1995
1
1996
+
1997
1994
1
1996
+ . . . +
1997 1
1996
+
1996
1996
_
,
e uma soma de n umeros inteiros.
6
33. Como n e impar,
(n i)
n
+ i
n
= ((n i) + i)((n i)
n1
(n i)
n2
i + . . . (n i)i
n2
+ i
n1
).
34. Fa ca n = 1005 e use a fatora cao de x
n
+ y
n
.
37. Fatore a expressao como:
(x 1998)(y 1996) = xy 1998y 1996x + 1998 1996 = 1997
2
.
Os divisores de 1997
2
s ao {1, 1997, 1997
2
}. Resolvendo os sistemas correspon-
dentes `a essas possibilidades, temos: (x, y) = (1999, 1997
2
+ 1996), (1997, 1997
2
+
1996), (3995, 3993), (1, 1), (1997
2
+ 1998, 1997), (1997
2
+ 1998, 1995).
Referencias
[1] F. E. Brochero Martinez, C. G. Moreira, N. C. Saldanha, E. Tengan - Teoria dos
N umeros um passeio com primos e outros n umeros familiares pelo mundo inteiro,
Projeto Euclides, IMPA, 2010.
[2] E. Carneiro, O. Campos and F. Paiva, Olimpadas Cearenses de Matem atica 1981-2005
(Nveis J unior e Senior), Ed. Realce, 2005.
[3] S. B. Feitosa, B. Holanda, Y. Lima and C. T. Magalhaes, Treinamento Cone Sul 2008.
Fortaleza, Ed. Realce, 2010.
[4] D. Fomin, A. Kirichenko, Leningrad Mathematical Olympiads 1987-1991, MathPro
Press, Westford, MA, 1994.
[5] D. Fomin, S. Genkin and I. Itenberg, Mathematical Circles, Mathematical Words, Vol.
7, American Mathematical Society, Boston, MA, 1966.
[6] I. Niven, H. S. Zuckerman, and H. L. Montgomery, An Introduction to the Theory of
Numbers.
Polos Olmpicos de Treinamento
Curso de Teoria dos Nmeros - Nvel 2
Prof. Samuel Feitosa
Aula
2
Divisibilidade II
Deni cao 1. Dados dois inteiros a e b, com a = 0, dizemos que a divide b ou que a e
um divisor de b ou ainda que b e um m ultiplo de a e escrevemos a | b se o r obtido pelo
algoritmo de divisao aplicado `a a e b e 0, ou seja, se b = aq para algum inteiro q.
Lema 2. Sejam a, b, c, d inteiros. Temos
i) (d divide) Se d | a e d | b, entao d | ax + by para quaisquer x e y inteiros.
ii) (Limitac ao) Se d | a, entao a = 0 ou |d| |a|.
iii) (Transitividade) Se a | b e b | c, entao a | c.
Em particular, segue da propriedade i) que d | a + b e d | a b.
Exemplo 3. (Olimpada de Maio 2006) Encontre todos os naturais a e b tais que a|b + 1 e
b|a + 1.
Pela propriedade da Limitacao, temos a b + 1 e b a + 1. Da, a 1 b a + 1.
Vejamos os casos:
(i) a = b. Como a|b + 1 e a | b(pois b = a) temos que a | [(b + 1) b] = 1. Assim, a = 1
Nesse caso, s o temos a solucao (a, b) = (1, 1)
(ii) a = b +1. Como b|a +1 e b|a 1(pois b = a 1) temos que b|[(a +1) (a 1)] = 2.
Assim, b = 1 ou b = 2 e nesse caso, s o temos as solucoes (3, 2) e (2, 1).
(iii) a = b 1. Esse caso e an alogo ao anterior e as solucoes para (a, b) s ao (1, 2) e (2, 3).
Exemplo 4. (Criterio de Divisibilidade por 7) Existem alguns metodos pr aticos para deci-
dirmos se um n umero e m ultiplo de outro. Certamente o leitor ja deve ter se deparado com
algum criterio de divisibilidade. Existe um criterio por 7 bastante popular: Para saber se
um inteiro e multiplo de 7, basta apagar seu ultimo dgito, multiplic a-lo por 2 e o subtrair
do n umero que restou. Se o resultado e m ultiplo de 7, entao o n umero original tambem e
m ultiplo de 7.
POT 2012 - Teoria dos N umeros - Nvel 2 - Aula 1 - Samuel Feitosa
Podemos aplicar esse algoritmo sucessivas vezes ate que o resultado obtido seja facil-
mente vericavel como um m ultiplo de 7. Por exemplo, para o n umero 561421 podemos
escrever:
56142 2 = 56140
5614 0 = 5614
561 8 = 553
55 6 = 49
Como 49 e m ultiplo de 7, nosso n umero original tambem e. Por que esse processo funciona?
Se o nosso n umero original esta escrito na forma 10a + b, entao o n umero obtido ap os a
operacao descrita e a 2b. Basta mostrarmos que se 7 | a 2b, entao 7 | 10a + b. Se
7 | a 2b, pela propriedade (i) do lema, conclumos que 7 | 10a 20b. Como 7 | 21b,
tambem temos que 7 | [(10a 20b) + 21b] = 10a + b.
Exemplo 5. Mostre que se 7 | 3a + 2b entao 7 | 4a 2b.
Veja que 7 | 7a e 7 | 3a + 2b, entao 7 | [7a (3a + 2b)] = 4a 2b. Na pratica, o que
zemos foi multiplicar o n umero 3a +2b por algum inteiro para posteriormente subtramos
um m ultiplo de 7 conveniente e obtermos o n umero 4a 2b. Existem outras formas de
fazermos isso. Observe os n umeros 3 0, 3 1, 3 2, 3 3, 3 4, 3 5, 3 6. O n umero 3 6 deixa
o mesmo resto que 4 por 7, pois 3 6 = 7 2 + 4. Como 7|3a + 2b podemos concluir que
7|(18a + 12b) e consequentemente 7 | [18a + 12b 14a)] = 4a + 12b. Mas 7 | 14b, entao
7 | [4a + 12b 14b] = 4a 2b.
Para o proximo exemplo, o leitor precisara lembrar dos criterios de divisibilidade por 9
e 3 vistos na aula passada.
Exemplo 6. Usando os dgitos 1, 2, 3, 4, 5, 6, 7, construmos v arios n umeros de sete dgitos
distintos. Existem dois deles, distintos, tais que um divide o outro?
N ao. Suponha, por absurdo, que m < n sejam dois desses n umeros, com m | n. Claramente
m | n m e 9 | n m, pois n e m possuem a mesma soma dos dgitos e consequentemente
possuem o mesmo resto na divisao por 9. Por outro lado, sabemos a soma dos dgitos de
m: 1+2+ +7 = 3 9+1. Da, m nao possui fator 9 e podemos garantir que 9m | nm.
Mas entao 9m n m 10m n n tem pelo menos oito dgitos, uma contradi cao.
Exemplo 7. (Leningrado 1989) Seja A um n umero natural maior que 1, e seja B um
n umero natural que e um divisor de A
2
+ 1. Prove que se B A > 0, entao B A >

A.
Seja B A = q. Assim, A + q | A
2
+ 1. Como (A q)(A + q) = A
2
q
2
e divisvel por
A + q, podemos concluir que A + q | [(A
2
+ 1) (A
2
q
2
)] = q
2
+ 1. Pela propriedade de
limitacao, A+q q
2
+1. Nessa desigualdade, nao podemos ter q = 1 pois A > 1. Usando
entao que q > 1, temos A q
2
q + 1 < q
2
, ou seja,

A < q.
Problema 8. (AIME 1986) Qual e o maior inteiro n para o qual n
3
+ 100 e divisvel por
n + 10?
2
POT 2012 - Teoria dos N umeros - Nvel 2 - Aula 1 - Samuel Feitosa
Para achar explicitamente o quociente de n
3
+100 por n+10 podemos fazer uso de alguma
fatora cao. Utilizaremos a soma dos cubos n
3
+ 10
3
= (n + 10)(n
2
10n + 100). Como,
n
3
+ 100 = (n + 10)(n
2
10n + 100) 900,
podemos concluir que o n umero 900 deve ser m ultiplo de n +10. O maior inteiro n para o
qual n + 10 divide 900 e 890. Veja que se n = 890, o quociente da divisao de n
3
+ 100 por
n + 10 e n
2
10n + 100 1 = 890
2
10 890 + 99.
Exemplo 9. (Extrado de [1]) Encontre todos os inteiros positivos n tais que 2n
2
+ 1 |
n
3
+ 9n 17.
Utilizando o 2n
2
+ 1 divide para reduzir o grau de n
3
+ 9n 17, temos que

2n
2
+ 1 | n
3
+ 9n 17
2n
2
+ 1 | 2n
2
+ 1
=2n
2
+ 1 | (n
3
+ 9n 17) 2 + (2n
2
+ 1) (n)
2n
2
+ 1 | 17n 34
Como o grau de 17n 34 e menor do que o de 2n
2
+ 1, podemos utilizar a limita cao
para obter uma lista nita de candidatos a n. Temos 17n 34 = 0 n = 2 ou
|2n
2
+ 1| |17n 34| n = 1, 4 ou 5. Destes candidatos, apenas n = 2 e n = 5 s ao
solucoes.
Exemplo 10. (Leningrado 1990) Sejam a e b n umeros naturais tais que b
2
+ ba + 1 divide
a
2
+ ab + 1. Prove que a = b.
Pela propriedade de limitacao, b
2
+ba+1 a
2
+ab+1 e da b a. Alem disso, b
2
+ab+1 >
a b. A igualdade b(a
2
+ ab + 1) a(b
2
+ ba + 1) = b a implica que a b e divisvel por
b
2
+ba +1. Se a b = 0, entao b
2
+ab +1 a b. Mas isso e um absurdo, logo a b = 0.
Problemas Propostos
Problema 11. Mostre que se 3 | a + 7b entao 3 | a + b.
Problema 12. Mostre que se 7 | a + 3b entao | 13a + 11b
Problema 13. Mostre que se 19 | 3x + 7y entao 19 | 43x + 75y
Problema 14. Mostre que se 17 | 3a + 2b entao 17 | 10a + b
Problema 15. Encontre todos os inteiros positivos n tais que n + 2009 divide n
2
+ 2009 e
n + 2010 divide n
2
+ 2010.
Problema 16. Seja n > 1 e k um inteiro positivo qualquer. Prove que (n1)
2
|(n
k
1) se,
e somente se , (n 1)|k.
3
POT 2012 - Teoria dos N umeros - Nvel 2 - Aula 1 - Samuel Feitosa
Problema 17. (OBM 2005) Prove que a soma 1
k
+ 2
k
+ . . . + n
k
, onde n e um inteiro e k
e mpar, e divisvel por 1 + 2 + . . . + n.
Problema 18. O n umero de seis dgitos X = abcdef satisfaz a propriedade de que abcdef
e divisvel por 7. Prove que X tambem e divisvel por 7.
Problema 19. (Bielor ussia 1996) Inteiros m e n, satisfazem a igualdade
(mn)
2
=
4mn
m + n 1
.
a) Prove que m + n e um quadrado perfeito.
b) Encontre todos os pares (m, n) satisfazendo a equa c ao acima.
Problema 20. (Olimpada de Leningrado) Os n umeros naturais a,b e c tem a propriedade
que a
3
e divisvel por b, b
3
e divisvel por c e c
3
e divisvel por a. Prove que (a +b +c)
13
e
divisvel por abc.
Problema 21. (OBM 2000)

E possvel encontrar duas potencias de 2, distintas e com o
mesmo n umero de algarismos, tais que uma possa ser obtida atraves de uma reordena c ao
dos dgitos da outra? (Dica: Lembre-se do criterio de divisibilidade por 9)
Problema 22. (IMO 1998) Determine todos os pares de inteiros positivos (x, y) tais que
xy
2
+ y + 7 divide x
2
y + x + y.
Dicas e Solu coes
11. Como 3 | 6b, segue que 3 | [(a + 7b) 6b] = a + b.
12. Como 7 | a + 3b, segue que 7 | 13a + 39b = (13a + 11b) + 28b. Mas 7 | 28b, portanto
7 | [(13a + 11b) + 28b 28b] = 13a + 11b.
13. Como 19 | 3x + 7y, segue que 19 | 27(3x + 7y) = (43x + 75y) + (38x + 114y). Mas
19 | 19(2x+6y), portanto 19 | [(43x+75y) +(38x+114y) 19(2x+6y)] = 43x+75y.
14. Como 17 | 3a + 2b, segue que 17 | 27a + 18b = (10a + b) + 17(a + b).
16. Veja que
n
k
1
(n 1)
2
=

n
k1
1
n 1
+
n
k2
1
n 1
+ . . . +
n 1
n 1
+
k
n 1

.
Como os n umeros
n
l
1
n1
sempre s ao inteiros, o n umero do lado esquerdo da equa cao
sera inteiro se, e somente se, o n umero
k
n1
for inteiro.
4
POT 2012 - Teoria dos N umeros - Nvel 2 - Aula 1 - Samuel Feitosa
17. Comece dividindo o problema quando em dois casos: n e par ou n e mpar. Sabemos
que 1 +2 +. . . +n =
n(n+1)
2
. Para n mpar, basta mostrar que o n umero em questao
e divisvel por n e
n+1
2
. O proximo passo e lembrar do problema 33 da aula 1. Pela
fatora cao de x
n
+ y
n
, temos que i
k
+ (n i)
k
e divisvel por n. Fa ca outros tipos de
pares para mostrar a divisibilidade por
n
2
. O caso quando n e par e an alogo.
18. Veja que X = 10
3
abc + def = 1001abc (abc def). Como 1001 e multiplo de 7,
conclumos que X e a soma de dois m ultiplos de 7.
19. Somando 4mn em ambos os lados, obtemos:
(m + n)
2
=
4mn
m + n 1
+ 4mn
=
4mn(m + n)
m + n 1

(m + n) =
4mn
m + n 1
= (mn)
2
.
Assim, m + n e o quadrado de um inteiro. Se m n = t, entao m + n = t
2
e
(m, n) = (
t
2
+t
2
,
t
2
t
2
).

E facil vericar que para qualquer t inteiro esse par e solucao
do problema.
20. Analise a expans ao pelo bin omio de Newton.
21. N ao. Suponha, por absurdo, que existam duas potencias de 2, 2
m
< 2
n
, satisfazendo
o enunciado. Como 2
n
e um m ultiplo de 2
m
, podemos ter: 2
n
= 22
m
, 42
m
, 82
m
, . . ..
Alem disso, como ambos possuem a mesma quantidade de dgitos, temos 1 <
2
n
2
m
<
10. Assim, as unicas possibilidade s ao 2
n
= 2 2
m
, 4 2
m
, 8 2
m
. Pelo criterio de
divisibilidade por 9, como 2
m
e 2
n
possuem os mesmos dgitos, podemos concluir
que 2
n
2
m
e um m ultiplo de 9. Entretanto, nenhuma das possibilidade anteriores
satisfaz essa condi cao e chegamos em um absurdo.
22. Come caremos usando a ideia do exemplo 10. A igualdade y(x
2
y + x + y) x(xy
2
+
y + 7) = y
2
7x implica que y
2
7x e divisvel por xy
2
+ y + 7. Se y
2
7x 0,
como y
2
7x < xy
2
+ y + 7, segue que y
2
7x = 0. Assim, (x, y) = (7t
2
, 7t) para
algum t N.

E facil checar que esses pares s ao realmente solucoes. Se y
2
7x < 0,
entao 7x y
2
> 0 e divisvel por xy
2
+ y + 7. Da, xy
2
+ y + 7 7x y
2
< 7x, que
nos permite concluir que y 2. Para y = 1, temos x+8 | 7x1 e consequentemente
x + 8 | 7(x + 8) (7x 1) = 57. Entao as unicas possibilidades s ao x = 11 e x = 49,
cujos pares correspondentes s ao (11, 1), (49, 1). Para y = 2, temos 4x + 9 | 7x 4
e consequentemente 7(4x + 9) 4(7x 4) = 79 e divisvel por 4x + 9. Nesse caso,
nao obtemos nenhuma solucao nova. Todas as solucoes para (x, y) s ao: (7t
2
, 7t)(t
N), (11, 1) e (49, 1).
5
Referencias
[1] F. E. Brochero Martinez, C. G. Moreira, N. C. Saldanha, E. Tengan - Teoria dos
N umeros um passeio com primos e outros n umeros familiares pelo mundo inteiro,
Projeto Euclides, IMPA, 2010.
[2] E. Carneiro, O. Campos and F. Paiva, Olimpadas Cearenses de Matem atica 1981-2005
(Nveis J unior e Senior), Ed. Realce, 2005.
[3] S. B. Feitosa, B. Holanda, Y. Lima and C. T. Magalhaes, Treinamento Cone Sul 2008.
Fortaleza, Ed. Realce, 2010.
[4] D. Fomin, A. Kirichenko, Leningrad Mathematical Olympiads 1987-1991, MathPro
Press, Westford, MA, 1994.
[5] D. Fomin, S. Genkin and I. Itenberg, Mathematical Circles, Mathematical Words, Vol.
7, American Mathematical Society, Boston, MA, 1966.
[6] I. Niven, H. S. Zuckerman, and H. L. Montgomery, An Introduction to the Theory of
Numbers.
Polos Olmpicos de Treinamento
Curso de Teoria dos Nmeros - Nvel 2
Prof. Samuel Feitosa
Aula
3
O Algoritmo de Euclides
Exemplo 1. Seja S um conjunto innito de inteiros n ao negativos com a seguinte propri-
edade: dados dois quaisquer de seus elementos, o valor absoluto da diferenca entre eles
tambem pertence a S. Se d e o menor elemento positivo de S, prove que S consiste de
todos os m ultiplos de d.
Considere um elemento m qualquer de S. Pelo algoritmo da divisao, m = qd + r com
0 r < d. Como todos os n umeros m d, m 2d, m 3d, . . . , m qd = r pertencem
a S e d e o menor elemento positivo de tal conjunto, devemos ter obrigatoriamente que
r = 0. Sendo assim, podemos concluir que todos os elementos de S s ao m ultiplos de d.
Resta mostrarmos que todos os m ultiplos de d estao em S. Seja kd um m ultiplo positivo
qualquer de d. Como S e innito, existe um inteiro m S tal que m = qd > kd. Assim
todos os n umeros md, m2d, . . . , m(q k)d = kd estao em S.
Deni cao 2. Um inteiro a e um divisor comum de b e c se a | b e a | c. Se b e c n ao sao
ambos nulos, denotaremos por mdc(b, c) o maximo divisor comum de b e c.
Como um inteiro nao nulo possui apenas um n umero nito de divisores, se b e c s ao ambos
nao nulos, o n umero mdc(b, c) sempre existe, isto e, sempre esta bem denido.
Lema 3. (Euclides) Se x = 0, mdc(x, y) = mdc(x, x + y)
Demonstrac ao. Seja d um divisor comum de x e y. Entao d | x + y e consequentemente d
tambem a um divisor comum de x e x + y. Reciprocamente, se f e um divisor comum de
x +y e x, f tambem divide (x +y) y = x e assim f e um divisor comum de x e y. Como
os conjuntos de divisores comuns dos dois pares de n umeros mencionados s ao os mesmos,
o maior divisor comum tambem e o mesmo.
Entao podemos calcular:
mdc(123, 164) = mdc(123, 41) = mdc(41, 123) = mdc(41, 82) = mdc(41, 41) = 41.
POT 2012 - Teoria dos N umeros - Nvel 2 - Aula 3 - Samuel Feitosa
Exemplo 4. Tres maquinas I, R, S imprimem pares de inteiros positivos em tickets. Para
a entrada (x, y), as maquinas I, R, S imprimem respectivamente (xy, y), (x+y, y), (y, x).
Iniciando com o par (1, 2) podemos alcan car
a) (819, 357)?
b) (19, 79)?
Para o item a), calculemos inicialmente mdc(819, 357):
mdc(819, 357) = mdc(462, 357) = mdc(105, 357) = mdc(105, 252) = . . . = mdc(21, 21) = 21.
Pelo Lema de Euclides, o mdc entre os dois n umeros em um ticket nunca muda. Como
mdc(1, 2) = 1 = 21 = mdc(819, 357), nao podemos alcan car o par do item a).
Para o item b), indiquemos com uma operacao de alguma das m aquinas. Veja que:
(2, 1)
R
(3, 1)
S
(1, 3)
R
(4, 3)
R
. . .
R
(19, 3)
S
(3, 19)
R
(22, 19)
R
(41, 19)
R

(60, 19)
R
(79, 19).
Observacao 5. Procurar invariantes sempre e uma boa estrategia para comparar con-
gurac oes diferentes envolvidas no problema. Conra o problema proposto 31.
Deni cao 6. Dizemos que dois inteiros p e q sao primos entre si ou relativamente primos
se mdc(p, q) = 1. Dizemos ainda que a frac ao
p
q
e irredutvel se p e q sao relativamente
primos.
Exemplo 7. (IMO 1959) Prove que
21n + 4
14n + 3
e irredutvel para todo n umero natural n.
Pelo lema de Euclides, mdc(21n+4, 14n+3) = mdc(7n+4, 14n+3) = mdc(7n+1, 7n+2) =
mdc(7n + 1, 1) = 1.
O seguinte lema sera provado na proxima aula.
Lema 8. (Propriedades do MDC) Seja mdc(a, b) = d, entao:
i) Se k = 0, mdc(ka, kb) = kd.
ii) mdc

a
d
,
b
d

= 1.
iii) Se mdc(a, c) = 1, entao mdc(a, bc) = d.
Exemplo 9. (Olimpada Inglesa) Se x e y sao inteiros tais que 2xy divide x
2
+y
2
x, prove
que x e um quadrado perfeito
2
POT 2012 - Teoria dos N umeros - Nvel 2 - Aula 3 - Samuel Feitosa
Se d = mdc(x, y), entao x = da e y = db, com mdc(a, b) = 1. Do enunciado, temos:
2abd
2
| d
2
a
2
+ d
2
b
2
da
d
2
| d
2
a
2
+ d
2
b
2
da
d
2
| da
d | a.
Logo, a = dc, para algum c. Como x | y
2
, obtemos d
2
c | d
2
b
2
, ou seja, c|b
2
e mdc(c, b
2
) = c.
Usando que mdc(a, b) = 1 e que todo divisor comum de b e c tambem e um divisor comum
de a e b, podemos concluir que mdc(c, b) = 1. Usando o item iii) do lema anterior,
mdc(c, b
2
) = 1. Assim, c = 1 e x = d
2
c = d
2
.
Exemplo 10. No planeta X, existem apenas dois tipos de notas de dinheiro: $5 e $78.

E
possvel pagarmos exatamente $7 por alguma mercadoria? E se as notas fossem de $ 3 e $
78?
Veja que 2 78 31 5 = 1 e consequentemente 14 78 217 5 = 7. Basta darmos
14 notas de de $ 78 para recebermos 217 notas de $ 5 como troco na compra de nossa
mercadoria. Usando as notas de $3 e $78 nao e possvel pois o dinheiro pago e recebido
como troco por algo sempre e m ultiplo de 3 e 7 nao e m ultiplo de 3.
Queremos estudar a versao mais geral desse exemplo. Quais s ao os valores que podemos
pagar usando notas de $a e $b? Em particular, estaremos interessados em conhecer qual o
menor valor que pode ser pago. Para responder essa pergunta, precisaremos do algoritmo
de Euclides:
Teorema 11. (O Algoritmo de Euclides) Para os inteiros b e c > 0, aplique sucessivamente
o algoritmo da divisao para obter a serie de equa c oes:
b = cq
1
+ r
1
, 0 < r
1
< c,
c = r
1
q
2
+ r
2
, 0 < r
2
< r
1
,
r
1
= r
2
q
3
+ r
3
, 0 < r
3
< r
2
,
.
.
.
r
j2
= r
j1
q
j
+ r
j
, 0 < r
j
< r
j1
,
r
j1
= r
j
q
j+1
A sequencia de restos n ao pode diminuir indenidamente pois 0 r
i
< r
i1
e existe apenas
um n umero nito de naturais menores que c. Assim, para algum j, obteremos r
j+1
= 0.
O maior divisor comum de b e c sera r
j
, ou seja, o ultimo resto n ao nulo da sequencia de
divisoes acima.
Demonstrac ao. Pelo Lema de Euclides,
mdc(x + qy, y) = mdc(x + (q 1)y, y) = mdc(x + (q 2)y, y) = . . . = mdc(x, y).
3
POT 2012 - Teoria dos N umeros - Nvel 2 - Aula 3 - Samuel Feitosa
Entao,
mdc(b, c) = mdc(c, r
1
) = mdc(r
1
, r
2
) = . . . = mdc(r
j1
, r
j
) = r
j
.
Exemplo 12. Calcule mdc(42823, 6409).
Pelo Algoritmo de Euclides,
42823 = 6 6409 + 4369
6409 = 1 4369 + 2040
4369 = 2 2040 + 289
2040 = 7 289 + 17
289 = 17 17.
Portanto, mdc(42823, 6409) = 17.
Podemos extrair mais informa coes do Algoritmo de Euclides. Para isso, iremos organizar
as equa coes do exemplo acima de outra forma.
Essencialmente, a equa cao mdc(x+qy, y) = mdc(x, y) nos diz que podemos subtrair q vezes
um n umero de outro sem alterar o m aximo divisor comum do par em questao. Realizando
esse procedimento sucessivas vezes, subtraindo o n umero menor do maior, podemos obter
pares com n umeros cada vez menores ate que chegarmos em um par do tipo (d, d). Como o
m aximo divisor comum foi preservado ao longo dessas opera coes, d sera o m aximo divisor
comum procurado. Iremos repetir o exemplo anterior registrando em cada operacao quantas
vezes um n umero e subtraido do outro. Isso sera feito atraves de dois pares de n umeros
auxiliares:
(42823, 6409) | (1, 0)(0, 1)
(4369, 6409) | (1, 6)(0, 1)
(4369, 2040) | (1, 6)(1, 7)
(289, 2040) | (3, 20)(1, 7)
(289, 17) | (3, 20)(22, 147)
(17, 17) | (355, 2372)(22, 147)
Da primeira linha para a segunda, como subtramos 6 vezes o n umero 6409 de 42823,
subtramos 6 vezes o par (0, 1) de (1, 0), obtendo: (1, 0) 6(0, 1) = (1, 6). Se em uma
dada linha, temos:
(x, x + qy)) | (a, b)(c, d);
entao, a proxima linha devera ser:
(x, y) | (a, b)(c aq, d bq);
4
POT 2012 - Teoria dos N umeros - Nvel 2 - Aula 3 - Samuel Feitosa
porque representar a a operacao de subtrairmos q vezes o primeiro n umero do segundo. Veja
que o par (a, b) foi subtraido de (c, d) exatamente q vezes. Os n umeros escritos nos ultimos
dois pares representam os coecientes dos n umeros originais para cada n umero do primeiro
par. Por exemplo, analisando a linha:
(289, 2040) | (3, 20)(1, 7);
obtemos que:
289 = 3 42823 20 6409,
2040 = 1 42823 + 7 6409.
Em cada linha, essa propriedade e mantida pois a mesma subtra cao que e realizada no
primeiro par tambem e realizada entre os dois ultimos pares. Analisando o ultimo par,
podemos escrever 17 como combinacao de 42823 e 6409 de duas formas diferentes:
17 = 22 42823 + 147 6409,
17 = 355 42823 +2372 6409,
Assim, se no planeta X tivessemos apenas notas de $42823 e $6409, poderamos comprar
algo que custasse exatamente $17.
Como conclusao da discuss ao anterior e do algoritmo de Euclides, podemos concluir que:
Teorema 13. (Bachet-B`ezout) Se d = mdc(a, b), entao existem inteiros x e y tais que
ax + by = d.
De fato, a discuss ao anterior tambem nos mostra um algoritmo para encontrarmos x e y.
Voltando `a discuss ao sobre o planeta X, podemos concluir em virtude do teorema anterior
que qualquer valor m ultiplo de d poder a ser pago usando apenas as notas de $a e $b.
Como todo valor pago, necessariamente e um m ultiplo do m aximo divisor comum de a e
b, descobrimos que o conjunto que procur avamos consiste precisamente do conjunto dos
m ultiplos de d.
Observacao 14. (Para professores) A prova mais comum apresentada para o teorema an-
terior baseia-se na analise do conjunto de todas as combinac oes lineares entre a e b e quase
sempre se preocupa apenas com mostrar a existencia de x e y. Acreditamos que o algoritmo
para encontrar x e y facilite o entendimento do teorema para os alunos mais jovens. Entre-
tanto, frequentemente utilizemos apenas a parte da existencia descrita no enunciado. Alem
disso, preferimos discutir um exemplo numerico ao inves de formalizarmos uma prova e
sugerimos que o professor faca o mesmo com mais exemplos em aula.
Exemplo 15. (Olimada Russa 1995) A sequencia a
1
, a
2
, ... de naturais satisfaz mdc(a
i
, a
j
) =
mdc(i, j) para todo i = j Prove que a
i
= i para todo i.
5
POT 2012 - Teoria dos N umeros - Nvel 2 - Aula 3 - Samuel Feitosa
Para qualquer inteiro n, mdc(a
2n
, a
n
) = mdc(2n, n) = n, consequentemente n | a
n
. Seja
d um divisor qualquer de a
n
diferente de n, entao d | mdc(a
d
, a
n
). De mdc(a
d
, a
n
) =
mdc(d, n), podemos concluir que d | n. Sendo assim, todos os divisores de a
n
que s ao
diferentes de n s ao divisores de n. Como ja sabemos que a
n
= nk, para algum k, nao
podemos ter k > 1 pois nk nao divide n e assim conclumos que a
n
= n.
Exemplo 16. Mostre que mdc(2
120
1, 2
100
1) = 2
20
1.
Pelo lema de Euclides,
mdc(2
120
1, 2
100
1) = mdc(2
120
1 2
20
(2
100
1), 2
100
1),
= mdc(2
20
1, 2
100
1),
= mdc(2
20
1, 2
100
1 2
80
(2
20
1)),
= mdc(2
20
1, 2
80
1),
= mdc(2
20
1, 2
80
1 2
60
(2
20
1)),
= mdc(2
20
1, 2
60
1),
= mdc(2
20
1, 2
60
1 2
40
(2
20
1)),
= mdc(2
20
1, 2
40
1),
= mdc(2
20
1, 2
40
1 2
20
(2
20
1)),
= mdc(2
20
1, 2
20
1) = 2
20
1.
Exemplo 17. (Olimpada Russa 1964) Sejam x, y inteiros para os quais a frac ao
a =
x
2
+ y
2
xy
e inteira. Ache todos os possveis valores de a.
A primeira estrategia e cancelar os fatores comuns com o objetivo de reduzir o problema
ao caso em que x e y s ao primos entre si. Seja d = mdc(x, y), com

x = d x
0
y = d y
0
, mdc(x
0
, y
0
) = 1,
entao
a =
x
2
+ y
2
xy
=
x
0
2
+ y
0
2
x
0
y
0

Nessa condi cao, como x


0
divide y
2
0
e y
0
divide x
2
0
, cada um deles e igual a 1, donde
a =
1
2
+ 1
2
1 1
= 2.
6
POT 2012 - Teoria dos N umeros - Nvel 2 - Aula 3 - Samuel Feitosa
Deni cao 18. Os inteiros a
1
, a
2
, . . . , a
n
, todos diferentes de zero, possuem m ultiplo comum
b se a
i
|b para i = 1, 2, . . . , n(note que a
1
a
2
. . . a
n
e um m ultiplo comum). O menor m ultiplo
comum positivo para tal conjunto de inteiros e chamado de mnimo m ultiplo comum e sera
denotado por mmc(a
1
, a
2
, . . . , a
n
).
Proposicao 19. Se a e b sao n ao nulos, entao: mmc(a, b) mdc(a, b) = |ab|.
(A prova desta proposicao tambem sera deixada para a pr oxima secao)
Exemplo 20. (Olimpada Russa 1995) Sejam m e n interios positivos tais que:
mmc(m, n) + mdc(m, n) = m + n.
Prove que um deles e divisvel pelo o outro.
Se d = mdc(m, n), entao podemos escrever m = da e n = db. Pela proposicao anterior,
mmc(m, n) =
d
2
ab
d
= dab.
Temos:
mmc(m, n) + mdc(m, n) mn = 0
dab + d da db = 0
ab + 1 a b = 0
(a 1)(b 1) = 0.
Portanto, ou a = 1 e m | n ou entao b = 1 e n | m.
Exemplo 21. (Torneio das Cidades 1998) Prove que, para quaisquer inteiros positivos a e
b, a equa c ao mmc(a, a + 5) = mmc(b, b + 5) implica que a = b.
Para o item a), como (a + 5) a = 5, temos mdc(a, a + 5) e igual a 1 ou 5. O mesmo vale
para mdc(b, b + 5). Pela proposicao anterior, temos:
mmc(a, a + 5) =
a(a + 5)
mdc(a, a + 5)
,
mmc(b, b + 5) =
b(b + 5)
mdc(b, b + 5)
.
Suponha que mdc(a, a +5) = 5 e mdc(b, b +5) = 1, entao a(a +5) = 5b(b +5). Consequen-
temente, a e m ultiplo de 5 e a(a +5) e m ultiplo de 25. Isso implica que b(b +5) tambem e
m ultiplo de 5 e que mdc(b, b + 5) > 1. Uma contradi cao. Analogamente, nao podemos ter
mdc(a, a + 5) = 1 e mdc(b, b + 5) = 5. Sendo assim, mdc(a, a + 5) = mdc(b, b + 5) e:
a(a + 5) b(b + 5) = 0
(a b)(a + b + 5) = 0.
Como a + b + 5 > 0, conclumos que a = b.
7
POT 2012 - Teoria dos N umeros - Nvel 2 - Aula 3 - Samuel Feitosa
Exemplo 22. Uma maquina f executa operac oes sobre o conjunto de todos os pares de
inteiros positivos. Para cada par de inteiros positivos, ela fornece um inteiro dado pelas
regras:
f(x, x) = x, f(x, y) = f(y, x), (x + y)f(x, y) = yf(x, x + y).
Determine f(2012, 2012! + 1).
Claramente mmc(x, x) = x e mmc(x, y) = mmc(y, x). Usando a proposicao anterior e o
lema de Euclides temos:
(x + y)mmc(x, y) = (x + y)
xy
mdc(x, y)
= y
x(x + y)
mdc(x, x + y)
= y mmc(x, x + y)
Temos entao uma forte suspeita de que f = mmc. Seja S o conjunto de todos os pa-
res de inteiros positivos (x, y) tais que f(x, y) = mmc(x, y), e seja (m, n) o par em S
com a soma m + n minima. Note que todo par da forma (n, n) nao esta em S pois
f(n, n) = n = mmc(n, n). Assim, devemos ter m = n. Suponha sem perda de generalidade
que n > m. Portanto:
nf(m, n m) = [m + (n m)]f(m, n m)
= (n m)f(m, m + (n m))
f(m, n m) =
n m
n
f(m, n)
Como o par (m, m n) nao esta em S, dado que a soma de seus elementos e menor que
m + n, temos:
f(m, n m) = mmc(m, n m)
n m
n
f(m, n) = (n m)mmc(m, m + (n m))
f(m, n) = mmc(m, n)
Uma contradi cao. Desse modo, S deve ser um conjunto vazio e f(x, y) = mmc(x, y)
para todos os pares de inteiros positivos. Como 2012 | 2012!, mdc(2012, 2012! + 1) = 1 e
consequentemente mmc(2012, 2012! + 1) = 2012(2012! + 1).
Problemas Propostos
Problema 23. Calcule:
a) mdc(n, n
2
+ n + 1).
b) mdc(3 2012, 2 2012 + 1).
8
POT 2012 - Teoria dos N umeros - Nvel 2 - Aula 3 - Samuel Feitosa
c) mdc

2
40
+ 1
2
8
+ 1
, 2
8
+ 1

.
Problema 24. Encontre mdc(2n + 13, n + 7)
Problema 25. Prove que a frac ao
12n+1
30n+2
e irredutvel.
Problema 26. Sejam a, b, c, d inteiros n ao nulos tais que adbc = 1. Prove que
a+b
c+d
e uma
frac ao irredutvel.
Problema 27. Mostre que mdc(a
m
1, a
n
1) = a
mdc(m,n)
1.
Problema 28. Mostre que se mdc(a, b) = 1, entao:
mdc(a + b, a
2
ab + b
2
) = 1 ou 3
Problema 29. Dado que mdc(a, 4) = 2, mdc(b, 4) = 2, prove que:
mdc(a + b, 4) = 4.
Problema 30. Prove que, para todo natural n,
mdc(n! + 1, (n + 1)! + 1) = 1.
Problema 31. No exemplo 4, determine todos os pares que podem ser obtidos comecando-se
com o par (1, 2).
Problema 32. Qual o maximo divisor comum do conjunto de n umeros:
{16
n
+ 10n 1, n = 1, 2, 3 . . .}?
Problema 33. A sequencia F
n
de Farey e a sequencia de todos as frac oes irredutveis
a
b
com 0 a b n arranjados em ordem crescente.
F
1
= {0/1, 1/1}
F
2
= {0/1, 1/2, 1/1}
F
3
= {0/1, 1/3, 1/2, 2/3, 1/1}
F
4
= {0/1, 1/4, 1/3, 1/2, 2/3, 3/4, 1/1}
F
5
= {0/1, 1/5, 1/4, 1/3, 2/5, 1/2, 3/5, 2/3, 3/4, 4/5, 1/1}
F
6
= {0/1, 1/6, 1/5, 1/4, 1/3, 2/5, 1/2, 3/5, 2/3, 3/4, 4/5, 5/6, 1/1}
Claramente, toda frac ao
a
b
< 1 com mdc(a, b) = 1, est a em algum F
n
. Mostre que se m/n
e m

/n

sao frac oes consecutivas em F


n
temos |mn

nm

| = 1.
Problema 34. (Resvista Quantum - Jornal Kvant) Todas as frac oes irredutveis cujos de-
nominadores n ao excedem 99 sao escritas em ordem crescente da esquerda para a direita:
1
99
,
1
98
, . . . ,
a
b
,
5
8
,
c
d
, . . .
Quais sao as frac oes
a
b
e
c
d
em cada lado de
5
8
?
9
POT 2012 - Teoria dos N umeros - Nvel 2 - Aula 3 - Samuel Feitosa
Problema 35. (OBM) Para cada inteiro positivo n > 1, prove que 1 +
1
2
+
1
3
+. . . +
1
n
n ao
e inteiro.
Problema 36. Determine todas as solu c oes em inteiros positivos para
1
a
+
1
b
=
1
c
.
Problema 37. Inteiros positivos a e b, relativamente primos, sao escolhidos de modo que
a + b
a b
seja tambem um inteiro positivo. Prove que pelo menos um dos n umeros ab + 1 e
4ab + 1 e um quadrado perfeito.
Problema 38. (IMO 1979) Sejam p, q n umeros naturais primos entre si tais que:
p
q
= 1
1
2
+
1
3
. . .
1
1318
+
1
1319
.
Prove que p e divisvel por 1979.
Respostas, Dicas e Solu coes
23. (a)
mdc(n, n
2
+ n + 1) = mdc(n, n
2
+ n + 1 n(n + 1)),
= mdc(n, 1),
= 1.
(b)
mdc(3 2012, 2 2012 + 1) = mdc(3 2012 (2 2012 + 1), 2 2012 + 1),
= mdc(2012 1, 2 2012 + 1),
= mdc(2012 1, 2 2012 + 1 2(2012 1)),
= mdc(2012 1, 3),
= mdc(2012 1 3 670, 3),
= mdc(2, 3) = 1.
Outra op cao seria observar que o mdc procurado deve dividir o n umero 3(2
2012 + 1) 2(3 2012) = 3 e que 2 2012 + 1 nao e m ultiplo de 3.
(c)
mdc

2
40
+ 1
2
8
+ 1
, 2
8
+ 1

= mdc

2
32
+ 2
24
+ 2
16
+ 2
8
+ 1, 2
8
+ 1

,
= mdc

(2
32
1) + (2
24
+ 1) + (2
16
1) + (2
8
+ 1) + 1, 2
8
+ 1

,
= mdc(1, 2
8
+ 1) = 1.
10
POT 2012 - Teoria dos N umeros - Nvel 2 - Aula 3 - Samuel Feitosa
24.
mdc(2n + 13, n + 7) = mdc(2n + 13 2(n + 7), n + 7),
= mdc(2n + 13 2(n + 7), n + 7),
= mdc(1, n + 7) = 1
25.
mdc(12n + 1, 30n + 2) = mdc(12n + 1, 30n + 2 2(12n + 1)),
= mdc(12n + 1, 6n),
= mdc(12n + 1 2(6n), 6n),
= mdc(1, 6n) = 1
26. Seja f = mdc(a + b, c + d). Entao f | d(a + b) b(c + d) = 1 e consequentemente
f = 1.
27. Veja que
mdc(a
m
1, a
n
1) = mdc(a
mn
1 + (a
n
1)a
mn
, a
n
1)
= mdc(a
mn
1, a
n
1)
O resultado segue aplicando o Algoritmo de Euclides aos expoentes.
28. Seja f = mdc(a + b, a
2
ab + b
2
). Entao f | (a + b)
2
(a
2
ab + b
2
) = 3ab. Se
mdc(f, a) > 0, devemos ter mdc(f, b) > 0 pois f | a + b. O mesmo argumento vale
para mdc(f, b) > 0. Assim, mdc(f, a) = mdc(f, b) = 1. Portanto, f | 3.
30. Pelo lema de Euclides,
mdc(n! + 1, (n + 1)! + 1) = mdc(n! + 1, (n + 1)! + 1 (n + 1)(n! + 1))
= mdc(n! + 1, n)
= mdc(n! + 1 n[(n 1)!], n) = 1
34. Sejam l = mmc{1, 2, . . . , n} e a
i
= l/i. A soma considerada e
a
1
+ a
2
+ . . . + a
n
l
.
Queremos analisar o expoente do fator 2 no numerador e no denominador. Seja k tal
que 2
k
n < 2
k+1
. Entao 2
k
||l e a
i
e par para todo i = 2
k
. Como a
2
k e mpar, segue
que o numerador e mpar enquanto que o denominador e par. Consequentemente a
fracao anterior nao representa um inteiro.
11
36. Sejam d = mdc(a, b), a = dx, b = dy. Consequentemente mdc(x, y) = 1 e podemos
escrever a equa cao como:
1
a
+
1
b
=
1
c

bc + ac = ab
dyc + dxc = d
2
xy
c(x + y) = dxy
Como mdc(xy, x +y) = 1 pois mdc(x, y) = 1, devemos ter xy | c e consequentemente
c = xyk. Assim, d = k(x + y). O conjunto solucao e formado pelas triplas (a, b, c)
onde (a, b, c) = (kx(x + y), ky(x + y), xyk) com mdc(x, y) = 1 e x, y e k inteiros
positivos.
38. Use a identidade de Catalao:
1
1
2
+
1
3

1
4
+ . . .
1
2n
=
1
n + 1
+
1
n + 2
+ . . . +
1
2n
Em seguida, agrupe os termos da forma
1
n + i
+
1
2n i + 1
e analise o numerador da
fracao obtida.
Referencias
[1] S. B. Feitosa, B. Holanda, Y. Lima and C. T. Magalhaes, Treinamento Cone Sul 2008.
Fortaleza, Ed. Realce, 2010.
[2] D. Fomin, A. Kirichenko, Leningrad Mathematical Olympiads 1987-1991, MathPro
Press, Westford, MA, 1994.
[3] D. Fomin, S. Genkin and I. Itenberg, Mathematical Circles, Mathematical Words, Vol.
7, American Mathematical Society, Boston, MA, 1966.
[4] I. Niven, H. S. Zuckerman, and H. L. Montgomery, An Introduction to the Theory of
Numbers.
Polos Olmpicos de Treinamento
Curso de Teoria dos Nmeros - Nvel 2
Prof. Samuel Feitosa
Aula
4
N umeros Primos, MDC e MMC.
Deni cao 1. Um inteiro p > 1 e chamado n umero primo se n ao possui um divisor d
satisfazendo 1 < d < p. Se um inteiro a > 1 n ao e primo, ele e chamado de n umero
composto. Um inteiro m e chamado de composto se |m| n ao e primo.
O proximo teorema nos diz que os primos s ao as pecasfundamentais dos n umeros inteiros:
Teorema 2. Todo inteiro n, maior que 1, pode ser expresso como o produto de n umero
primo.
Demonstrac ao. Se o inteiro n e um primo, entao ele mesmo e o produto de um unico fa-
tor primo. Se o inteiro n nao e primo, existe uma decomposicao do tipo: n = n
1
n
2
com
1 < n
1
< n e 1 < n
2
< n. Repetindo o argumento para n
1
e n
2
, podemos escrever n como
o produto de primos ou podemos obter parcelas menores escrevendo n como um produto
de naturais. Como nao existe uma sucess ao innita de naturais cada vez menores, ap os um
n umero nito de operacoes desse tipo, poderemos escrever n como um produto de n umeros
primos.
Quantos n umeros primos existem?
Teorema 3. (Euclides) Existem innitos n umeros primos.
Demonstrac ao. Suponha, por absurdo, que exita apenas uma quantidade nita de primos:
p
1
, p
2
, . . . , p
n
. Considere o n umero X = p
1
p
1
. . . p
n
+1. Pelo teorema anterior, esse n umero
deve ser o produto de alguns elementos do conjunto de todos os n umeros primos. Entre-
tanto, nenhum dos primos p
i
divide X.
Exemplo 4. Existe um bloco de 1000 inteiros consecutivos n ao contendo nenhum primo?
Sim. Um exemplo e o conjunto 1001! +2, 1001! +3, . . . , 1001! +1001. Veja i | 1001! +i para
todo i = 2, 3, . . . , 1001.
POT 2012 - Teoria dos N umeros - Nvel 2 - Aula 4 - Samuel Feitosa
Exemplo 5. (Torneio das Cidades) Existe um bloco de 1000 inteiros consecutivos contendo
apenas um primo?
Para cada bloco de 1000 n umeros consecutivos, contemos sua quantidade de n umeros pri-
mos. Por exemplo, no bloco 1, 2, 3, . . . , 1000, temos 168 n umeros primos (mas s o usaremos
o fato de que existem mais de dois primos nesse bloco). Comparando os blocos consecuti-
vos k + 1, k + 2, . . . , k + 1000 e k + 2, k + 3, . . . , k + 1001, ou o n umero de n umeros primos
aumenta em uma unidade, ou ca constante ou diminui em uma unidade. Analisando to-
dos os blocos consecutivos desde 1, 2, . . . , 1000 ate 1001! + 2, 1001! + 3, . . . , 1001! + 1001,
o n umero de n umeros primos deve ser igual `a 1 em algum deles. Para ver isso, usare-
mos um argumento de continuidade discreta: Come cando com o n umero 168 e realizando
altera coes de no m aximo uma unidade na quantidade de primos em cada bloco, para che-
garmos no n umero 0, necessariamente deveremos passar pelo n umero 1 em algum momento.
Relembremos um importante resultado da aula passada:
Teorema 6. (Bachet- B`ezout) Se d = mdc(a, b), entao existem inteiros x e y tais que
ax + by = d.
Proposicao 7. Sejam a, b e c inteiros positivos com a | bc e mdc(a, b) = 1. Ent ao, a | c.
Demonstrac ao. Pelo teorema anterior, existem x e y inteiros tais que ax + by = 1. Assim,
acx + bcy = c. Como a | acx e a | bcy, podemos concluir que a | c.
Em particular, se p e um n umero primo e p | ab, entao p | a ou p | b. Podemos usar esse
fato para garantir a unicidade em nosso primeiro teorema, obtendo o importante:
Teorema 8. (Teorema Fundamental da Aritmetica) A fatorac ao de qualquer inteiro n > 1,
em fatores primos, e unica a menos da ordem dos fatores.
Exemplo 9. (R ussia 1995)

E possvel colocarmos 1995 n umeros naturais ao redor de um
crculo de modo que para quaisquer dois n umeros vizinhos a razao entre o maior e o menor
seja um n umero primo?
N ao, e impossvel. Suponha, por absurdo, que isso seja possvel e denotemos por
a
0
, a
1
, . . . , a
1995
= a
0
tais inteiros. Entao, para k = 1, . . . , 1995,
a
k1
a
k
e primo ou o in-
verso de um primo. Suponha que a primeira situacao ocorra m vezes e a segunda ocorra
1995 m vezes entre esses quocientes. Como o produto de todos os n umeros da forma
a
k1
a
k
, para k = 1, . . . , 1995 e igual `a 1, podemos concluir que o produto de m primos deve
ser igual ao produto de 1995 m primos. Em virtude da fatora cao unica, m = 1995 m.
Um absurdo pois 1995 e mpar.
Proposicao 10. Se as fatorac oes em primos de n e m sao:
n = p

1
1
p

2
2
. . . p

k
k
,
m = p

1
1
p

2
2
. . . p

k
k
.
2
POT 2012 - Teoria dos N umeros - Nvel 2 - Aula 4 - Samuel Feitosa
Ent ao, mdc(m, n) = p

1
1
p

2
2
. . . p

k
k
e mmc(m, n) = p

1
1
p

2
2
. . . p

k
k
, onde
i
e o menor dentre
{
i
,
i
} e
i
e o maior dentre {
i
,
i
}.
Proposicao 11. Se a e b sao inteiros positivos, mostre que mmc(a, b)mdc(a, b) = ab.
Demonstrac ao. Basta usar a proposicao anterior e observar que:
max{x, y} + min{x, y} = x + y.
Exemplo 12. (Torneio das Cidades 1998)

E possvel que mmc(a, b) = mmc(a + c, b + c)
para alguma conjunto {a, b, c} de inteiros positivos?
N ao. Suponha que a+c e b+c possuem algum divisor primo p. Como p | mmc(a+c, b+c),
caso existam tais inteiros, devemos ter que p | mmc(a, b). Assim, usando que pelo menos
um dentre a e b e divisvel por p podemos concluir que c tambem e divisvel por p. Entao,
podemos cancelar o fator p:
mmc

a
p
,
b
p

=
mmc(a, b)
p
=
mmc(a + c, b + c)
p
= mmc

a + c
p
,
b + c
p

.
Efetuando alguns cancelamentos, podemos supor entao que a+c e b+c nao possuem fatores
primos em comum. Obtivemos um absurdo pois:
mmc(a + c, b + c) = (a + c)(b + c) > ab mmc(a, b).
Exemplo 13. (OCM 2005) Determinar os inteiros n > 2 que sao divisveis por todos os
primos menores que n.
Como mdc(n, n 1) = 1, se n 1 possui algum fator primo, ele nao dividira n. Assim,
n 1 < 2. Consequentemente nao existe tal inteiro.
Exemplo 14. Mostre que n
4
+ n
2
+ 1 e composto para n >1.
Veja que n
4
+ n
2
+ 1 = n
4
+ 2n
2
+ 1 n
2
= (n
2
+ 1)
2
n
2
= (n
2
+ n + 1)(n
2
n + 1).
Para n > 1, n
2
n +1 = n(n 1) +1 > 1 e assim n
4
+n
2
+1 e o produto de dois inteiros
maiores que 1.
Exemplo 15. Mostre que n
4
+ 4
n
e composto para todo n > 1.
Se n e par, certamente o n umero em quest ao e divisvel por 4. Para o caso em que n e
impar, iremos usar a fatora cao:
a
4
+4b
4
= a
4
+4a
2
b
2
+4b
4
4a
2
b
2
= (a
2
+2b
2
) 4b
2
b
2
= (a
2
2ab +2b
2
)(a
2
+2ab +2b
2
).
Para n da forma 4k + 1, fa ca a = n e b = 4
k
. Para n da forma 4k + 3, fa ca a = n e
b = 2
2k+1
.
Exemplo 16. Se 2
n
+1 e um primo mpar para algum inteiro positivo n, prove que n e uma
potencia de 2.
3
POT 2012 - Teoria dos N umeros - Nvel 2 - Aula 4 - Samuel Feitosa
Ja vimos que a
n
1 = (a 1)(a
n1
+ a
n2
+ . . . + 1). Se n e impar,
(a)
n
1 = (a 1)((a)
n1
+ (a)
n2
+ . . . + 1)
a
n
+ 1 = (a + 1)(a
n1
a
n2
+ . . . a + 1)
Sendo assim, se n possusse algum divisor primo mpar p com n = pb, poderamos escrever:
2
n
+1 = (a+1)(a
n1
a
n2
+. . . a+1), onde a = 2
b
. Como a
n1
a
n2
+. . . a+1 > 1,
o n umero 2
n
+ 1 nao seria primo.
Exemplo 17. Dados que p, p + 10 e p + 14 sao n umeros primos, encontre p.
Vamos analisar os possveis restos na divisao por 3 de p. Se p deixa resto 1, entao p + 14
e um m ultiplo de 3 maior que 3 e consequentemente nao poder a ser um n umero primo. Se
o resto e 2, entao p + 10 e um m ultiplo de 3 maior que 3 e tambem nao poder a ser um
n umero primo. Assim, o resto de p por 3 e 0 e consequentemente p = 3.
Exemplo 18. (

Austria-Polonia) Dados naturais n e a > 3 mpar, mostre que a


2
n
1 tem
pelo menos n + 1 divisores primos distintos.
Usando a fatora cao da diferen ca de quadrados, temos que:
a
2
k
1 = (a
2
k1
+ 1)(a
2
k2
+ 1) . . . (a + 1)(a 1).
Assim, a
2
m
+ 1 | a
2
k
1 se k > m. Como a e impar, podemos concluir que:
mdc(a
2
k
+ 1, a
2
m
+ 1) = mdc(a
2
k
1 + 2, a
2
m
+ 1) = mdc(2, a
2
m
+ 1) = 2.
Sendo assim, na fatora cao:
a
2
n
1
2
n
=
(a
2
n1
+ 1)
2
(a
2
n2
+ 1)
2
. . .
(a + 1)
2
(a 1)
2
,
temos o produto de pelo menos n inteiros primos entre si e consequentemente seus fatores
primos s ao distintos. Para cada termo
(a
2
i
+1)
2
, temos um fator primo p
i+1
diferente de 2.
Da, a
2
n
1 possui pelo menos n + 1 fatores primos distintos, a saber, {2, p
1
, p
2
, . . . , p
n
}.
Exemplo 19. (Rioplatense 1999) Sejam p
1
, p
2
, . . . , p
k
primos distintos. Considere todos os
inteiros positivos que utilizam apenas esses primos (n ao necessariamente todos) em sua
fatorac ao em n umeros primos, formando assim uma seq uencia innita
a
1
< a
2
< < a
n
< .
Demonstre que, para cada natural c, existe um natural n tal que
a
n+1
a
n
> c.
4
POT 2012 - Teoria dos N umeros - Nvel 2 - Aula 4 - Samuel Feitosa
Suponha, por absurdo, que exista c > 0 tal que a
n+1
a
n
c, n N. Isso signica que
as diferen cas entre os termos consecutivos de (a
n
)
n1
pertencem ao conjunto {1, 2, . . . , c},
logo s ao nitas. Sejam d
1
, d
2
, . . . , d
r
essas diferen cas. Seja
i
o maior expoente de p
i
que
aparece na fatora cao de todos os d
j
.
Considere entao o n umero M = p

1
+1
1
p

2
+1
2
p

k
+1
k
.

E claro que M pertence `a seq uencia,
ou seja, M = a
n
, para algum n. Vejamos quem sera a
n+1
. Por hip otese, existe i tal que
a
n+1
a
n
= d
i
. Como a
n+1
> a
n
, existe um primo p
j
que divide a
n+1
com expoente maior
ou igual a
j
+ 1. Caso contrario,
a
n
< a
n+1
< p

1
+1
1
p

2
+1
2
p

k
+1
k
= a
n
,
absurdo. Da, p

j
+1
j
|a
n
p

j
+1
j
|d
i
, novamente um absurdo, pela maximalidade de
j
.
Logo, o conjunto de todas as diferen cas nao pode ser nito e, portanto, dado qualquer
c > 0, existe um natural n tal que a
n+1
a
n
> c.
Problemas Propostos
Problema 20. Dado que p, 2p + 1 e 4p
2
+ 1 sao n umeros primos, encontre p.
Problema 21. Dado o par de primos p e 8p
2
+ 1, encontre p.
Problema 22. Dado o par de primos p e p
2
+ 2, prove que p
3
+ 2 tambem e um n umero
primo.
Problema 23. Dado que p, 4p
2
+ 1 e 6p
2
+ 1 sao n umeros primos, encontre p.
Problema 24. Os n umeros de Fermat sao os n umeros da forma 2
2
n
+ 1. Prove que o
conjunto dos divisores primos dos termos da seq uencia de Fermat e innito.
Problema 25. Mostre que todo inteiro n pode ser escrito de maneira unica na forma n = ab,
onde a e um inteiro livre de quadrado e b e um quadrado perfeito. Um inteiro e dito livre
de quadrado se n ao e divisvel por nenhum quadrado perfeito maior que 1.
Problema 26. Prove que todo primo maior que 3 e da forma 6k+1 ou 6k+5.
Problema 27. Prove que todo inteiro da forma 3k+2 tem um fator primo da mesma forma.
Problema 28. Prove que existem innitos primos da forma 4k+3 e 6k +5.
Problema 29. Prove que se n e composto, entao possui um fator primo p

n.
Problema 30. (OBM 1998) S ao dados 15 n umeros naturais maiores que 1 e menores que
1998 tais que dois quaisquer sao primos entre si. Mostre que pelo menos um desses 15
n umeros e primo.
Problema 31. Mostre que n|(n-1)! para todo n umero composto n.
5
POT 2012 - Teoria dos N umeros - Nvel 2 - Aula 4 - Samuel Feitosa
Problema 32. Suponha que n >1. Mostre que a soma dos inteiros dos inteiros positivos
n ao excedendo n divide o produto dos inteiros positivos n ao excedendo n se, e somente se,
n e composto.
Exemplo 33. (R ussia 1995) Encontre todos os primos p para os quais p
2
+11 tenha exata-
mente seis divisores distintos, incluindo 1 e p
2
+ 11.
Problema 34. (Irlanda 2002 ) Encontre todas as solu c oes inteiras positivas de p(p + 3) +
q(q + 3) = n(n + 3), onde p, q sao primos.
Exemplo 35. Prove que qualquer quadrado perfeito positivo tem mais divisores que deixam
resto 1 na divisao por 3 do que divisores que deixam resto 2 na divisao por 3.
Dicas e Solu coes
19. Analisemos o resto de p na divisao por 3. Se p deixar resto 1, o n umero 2p + 1 sera
divisvel por 3. Se p deixar resto 2, o n umero 4p + 1 sera divisvel por 3. Em ambos
os casos, 2p + 1, 4p + 1 > 3 e obtemos assim um absurdo.
20. Analisemos o resto de p na divisao por 3. Se p deixa resto 1 ou 2, p
2
deixa resto 1
e consequentemente 8p
2
+ 1 deixa resto 0 por 3 mas certamente e maior que 3. Um
absurdo, logo p = 3.
21. Analisemos o resto na divisao por 3. Se p nao e m ultiplo de 3, p
2
+ 2 e divisvel por
3 e maior que 3. Um absurdo, logo p = 3 e p
3
+ 2 = 29.
22. Analise os restos na divisao por 5.
23. Iremos usar a fatora cao do exemplo 17:
2
2
n
1 = (2
2
n1
+ 1)(2
2
n2
+ 1) . . . (2 + 1)(2 1).
Assim, se k > m,
mdc(2
2
k
+ 1, 2
2
m
+ 1) = mdc(2
2
k
1 + 2, 2
2
m
+ 1) = mdc(2, 2
2
m
+ 1) = 1,
produzindo que quaisquer dois n umeros de Fermat distintos s ao primos entre si e isso
necessariamente implica que o conjunto de seus divisores primos e innito.
24. Analise os restos na divisao por 2 e 3.
27. Tente imitar a prova de Euclides para a existencia de innitos primos.
29. Se n e composto, podemos escrever n = ab com 1 < a b <. Assim, a
2
n e
a

n. Para terminar, basta considerar qualquer divisor primo de a.
30. Dado 1 < n < 1998, se ele nao for primo, usando o exerccio anterior, ele tem que
ter um fator primo menor que 1998, ou seja, um fator primo menor que 45. Como s o
existem 14 primos menores que 45, e s ao 15 n umeros, um deles sera primo.
6
31. Escreva n = ab e analise as aparicoes de a e b no produto (n 1) (n 2) . . . 2 1.
33. Se p = 3, 3 | p
2
+ 11. Analogamente, se p = 2, 4 | p
2
+ 11. Assim, exceto nes-
ses dois casos, 12 | p
2
+ 11 e podemos encontrar mais que 6 divisores distintos:
{1, 2, 3, 4, 6, 12, p
2
+11}. Agora, teste p = 2 e p = 3 para vericar que p = 3 e a unica
solucao.
34. Seja
n = 3

1
1
p

n
n
q

1
1
q

m
m
a decomposicao de n em fatores primos, onde cada p
i
deixa resto 1 por 3 e cada q
j
deixa resto 2 por 3. Entao
n
2
= 3
2
p
2
1
1
p
2
n
n
q
2
1
1
q
2
m
m
.
Um divisor de n
2
deixa resto 1 por 3 se e somente se possuir uma quantidade par de
primos q
j
, contados com repeticao. Mais especicamente, se e somente se a soma dos
expoentes de q
1
, . . . , q
m
for par. Assim, a quantidade de divisores dessa forma e igual
a
D
1
= (2
1
+ 1) (2
n
+ 1)

1
2
(2
1
+ 1)(2
2
+ 1) (2
m
+ 1) + 1

.
Enquanto para se obter um divisor que deixe resto 2 por 3, precisamos de uma
quantidade mpar de fatores primos da forma 3k+2. Assim, a quantidade de divisores
dessa forma e:
D
2
:= (2
1
+ 1)(2
2
+ 1) (2
n
+ 1)

1
2
(2
1
+ 1)(2
2
+ 1) (2
m
+ 1)

.
Da, segue facilmente que D
1
> D
2
.
Referencias
[1] E. Carneiro, O. Campos and F. Paiva, Olimpadas Cearenses de Matem atica 1981-2005
(Nveis J unior e Senior), Ed. Realce, 2005.
[2] S. B. Feitosa, B. Holanda, Y. Lima and C. T. Magalhaes, Treinamento Cone Sul 2008.
Fortaleza, Ed. Realce, 2010.
[3] D. Fomin, A. Kirichenko, Leningrad Mathematical Olympiads 1987-1991, MathPro
Press, Westford, MA, 1994.
[4] D. Fomin, S. Genkin and I. Itenberg, Mathematical Circles, Mathematical Words, Vol.
7, American Mathematical Society, Boston, MA, 1966.
[5] I. Niven, H. S. Zuckerman, and H. L. Montgomery, An Introduction to the Theory of
Numbers.
Polos Olmpicos de Treinamento
Curso de Teoria dos Nmeros - Nvel 2
Prof. Samuel Feitosa
Aula
5
Congruencias I
Deni cao 1. Dizemos que os inteiros a e b sao congrentes modulo m se eles deixam o
mesmo resto quando divididos por m. Denotaremos isso por a b (mod m).
Por exemplo, 7 2 (mod 5), 9 3 (mod 6), 37 7 (mod 10) mas 5 3 (mod 4). Veja
que a b (mod m) se, e somente se, m | a b.
Teorema 2. Se a b (mod m) e c d (mod m), entao:
i) a + c b + d (mod m)
ii) a c b d (mod m)
iii) ka kb (mod m) k Z
iv) ac bd (mod m)
v) a
k
b
k
(mod m) k N
vi) Se mdc(k, m) = d, entao ka kb (mod m) a b (mod m/d)
Demonstrac ao. Sejam q
1
e q
2
tais que:
a b = q
1
m
c d = q
2
m
Entao, (a + c) (b + d) = (q
1
+ q
2
)m. Logo, a + c e b + d deixam o mesmo resto por m e
consequentemente a+c b +d (mod m). Usando que ab (mod a)
k
b
k
e que m | ab,
conclumos que m (mod a)
k
b
k
. Os demais itens serao deixados para o leitor.
Em termos praticos, podemos realizar quase todas as opera coes elementares envolvendo
igualdade de inteiros. Uma das diferen cas cruciais e a operacao de divisao como mostra o
ultimo item do teorema anterior.
POT 2012 - Teoria dos N umeros - Nvel 2 - Aula 5 - Samuel Feitosa
Exemplo 3. Calcule o resto de 4
100
por 3.
Como 4 1 (mod 3), temos 4
100
1
100
= 1 (mod 3).
Exemplo 4. Calcule o resto de 4
100
por 5.
Como 4 1 (mod 5), temos 4
100
(1)
100
= 1 (mod 5).
Exemplo 5. Calcule o resto de 4
100
por 7.
Voce deve ter percebido que encontrar rela coes do tipo a 1 (mod m) podem simplicar
bastante o calculo de a
k
(mod m). Procuremos alguma rela cao como essa para 4 e 7. Veja
que:
4
0
1 (mod 7), 4
1
4 (mod 7), 4
2
2 (mod 7), 4
3
1 (mod 7).
Assim,
4
99
= (4
3
)
33
1
33
= 1 (mod 7).
Como 4
3
1 (mod 7), os restos das potencias de 4 na divisao por 7 se repetem periodica-
mente de 3 em 3 pois 4
3k+r
4
3k
4
r
4
r
(mod 7).
Exemplo 6. Qual o resto de 36
36
+ 41
41
na divisao por 77?
Inicialmente devemos perceber que existe uma rela cao entre os n umeros do problema: 36+
41 = 77. Assim:
36 41 (mod 77),
(36)
41
41
41
(mod 77),
36
36
(1 36
5
) 36
36
+ 41
41
(mod 77).
Nosso proximo passo e encontrar o resto de 36
5
na divisao por 77. Como 36 1 (mod 7),
36
5
1 (mod 7). Alem disso, 36 3 (mod 1)1 produzindo 36
5
3
5
1 (mod 1)1.
Como mdc(7, 11) = 1 e ambos dividem 36
5
1, podemos concluir que 77 | 36
5
1. Logo,
36
36
+ 41
41
deixa resto 0 na divisao por 77.
Exemplo 7. Prove que p
2
1 e divisvel por 24 se p e um primo maior que 3.
Se p e um primo maior que 3, p 1 (mod 3) e p 1 (mod 2). Da, p
2
1 (mod 3).
Alem disso, se p = 2k + 1, segue que p
2
= 4k(k + 1) + 1 1 (mod 8) pois k(k + 1) e par.
Como mdc(8, 3) = 1 e ambos dividem p
2
1, segue que 24 | p
2
1.
Exemplo 8. (OCM-2001) Achar o menor natural n tal que 2001 e a soma dos quadrados
de n inteiros
Podemos concluir da solucao do problema anterior que todo todo inteirompar ao quadrado
deixa resto 1 por 8. Usemos isso para estimar o valor de n. Sejam x
1
, x
2
, . . . , x
n
inteiros
mpares tais que:
x
2
1
+ x
2
2
+ . . . x
2
n
= 2001.
2
POT 2012 - Teoria dos N umeros - Nvel 2 - Aula 5 - Samuel Feitosa
Analisando a congruencia m odulo 8, obtemos:
x
2
1
+ x
2
2
+ . . . x
2
n
= 2001 (mod 8)
1 + 1 + . . . + 1 1 (mod 8)
n 1 (mod 8)
Como 2001 nao e quadrado perfeito, nao podemos ter n = 1. O proximo candidado para
n seria 1 + 8 = 9. Se exibirmos um exemplo para n = 9, teremos achado o valor mnimo.
Veja que:
2001 = 43
2
+ 11
2
+ 5
2
+ 1
2
+ 1
2
+ 1
2
+ 1
2
+ 1
2
+ 1
2
.
Exemplo 9. (IMO) Seja s(n) a soma dos dgitos de n. Se N = 4444
4444
, A = s(N) e
B = s(A). Quanto vale s(B)?
Pelo criterio de divisibilidade por 9, N A B (mod 9). Inicialmente calculemos o
resto de N por 9. Como 4444 16 7 (mod 9), precisamos encontrar 7
4444
(mod 9).
Seguindo os metodos dos primeiros exemplos, seria interessante encontrarmos um inteiro r
tal que 7
r
1 (mod 9). O menor inteiro positivo com essa propriedade e r = 3. Como
4444 = 1481 3 + 1, temos:
7
4444
7
14813+1
(7
3
)
1481
7 7 (mod 9).
Nosso proximo passo e estimar o valor de s(B). Como N = 4444
4444
< 10
54444
, A =
s(N) 5 4444 9 = 199980. Alem disso, B = s(A) 1 + 9 5 = 46 e s(B) 12. O unico
inteiro menor ou igual a 12 com resto 7 por 9 e o proprio 7, da s(B) = 7.
Exemplo 10. Prove que 11
n+2
+ 12
2n+1
e divisvel por 133 para qualquer natural n.
Duas rela coes que podemos extrair dos n umeros envolvidos s ao: 14411 = 133 e 13312 =
121. Assim:
144 11 (mod 133),
12
2
11 (mod 133),
12
2n
11
n
(mod 133),
12
2n+1
11
n
12 (mod 133),
12
2n+1
11
n
(121) + 133 11
n
(mod 133),
12
2n+1
11
n+2
(mod 133).
Exemplo 11. Prove que n
5
+ 4n e divisvel por 5 para todo inteiro n
Inicialmente note que n
5
+ 4n = n(n
4
+ 4). Se n 0 (mod 5), nao ha o que fazer. Se
n 1 (mod 5), n
4
+4 1+4 = 0 (mod 5). Finalmente, se n 2 (mod 5), n
2
4 1
(mod 5) e consequentemente n
4
+ 4 1 + 4 = 0 (mod 5).
Exemplo 12. Seja n > 6 um inteiro positivo tal que n 1 e n + 1 sao primos. Mostre que
n
2
(n
2
+ 16) e divisvel por 720. A recproca e verdadeira?
3
POT 2012 - Teoria dos N umeros - Nvel 2 - Aula 5 - Samuel Feitosa
Veja que n e da forma 6k, pois n 1 e n + 1 s ao primos maiores que 3, portanto da forma
6k 1 e 6k + 1, respectivamente. Logo,
n
2
(n
2
+ 16) = 144(9k
4
+ 4k
2
).
Resta provar que 9k
4
+4k
2
e um m ultiplo de 5. Vamos analisar a igualdade acima m odulo
5.
i) Se k 0, 2 ou 3 (mod 5), temos 9k
4
+ 4k
2
0 (mod 5);
ii) Se k 1 (mod 5) n 1 (mod 5), temos n 1 0 (mod 5), um absurdo;
iii) Se k 4 (mod 5) n 4 (mod 5), temos n+1 0 (mod 5), novamente um absurdo.
Isso conclui a demonstra cao. A recproca nao e verdadeira. Basta tomar, por exemplo,
n = 90.
Problemas Propostos
Problema 13. Determine o resto de 2
20
1 na divisao por 41.
Problema 14. Qual o resto de 1
2000
+ 2
2000
+ . . . + 2000
2000
na divisao por 7?
Problema 15. Qual o resto na divisao de 2
70
+ 3
70
por 13?
Problema 16. Qual o resto de 3
200
por 100?
Problema 17. (Estonia 2000) Determine todos os possveis restos da divisao de um qua-
drado de um n umero primo com o 120 por 120.
Problema 18. Qual o ultimo dgito de 777
777
?
Exemplo 19. Prove que 2222
5555
+ 5555
2222
e divisvel por 7.
Problema 20. Prove que o n umero n
3
+ 2n e divisvel por 3 para todo natural n.
Problema 21. Prove que n
2
+ 1 n ao e divisvel por 3 para nenhum n inteiro.
Problema 22. Prove que n
3
+ 2 n ao e divisvel por 9 para nenhum n inteiro.
Problema 23. Prove que p
2
q
2
e divisvel por 24 se p e q sao primos maiores que 3.
Problema 24. Prove que se 2n + 1 e 3n + 1 sao ambos quadrados perfeitos, entao n e
divisvel por 40.
Problema 25. Prove que 7|2
2n+1
+ 3
n+2
.
Problema 26. Seja d(n) a soma dos dgitos de n. Suponha que n+d(n) +d(d(n)) = 1995.
Quais os possveis restos da divisao de n por 9?
Problema 27. Prove que n ao existem inteiros positivos x
1
, x
2
, . . . , x
14
tais que:
x
4
1
+ x
4
2
+ . . . + x
4
14
= 1599.
4
Problema 28. Escreva uma unica congruencia que e equivalente ao par de congruencias
x 1 (mod 4) e x 2 (mod 3).
Problema 29. Prove que 20
15
1 e divisvel por 11 31 61
Problema 30. (Alemanha 1997) Determine todos os primos p para os quais o sistema
p + 1 = 2x
2
p
2
+ 1 = 2y
2
tem uma solu c ao nos inteiros x, y.
Problema 31. Mostre que se n divide um n umero de Fibonacci entao ele dividir a uma
innidade.
Dicas e Solu coes
13 - 31 Em breve.
Referencias
[1] E. Carneiro, O. Campos and F. Paiva, Olimpadas Cearenses de Matem atica 1981-2005
(Nveis J unior e Senior), Ed. Realce, 2005.
[2] S. B. Feitosa, B. Holanda, Y. Lima and C. T. Magalhaes, Treinamento Cone Sul 2008.
Fortaleza, Ed. Realce, 2010.
[3] D. Fomin, A. Kirichenko, Leningrad Mathematical Olympiads 1987-1991, MathPro
Press, Westford, MA, 1994.
[4] D. Fomin, S. Genkin and I. Itenberg, Mathematical Circles, Mathematical Words, Vol.
7, American Mathematical Society, Boston, MA, 1966.
[5] I. Niven, H. S. Zuckerman, and H. L. Montgomery, An Introduction to the Theory of
Numbers.
Polos Olmpicos de Treinamento
Curso de Teoria dos Nmeros - Nvel 2
Prof. Samuel Feitosa
Aula
6
Congruencias II
Na aula de hoje, aprenderemos um dos teoremas mais importantes do curso: o pe-
quenoteorema de Fermat. Come caremos relembrando um resultado da aula passada:
Lema 1. Se ka kb (mod m) e mdc(m, k) = 1, entao a b (mod m).
Demonstrac ao. Como m | k(a b) e mdc(m, k) = 1, segue que m | a b.
Teorema 2. (Teorema de Fermat) Seja p um primo. Se p n ao divide a entao
a
p1
1 (mod p).
Alem disso, para todo inteiro a, a
p
a (mod p)
Demonstrac ao. Considere o conjunto de inteiros B = {a, 2a, 3a, . . . , (p 1)a} onde a e um
inteiro satisfazendo mdc(a, p) = 1. Nenhum deles e divisvel por p e quaisquer dois deles
s ao incongruentes m odulo p, em virtude do lema anterior. Assim, o conjunto dos restos
dos elementos de B coincide com o conjunto dos restos nao nulos na divisao por p, a saber,
{1, 2, 3, . . . , p 1}. Portanto,
a 2a 3a . . . (p 1)a 1 2 3 . . . (p 1) (mod p),
a
p1
(p 1)! (p 1)! (mod p).
Podemos cancelar o termo (p 1)! em ambos os lados pois mdc((p 1)!, p) = 1, concluindo
assim a demonstra cao do teorema.
Exemplo 3. Prove que
n
5
5
+
n
3
3
+
7n
15
e um inteiro para todo inteiro n.
Primeiramente note que
n
5
5
+
n
3
3
+
7n
15
=
3n
5
+ 5n
3
+ 7n
15
. Como mdc(3, 5) = 1, basta
mostrarmos que o numerador e m utiplo de 3 e 5. Pelo teorema de Fermat:
3n
5
+ 5n
3
+ 7n 5n
3
+ 7n 5n + 7n = 12n 0 (mod 3),
3n
5
+ 5n
3
+ 7n 3n
5
+ 7n 3n + 7n = 10n 0 (mod 5).
POT 2012 - Teoria dos N umeros - Nvel 2 - Aula 5 - Samuel Feitosa
Problema 4. Mostre que n
7
n (mod 42), n N
Pelo teorema de Fermat,
n
7
n (mod 7)
n
7
(n
3
)
2
n n
2
n = n
3
n (mod 3)
n
7
(n
2
)
3
n n
3
n = (n
2
)
2
n
2
n (mod 2)
Como 2, 3 e 7 s ao primos entre si, n
7
n (mod 2 3 7 = 42).
Exemplo 5. (Bulgaria 95) Encontre o n umero de inteiros n > 1 para os quais o n umero
a
25
a e divisvel por n para cada inteiro a.
Se n satisfaz o enunciado, p
2
(p primo) nao pode divid-lo, pois p
25
p nao e divisvel
por p
2
. Assim, n e m ultiplo de primos diferentes. Os fatores primos de n s ao fatores de
2
25
2 = 2 3
2
5 7 13 17 241. Entretanto, n nao e divisvel por 17 e 241 pois 3
25
3
(mod 17) e 3
25
32 (mod 241). Seguindo o exemplo anterior, podemos usar o teorema de
Fermat para mostrar que a
25
a (mod p) para p {2, 3, 5, 7, 13}. Portanto, n deve ser
igual a um dos divisores de 2 3 5 7 13 diferente de 1. A quantidade de tais divisores e
2
5
1 = 31.
Exemplo 6. Prove que para cada primo p, a diferenca
111 . . . 11222 . . . 22333 . . . 33 . . . 888 . . . 88999 . . . 99 123456789
(onde cada digito est a escrito exatamente p vezes) e m ultiplo de p.
Uma boa maneira de associar os n umeros do problema com o teorema de Fermat e perceber
que:
111 . . . 11

p uns
=
10
p
1
9
.
Assim, podemos escrever o n umero S = 111 . . . 11222 . . . 22333 . . . 33 . . . 888 . . . 88999 . . . 99
como:
S =
10
p
1
9
10
8p
+ 2
10
p
1
9
10
7p
+ . . . 9
10
p
1
9
9S = (10
p
1) 10
8p
+ 2 (10
p
1) 10
7p
+ . . . 9 (10
p
1)
Para p = 2 ou p = 3, o resultado do enunciado segue dos criterios de divisibilidade por 2 e
3. Podemos entao nos concentrar no caso p > 3. Nesse caso, e suciente mostrarmos que
9(S 123456789) e divisvel por p pois mdc(p, 9) = 1. Pelo teorema de Fermat:
9S = (10
p
1) 10
8p
+ 2 (10
p
1) 10
7p
+ . . . 9 (10
p
1)
(10 1) 10
8
+ 2 (10 1) 10
7
+ . . . 9 +(10 1) (mod p)
9 123456789 (mod p).
2
POT 2012 - Teoria dos N umeros - Nvel 2 - Aula 5 - Samuel Feitosa
Exemplo 7. Dado um primo p, prove que existem innitos naturais n tais que p divide
2
n
n.
Se p = 2, n pode ser qualquer n umero par. Suponha que p > 2. Considere (p 1)
2k
, pelo
teorema de Fermat temos:
2
(p1)
2k
(2
p1
)
(p1)
2k1
1
(p1)
2k1
= 1 (p 1)
2k
(mod p).
Assim, para qualquer k, n = (p 1)
2k
satisfaz o problema.
Lema 8. Se mdc(a, m) = 1 entao existe um inteiro x tal que
ax 1 (mod m).
Tal x e unico modulo m. Se mdc(a, m) > 1 entao n ao existe tal x.
Demonstrac ao. Pelo teorema de Bachet-Bezout, existem inteiros x e y tais que ax+my = 1.
Analisando essa congruencia m odulo m, obtemos ax 1 (mod m). Se y e outro inteiro
que satisfaz a congruencia, temos ax ay (mod m). Pelo primeiro lema, x y (mod m).
Se d = mdc(a, m) > 1, nao podemos ter d | m e m | ax 1 pois d ax 1.
Teorema 9. (Teorema de Wilson) Se p e primo, entao
(p 1)! 1 (mod p)
Demonstrac ao. Em virtude do lema anterior, para cada a {2, 3, . . . , p 2}, existe um
resto x {0, 1, 2, . . . , p1} tal que ax 1 (mod p). Se x = 1 ou x = p1, teramos a = 1
ou p 1. Alem disso, nao podemos ter a = x pois os unicos restos que satisfazem a
2
1
(mod p) s ao 1 e p 1 (Veja o problema 20). Com isso, podemos agrupar os n umeros de
{2, 3, . . . , p 2} em pares onde o produto deixa resto 1 por p, o que nos permite concluir
que o produto de todos eles tambem deixa resto 1 por p. Logo,
(p 1)! 1 (p 1) 1 (mod p).
Exemplo 10. (Estonia 2000) Prove que n ao e possvel dividir qualquer conjunto de 18
inteiros consecutivos em dois conjuntos disjuntos A e B tais que o produtos dos elementos
de A seja igual ao produto dos elementos de B.
Suponha, por absurdo, que existam tais conjuntos. Considere o primo p = 19. Como o
produtos dos elementos de A e igual ao produtos dos elementos de B, se um dos conjuntos
contem um m ultiplo de 19, o outro necessariamente tambem conter a. Como entre 18
inteiros consecutivos nao existem dois m ultiplos de 19, nenhum dos conjuntos do problema
contem tais n umeros. Seja x o resto na divisao por 19 dos produtos dos elementos de A.
Calculemos entao o resto na divisao por 19 do produto de todos os 18 inteiros consecutivos:
x x n(n + 1)(n + 2)(n + 3) . . . (n + 17)
1 2 3 . . . 18
1 (mod 19)(Pelo teorema de Wilson).
Como x
2
1 (mod 19), x
18
(1)
9
1 (mod 1)9. Isso contraria o teorema de Fermat
e obtemos um absurdo.
3
POT 2012 - Teoria dos N umeros - Nvel 2 - Aula 5 - Samuel Feitosa
Deni cao 11. Um conjunto S e chamado de sistema completo de resduos modulo n, de-
notado abreviadamente por scr, se para cada 0 i n 1, existe um elemento de s S
tal que i s (mod n). Para qualquer a, o conjunto {a, a + 1, a + 2, . . . , a + (n 1)} e um
exemplo de scr.
Exemplo 12. Se mdc(m, s) = 1, mostre que {t, t + s, t + 2s, . . . t + (m1)s} e um scr.
Pelo primeiro lema, se t + is t + js (mod m), temos is js (mod m) e i j (mod m).
Como i, j {0, 1, . . . , m 1}, i = j. Isso nos diz que temos m inteiros que deixam restos
distintos na divisao por m. Como existem exatamente m restos na divisao por m, o conjunto
e um scr.
Exemplo 13. Seja m um inteiro positivo par. Suponha que {a
1
, a
1
, . . . , a
m
} e {b
1
, b
2
, . . . , b
m
}
sao dois sistemas completos de resduso modulo m. Prove que
S = {a
1
+ b
1
, a
2
+ b
2
, . . . , a
m
+ b
m
}
n ao e um sistema completo de resduos.
Suponha que S seja um scr, entao:
1 + 2 + . . . + m (a
1
+ b
1
) + (a
2
+ b
2
) + . . . + (a
n
+ b
n
) (mod m)
(a
1
+ a
2
+ . . . + a
n
) + (b
1
+ b
2
+ . . . + b
n
)
2(1 + 2 + . . . + n)
2(1 + 2 + . . . + m)
Isso implica que m |
m(m + 1)
2
, ou seja,
m + 1
2
e inteiro. Um absurdo pois m e par.
Exemplo 14. (Polonia 1997) Prove que a sequencia a
n
denida por a
1
= 1 e
a
n
= a
n1
+ a

n
2

contem innitos termos divisveis por 7.


Uma maneira natural para mostrarmos que existem innitos inteiros m ultiplos de 7 na
sequencia e vericar que o aparecimento de um m ultiplo de 7 acarreta o aparecimento de
outro m ultiplo na sequencia com um ndice maior. Suponha que a
k
e m ultiplo de 7. Seja
a
2k1
= s. Entao:
a
2k1
= s
a
2k
= s + a
k
s (mod 7)
a
2k+1
= a
2k
+ a
k
s (mod 7)
4
POT 2012 - Teoria dos N umeros - Nvel 2 - Aula 5 - Samuel Feitosa
Ou seja, o aparecimento de um inteiro m ultiplo de 7 implica no aparecimento de 3 inteiros
com o mesmo resto por 7. Exploremos essa ideia mais uma vez.
a
4k3
= t
a
4k2
t + a
2k1
t + s (mod 7)
a
4k1
t + s + a
2k1
t + 2s (mod 7)
a
4k
t + 2s + a
2k
t + 3s (mod 7)
a
4k+1
t + 3s + a
2k
t + 4s (mod 7)
a
4k+2
t + 4s + a
2k+1
t + 5s (mod 7)
a
4k+3
t + 5s + a
2k+2
t + 6s (mod 7)
Se s e m ultiplo de 7, ja teremos conseguido outro m ultiplo de 7 na sequencia. Em caso
contrario, o conjunto {t, t + s, t + 2s, . . . , t + 6s} e um scr e conter a um m ultiplo de 7.
Exemplo 15. Sejam x, y inteiros. Prove que 3x
2
+ 4y
2
e 4x
2
+ 3y
2
n ao podem ser ambos
quadrados perfeitos.
Comecemos com um lema bastante util:
Lema 16. Seja p um n umero primo da forma 4k + 3. Ent ao
p | m
2
+ n
2
p | m e p | n.
Fa camos inicialmente a primeira implicacao. Se p m, entao m
p1
1 (mod p), e da
temos as equivalencias m odulo p
n
2
m
2
(nm
p2
)
2
(m
p1
)
2
1
(nm
p2
)
p1
(1)
p1
2
(1)
2k+1
1,
o que contraria o teorema de Fermat. Assim, p | m e p | n.
A recproca e obvia. Voltando ao problema, suponha que existam w, z inteiros positivos
tais que
3x
2
+ 4y
2
= w
2
e
4x
2
+ 3y
2
= z
2
.
Entao 7x
2
+7y
2
= w
2
+z
2
(). Armamos que a equa cao () nao possui solucao. Para isso,
seja S o conjunto formado pelas solucoes inteiras (x, y, w, z) de (), e tome (a, b, c, d) S
5
POT 2012 - Teoria dos N umeros - Nvel 2 - Aula 5 - Samuel Feitosa
com c
2
+ d
2
mnimo. Pelo lema, temos que 7|c e 7|d, e da c = 7c

e d = 7d

. Mas entao
a
2
+ b
2
= 7c
2
+ 7d
2
(c

, d

, a, b) S, com
a
2
+ b
2
< 7(a
2
+ b
2
) = c
2
+ d
2
,
o que contraria a minimalidade de (a, b, c, d).
Problemas Propostos
Problema 17. Prove que se p e primo entao
a
p
b
p
(mod p) a
p
b
p
(mod p
2
)
Problema 18. Encontre os restos da divisoes de:
a) 300
3000
1 por 1001
b) 7
120
1 por 143
Problema 19. Encontre o resto de 111 . . . 11

p1 uns
por p, onde p e um primo maior que 5.
Problema 20. Prove que se n e mpar, entao n
5
n (mod 240).
Problema 21. Sejam p e q primos distintos. Mostre que
i) (a + b)
p
a
p
+ b
p
(mod p)
ii) p
q
+ q
p
p + q (mod pq)
iii)

p
q
+ p
q
pq

e par se p, q = 2.
Problema 22. Mostre que se p e primo e a
2
b
2
(mod p), entao a b (mod p).
Problema 23. Encontre os ultimos tres dgitos de 7
9999
Problema 24. Prove que 20
15
1 e divisvel por 11 31 61
Problema 25. Sejam {a
1
, a
2
, ..., a
101
} e {b
1
, b
2
, ..., b
101
} sistemas completos de resduos
modulo 101. Pode {a
1
b
1
, a
2
b
2
, ..., a
101
b
101
} ser um sistema completo de resduos modulo
101?
Problema 26. (Balc anica 2003) Existe um conjunto B de 4004 inteiros positivos tal que,
para cada subconjunto A de B com 2003 elementos, a soma dos elementos em A n ao e
divisvel por 2003?
Problema 27. Para um inteiro mpar n > 1, seja S o conjunto de inteiros x,1 x n,
tal que ambos x e x + 1 sao relativamente primos com n. Mostre que o produto de todos
os elementos de S deixa resto 1 na divisao por n.
6
POT 2012 - Teoria dos N umeros - Nvel 2 - Aula 5 - Samuel Feitosa
Problema 28. Sejam n um inteiro positivo maior que 1 e p um primo positivo tal que n
divide p 1 e p divide n
3
1. Mostre que 4p 3 e um quadrado perfeito.
Dicas e Solu coes
17. Pelo teorema de Fermat, a a
p
b
p
b (mod p). Assim,
a
p1
+ a
p2
b + . . . + ab
p2
+ b
p1
a
p1
+ a
p1
+ . . . + a
p1
pa
p1
0 (mod p)
Como a b 0 (mod p), temos:
a
p
b
p
= (a b)(a
p1
+ a
p2
b + . . . + ab
p2
+ b
p1
) 0 (mod p
2
)
19. Veja que:
111 . . . 11

p1 uns
=
999 . . . 99
9
=
10
p1
1
9
Pelo teorema de Fermat, o numerador 10
p1
1 e divisvel por p visto que p = 5.
Alem disso, usando que p = 2 e 3, segue que
10
p1
1
9
tambem e m ultiplo de p.
20. Proceda como no exemplo 20.
21. i)Pelo teorema de Fermat:
(a + b)
p
a + b
a
p
+ b
p
(mod p).
ii) Pelo teorema de Fermat,
p
q
+ q
p
0 + q p + q (mod p)
p
q
+ q
p
p + 0 p + q (mod q)
22. Veja que (ab)(a+b) 0 (mod p) e assim ab 0 (mod p) ou a+b 0 (mod p).
25. Suponha, por abusurdo, que seja possvel. Sejam a
i
e b
j
tais que a
i
b
j
0
(mod 101). Se i = j, o conjunto {a
1
b
1
, a
2
b
2
, ..., a
101
b
101
} teria dois inteiros com resto
7
POT 2012 - Teoria dos N umeros - Nvel 2 - Aula 5 - Samuel Feitosa
0 na divisao por p e nao poderia ser um scr. Suponha sem perda de generalidade que
i = j = 101, entao:
100! (a
1
b
1
)(a
2
b
2
) . . . (a
100
b
100
)
(a
1
a
2
. . . a
100
)(b
1
b
2
. . . b
100
)
(100!)(100!)
(100!)
2
(mod 101)
Assim, 100! 1 (mod 101). Isso contradiz o teorema de Wilson.
26. Sim. Um exemplo de tal conjunto e a uni ao de um conjunto de 2002 inteiros positivos
que deixem resto 0 com outro conjunto composto por 2002 inteiros que deixem resto
1 por 2003.
8
Polos Olmpicos de Treinamento
Curso de Teoria dos Nmeros - Nvel 2
Prof. Samuel Feitosa
Aula
7
Aula de Revisao e Aprofundamento
Observacao 1.

E recomendavel que o professor instigue seus alunos a pensarem nos pro-
blemas abaixo antes de resolve-los na aula.
Exemplo 2. (ASHME 1990) Para quantos inteiros N entre 1 e 1990 a frac ao
N
2
+ 7
N + 4
n ao
e irredutvel?
Seja d = (N +4, N
2
+7). Um boa estrategia e procurar um m ultiplo de N +4 proximo de
N
2
+7 pois assim conseguiremos estimar d. Usando a diferen ca de quadrados, d | N
2
16 e
consequentemente d | N
2
+7(N
2
16) = 23. Como 23 e primo, a fra cao nao sera irredutvel
apenas quando d = 23. Para isso acontecer, basta que 23 | N+4 pois N
2
+7 = N
2
16+23.
O maior m ultiplo de 23 menor que 1990 e 1978 = 23 86 e 1990 +4 < 23 87. Sendo assim,
a quantidade de inteiros procurada e 86.
Exemplo 3. Dados os primos p e q satisfazendo:
q | p
2
+ 1 e p | q
2
1.
Prove que o n umero p + q + 1 e composto.
Como p | q
2
1 = (q +1)(q 1), temos que p | q +1 ou p | q 1. No primeiro caso, p+q +1
e um m ultiplo de p. No segundo caso, podemos escrever q 1 = pk para algum natural k.
Usando que q | p
2
+ 1, conclumos que q | p
2
+ 1 (pk + 1) = p(p k). Como p e q nao
podem ser primos iguais, q | p k. Temos tres casos a considerar:
1. p > k. Entao:
p k kp + 1,
(p + 1)(1 k) 2
2. p < k. Entao:
k p kp + 1,
(k + 1)(1 p) 2
3. p = k e q = p
2
+1. Como o unico primo par e 2, segue que p = 2, q = 5 e p+q+1 = 8.
Os dois primeiros casos conduzem a um absurdo. Logo, ou p + q + 1 e m ultiplo de p ou e
igual `a 8.
Exemplo 4. (AIME 1985) Os n umeros da sequencia
101, 104, 109, 116, . . .
sao da forma a
n
= 100 + n
2
, onde n = 1, 2, 3, . . . Para cada n, seja d
n
o maximo divisor
comum de a
n
e a
n+1
. Encontre o valor maximo de d
n
quando n varia sobre todo o conjunto
dos inteiros positivos.
Uma boa estrategia e buscar alguma fatora cao que nos permita identicar fatores comuns
entre os termos da sequencia. Um termo generico da sequencia possui a forma a
n
=
k + n
2
. Sendo assim, a
2k
= k(4k + 1),a
2k+1
= (k + 1)(4k + 1) e consequentemente
mdc(a
2k
, a
2k+1
) = 4k + 1. Nosso proximo passo sera mostrar que realmente esse e o valor
m aximo. Considere dois termos genericos a = a
n
= k + n
2
, b = a
n+1
= k + (n + 1)
2
e seja
d = mdc(a, b). Usando que d | b a = 2n+1, obtemos d | (2n+1)(b a) = 4n
2
1. Como
d | 4a = 4n
2
+ 4k, segue que d | 4n
2
+ 4k (4n
2
1) = 4k + 1. Assim, 4k + 1 e realmente
o maior valor possvel entre os termos da sequencia d
n
.
Exemplo 5. Prove que para qualquer inteiro n > 1, o n umero n
5
+n
4
+1 n ao e um n umero
primo.
Considere a fatora cao:
n
5
+ n
4
+ 1 = n
5
+ n
4
+ n
3
n
3
n
2
n + n
2
+ n + 1
= n
3
(n
2
+ n + 1) n(n
2
+ n + 1) + (n
2
+ n + 1)
= (n
2
+ n + 1)(n
3
n + 1)
Como n > 1, n
3
n + 1 > 1 e obtemos assim o produto de dois inteiros maiores que 1.
Exemplo 6. (Olimpada Grega) Encontre todos os inteiros n para os quais 5
4
+5
5
+5
n
e
um quadrado perfeito.
Como 5
4
+ 5
5
= 2500, queremos encontrar m e n tais que:
5
n
= m
2
2500 = (m50)(m + 50).
Isto implica que m + 50 = 5
j
e m50 = 5
i
, com i < j. Assim,
100 = 5
i
(5
ji
1).
Usando a fatora cao em primos de 100, encontramos que i = 2 e j i = 1. Portanto, m = 75
e n = 5.
2
Exemplo 7. (Irlanda) Sejam p um n umero primo, a e n e inteiros positivos. Prove que se
2
p
+ 3
p
= a
n
,
entao n = 1.
Se p = 2, claramente a = 13 e n = 1. Se p > 2, p e mpar e 5 | 2
p
+ 3
p
. Consequentemente
5 divide a. Se fosse n > 1, 25 | a
n
e teramos:
0
a
n
5

2
p
+ 3
p
5
= 2
p1
2
p2
3 + . . . + 2 3
p2
+ 3
p1
2
p1
+ 2
p1
+ . . . + 2
p1
p2
p1
(mod 5)
A unica possbilidade e termos p = 5. Entretanto, 2
5
+3
5
nao e uma potencia perfeita nao
trivial. Logo, n = 1.
Exemplo 8. Um inteiro n > 1 tem a seguinte propriedade: para todo divisor positivo d de
n, d + 1 e um divisor de n + 1. Prove que n e primo.
Seja p o menor fator primo de n e seja d =
n
p
. Entao,
np + p
n + p
=
p(n + 1)
p(d + 1)
=
n + 1
d + 1
e um n umero inteiro. Como n + p tambem divide p(n + p), podemos concluir que n + p |
(np + p
2
) (np + p) = p
2
p. Em particular,
n + p p
2
p
n p
2
2p
n p
2
2p + 1 = (p 1)
2
n < p
2
d < p.
Em virtude da minimalidade de p, d nao possui fatores primos e consequentemente n = p.
Exemplo 9. (Olimpada Russa) Mostre que qualquer natural pode ser escrito como a dife-
renca de dois n umeros naturais tendo o mesmo n umero de fatores primos.
Se n e par, podemos escreve-lo como 2n n e e facil vericar que 2n e n possuem o
mesmo n umero de fatores primos. Seja d o menor primo mpar que nao divide n. Escreva
n = dn (d 1)n. O termo dn contem exatamente um primo a mais que n. Pela escolha
de d, todos os outros fatores primos diferentes 2 do n umero d1 s ao divisores de n e assim
(d 1)n tambem contem extamente um primo a mais que n, a saber, o primo 2.
3
Exemplo 10. Os n umeros naturais a e b sao tais que
a + 1
b
+
b + 1
a
e um n umero inteiro. Mostre que o maximo divisor comun de a e b n ao e maior que

a + b.
Seja d = mdc(a, b), com a = md e b = nd. Entao:
md + 1
nd
+
nd + 1
md
=
m
2
d + m + n
2
d + n
mnd
e um inteiro. Em particular, d | m
2
d + m + n
2
d + n e consequentemente d | m + n. Da,
d m + n

m + n
d

d(m + n)
=

a + b
Exemplo 11. Encontre todos os conjuntos A N de pelo menos dois elementos tais que
x, y A =
x + y
mdc(x, y)
A.
Vamos dividir o problema em dois casos: Primeiro caso: Se 1 A.
Mostremos que nesse caso devemos ter A = {2, 3, . . .}. Fa camos isso seguindo as seguintes
armacoes:
1. 2 A.
Para ver isso, basta tomar dois elementos iguais.
2. Existe elemento mpar em A.
Suponha, por absurdo, que nao existe elementompar em A. Seja 2k o menor elemento
de par de A maior que 2. Logo,
k + 1 =
2k + 2
mdc(2k, 2)
A.
Como k + 1 < 2k, k + 1 e mpar. Absurdo! Note que todos os mpares maiores que
k + 1 pertencem a A. Para tal, basta escolhermos 2 e k + 1 para obtermos k + 3 e
aplicar isso sucessivamente.
3. 3 A.
4
Tome 2l 1 > 2k + 1. Assim,
(2l 1)(2k + 1), (2k + 1) A
(2l 1)(2k + 1) + (2k + 1)
2k + 1
= 2l A;
2l 1, 2l + 1 A
2l 1 + 2l + 1
mdc(2l 1, 2l + 1)
= 4l A;
2l, 4l A
4l + 2l
mdc(4l, 2l)
= 3 A
Isso mostra que todos os mpares maiores ou iguais a 3 estao presentes. Para isso,
basta tomar cada mpar e o 2.
4. Todos os n umeros pares estao em A.
Para isso, tome 2k 1 e (2k 1)
2
:
(2k 1)
2
+ (2k 1)
2k 1
= 2k A,
para todo o k 2.
Segundo caso: Se 1 A.
Nesse caso, armamos que A = N. Veja que:
1 A
1 + 1
1
= 2 A
x A
x + 1
mdc(1, x)
= x + 1 A
Sendo assim, repetindo esse processo, seguir a por indu c ao que A = N.
Exemplo 12. (Olimpada Matem atica Argentina) Sejam p
1
, p
2
, . . . , p
n
n umeros primos.
Bruno deve escolher n + 1 inteiros positivos que utilizem apenas estes primos em sua de-
composic ao. Bernardo deve escolher alguns desses n umeros de modo que o produto deles
seja um quadrado perfeito. Determine se e possvel, para algum n, que Bruno escolha seus
n + 1 n umeros de maneira que Bernardo n ao consiga cumprir seu objetivo.
Vamos mostrar que Bernardo sempre consegue cumprir seu objetivo. Para decidirmos se
um n umero natural e quadrado perfeito, basta analisarmos a paridade dos expoentes de seus
fatores primos. Para cada n umero p
m
1
1
p
m
2
2
. . . p
m
n
n
escolhido por Bruno, associe a n-upla de
zeros e uns, (r
1
, r
2
, . . . , r
n
), onde cada r
i
e o resto na divisao por 2 de m
i
. A multiplica cao
de dois inteiros se traduz na soma m odulo 2 de tais uplas, i.e., a n-upla associada ao
produto p
m
1
1
p
m
2
2
. . . p
m
n
n
p
l
1
1
p
l
2
2
. . . p
l
n
n
e igual `a (r
1
+q
1
(mod 2), r
2
+q
2
(mod 2), . . . , r
n
+q
n
(mod 2)) onde r
i
e q
i
s ao os restos na divisao por 2 de m
i
e l
i
, respectivamente. Nosso
problema pergunta se e possivel Bernardo encontrar algumas uplas que somadas deem a
upla (0, 0, 0, . . . , 0). Como temos n + 1 uplas, podemos formar 2
n+1
1 > 2
n
somas de
subconjuntos nao vazios de uplas. Cada soma corresponde a uma nova upla, como existem
5
apenas 2
n
tipos de uplas distintas, alguma delas se repetira dentre as somas (pelo princpio
da casa dos pombos). Suponha que para dois conjuntos de uplas A e B tenhamos a mesma
upla associada como soma, entao a soma dos elementos de A e B que nao pertencem `a
A B corresponde `a upla (0, 0, . . . , 0).
Problemas Propostos
Problema 13. Sejam p > 2 um primo mpar e n um inteiro positivo. Prove que p divide
1
p
n
+ 2
p
n
+ . . . + (p 1)
p
n
.
Problema 14. (Olimpada Romena) Sejam a, b, c, d inteiros n ao nulos com a = c e tais que
a
c
=
a
2
+ b
2
c
2
+ b
2
.
Prove que a
2
+ b
2
+ c
2
n ao pode ser um n umero primo.
Problema 15. Encontre todos os n tais que n! e um quadrado perfeito.
Problema 16. (Hungria) O produto de alguns primos e dez vezes maior que a sua soma.
Quais sao esses primos?(n ao necessariamnete distintos).
Problema 17. Qual o maximo divisor comum entre dois n umeros de Fibonacci consecuti-
vos?
Observa c ao: Os n umeros de Fibonacci sao os n umeros da sequencia denida por F
1
= F
2
=
1 e F
n+1
= F
n
+ F
n1
.
Problema 18. Mostre que a soma de dois primos consecutivos nunca e o dobro de um
primo.
Problema 19. (Israel) Se S
n
e a soma dos n primeiros n umeros primos, prove que ha ao
menos um quadrado perfeito entre S
n
e S
n+1
Problema 20. (Olimpada Balcanica) Prove que, para todo natural dado n, existe um na-
tural m > n tal que a representac ao decimal de 5
m
e obtida da representac ao decimal de
5
n
pelo acrescimo de algarismos `a esquerda de 5
n
.
Problema 21. (Inglaterra 1995)
a) Encontre o primeiro inteiro positivo cujo quadrado termina em tres quatros.
b) Encontre todos os inteiros positivos cujo quadrado termina em tres quatros.
c) Mostre que nenhum quadrado perfeito termina em quatro quatros.
6
Exemplo 22. (Olimpada Indiana) Seja n um inteiro positivo tal que n e um divisor da
soma
1 + (1
n1
+ 2
n1
+ . . . + (n 1)
n1
).
Prove que n n ao e divisvel por qualquer quadrado maior que 1.
Problema 23. Seja S um conjunto de primos tal que a, b S (a e b n ao precisam ser
distintos) implicam ab + 4 S. Mostre que S tem que ser vazio
Dicas e Solu coes
1. Em breve!
7
Polos Olmpicos de Treinamento
Curso de Teoria dos Nmeros - Nvel 2
Prof. Samuel Feitosa
Aula
8
Equac oes Diofantinas I
Exemplo 1. Em Gugul andia, o jogo de basquete e jogado com regras diferentes. Existem
apenas dois tipo de pontuac oes para as cestas: 5 e 11 pontos.

E possvel um time fazer 39
pontos em uma partida?
Sejam x e y os n umeros de cestas de 5 e 11 pontos, respectivamente. O problema se resume
em descobrirmos se existem inteiros nao negativos x e y tais que 5x + 11y = 39. Ao inves
de testarmos os valores de x e y, somemos 11 + 5 em ambos os lados da equa cao:
5(x + 1) + 11(y + 1) = 55.
Como 5 | 55 e 5 | 5(x + 1), segue que 5 | 11(y + 1) e, com mais razao, 5 | y + 1 pois
mdc(5, 11) = 1. Do mesmo modo, 11 | x + 1. Assim,
55 = 5(x + 1) + 11(y + 1) 5 11 + 11 5 = 110,
pois x + 1, y + 1 1. Obtemos uma contradi cao.
Exemplo 2. Qual o menor inteiro positivo m para o qual todo n umero maior que m pode
ser obtido como pontuac ao no jogo de basquete mencionado anteriormente?
Como ja sabemos que 39 nao e possvel, e natural comecarmos procurando os n umeros
maiores que 39 que nao podem ser pontuacoes. Veja que:
40 = 5 8 + 11 0
41 = 5 6 + 11 1
42 = 5 4 + 11 2
43 = 5 2 + 11 3
44 = 5 0 + 11 4
Ao somarmos 5 a cada uma dessas representa coes, obteremos representa coes para os proximos
5 n umeros. Repetindo esse argumento, poderemos escrever qualquer n umero maior que 39
na forma 5x + 11y com x e y inteiros nao negativos. Conclumos assim que m = 39. Po-
deramos mostrar que todo n umero maior que 44 e da forma 5x + 11y com x e y inteiros
nao negativos de outro modo. Se n > 44, considere o conjunto:
n 11 0, n 11 1, n 11 2, n 11 3, n 11 4.
Como mdc(11, 5) = 1, o conjunto anterior e um sistema completo de restos m odulo 5 e
consequentemente existe y {0, 1, 2, 3, 4} tal que
n 11 y = 5x
Como n > 44, segue que x > 0.
Exemplo 3. Quais e quantos sao os inteiros positivos n que n ao podem ser obtidos como
pontuac ao nesse jogo de basquete?
Precisaremos relembrar um teorema da aula 03:
Teorema 4. (Bachet-B`ezout) Se d = mdc(a, b), entao existem inteiros x e y tais que
ax + by = d.
A primeira observa cao que fazemos e que uma vez encontrados inteiros x e y, qualquer
m ultiplo de d pode ser representado como uma combinacao linear de a e b:
a(kx) + b(ky) = kd.
Isso e particularmente interessante quando mdc(a, b) = 1, onde obtemos que qualquer in-
teiro e uma combinacao linear de a e b. Veja que isso nao entra em conito com os exemplos
anteriores pois os inteiros x e y mencionados no teorema podem ser negativos.
A proxima propopsicao conter a o que procuramos:
Proposicao 5. Todo inteiro positivo k pode ser escrito(de modo unico) de uma e, somente
uma, das seguintes formas:
11y 5x, ou 11y + 5x, com 0 y < 5 e x 0
Pelo teorema de Bachet-B`ezout, existem m e n tais que 5m + 11n = 1. Sejam q e r o
quociente e resto da divisao de kn por 5, i.e., kn = 5q + r, 0 r < 5. Assim,
k = 5(km) + 11(kn)
= 5(km) + 11(5q + r)
= 5(km + 11q) + 11r.
2
Basta fazer x = km + 11q e r = y.
Para ver a unicidade, suponha que 11m 5n = 11a 5b com 0 m, a < 5. Entao
11(m a) = 5(b n). Usando que mdc(11, 5) = 1, segue que 5 | m a. A unica op cao
e termos m = a pois o conjunto {0, 1, 2, 3, 4} e um scr. Consequentemente 5n = 5b e
n = b.
Sendo assim, os elementos do conjunto
B(5, 11) = {11y 5x Z

+
; 0 y < 5 e x > 0}
constituem o conjunto das pontuacoes que nao podem ser obtidas. Seus elementos s ao:
y = 1 11y 5x = 1, 6
y = 2 11y 5x = 2, 7, 12, 17
y = 3 11y 5x = 3, 8, 13, 18, 23, 28
y = 4 11y 5x = 4, 9, 14, 19, 24, 29, 34, 39
A quantidade de tais inteiros e
20 =
(5 1)
2

(11 1)
2
.
Vale o resultado geral:
Proposicao 6. Dados os inteiros positivos a e b com mdc(a, b) = 1, existem exatamente
(a 1)
2

(b 1)
2
n umeros inteiros n ao negativos que n ao sao da forma ax + by com x, y 0.
Provaremos tal resultado em uma aula futura fazendo o uso da funcao parte inteira.
Exemplo 7. Suponha agora que as pontua c oes das cestas do basquete de Gugul andia tenham
mudado para a e b pontos com 0 < a < b. Sabendo que existem exatamente 35 valores
impossveis de pontuac oes e que um desses valores e 58, encontre a e b.
Perceba que devemos ter mdc(a, b) = 1 pois caso contrario qualquer valor que nao fosse
m ultiplo de mdc(a, b) nao seria uma pontuacao possvel e sabemos que existe apenas um
n umero nito de tais valores. Em virtude da proposicao anterior, (a1)(b1) = 235 = 70.
Analisemos os possveis pares de divisores de 70 tendo em mente que a < b:
(a 1)(b 1) = 1 70 (a, b) = (2, 71)
(a 1)(b 1) = 2 35 (a, b) = (3, 36)
(a 1)(b 1) = 5 14 (a, b) = (6, 15)
(a 1)(b 1) = 7 10 (a, b) = (8, 11)
3
N ao podemos ter (a, b) = (2, 71) pois 58 = 2 29. Excluindo os outros dois casos em que
mdc(a, b) = 1, temos a = 8 e b = 11.
A equa cao ax + by = c e um exemplo de uma equan cao diofantina, i.e., uma equa cao
em que buscamos valores inteiros para as variaveis. Tais equa coes recebem esse nome em
homenagem ao matem atico grego Diofanto.
Exemplo 8. Determine todas as solu c oes inteiras da equa c ao 2x + 3y = 5.
Por paridade, 3y e mpar, donde y = 2k + 1 para algum inteiro k. Da,
x =
5 3(2k + 1)
2
= 1 3k,
e consequentemente todas as solucoes da equa cao s ao da forma (x, y) = (1 3k, 2k + 1).
Exemplo 9. Determine todas as solu c oes inteiras da equa c ao 5x + 3y = 7.
Analisando agora m odulo 3, 5x 7 1 (mod 3). Essa condi cao impoe restri coes sobre o
resto de x na divisao por 3. Dentre os possveis restos na divisao por 3, a saber {0, 1, 2}, o
unico que satisfaz tal congruencia e o resto 2. Sendo assim, x e da forma 3k + 2 e
y =
7 5(3k + 2)
3
= 1 5k,
consequentemente, todas as solucoes da equa cao s ao da forma (x, y) = (3k + 2, 1 5k).
Notemos que para a solucao da congruencia x = 2, obtemos a solucao (x, y) = (2, 1) da
equa cao. Baseado nesses exemplos, e natural imaginarmos que conhecendo uma solucao da
congruencia consigamos descrever todas as outras.
Teorema 10. A equa c ao ax+by = c, onde a, b, c sao inteiros, tem uma solu c ao em inteiros
(x, y) se, e somente se, d = mdc(a, b) divide c. Nesse caso, se (x
0
, y
0
) e uma solu c ao, entao
os pares
(x
k
, y
k
) =

x
0
+
bk
d
, y
0

ak
d

, k Z
sao todas as solu c oes inteiras da equa c ao.
Dada a discuss ao anterior, resta apenas encontrarmos a forma das solucoes. Se (x, y) e
outra solucao, podemos escrever:
ax + by = ax
0
+ by
0
a(x x
0
) = b(y
0
y)
a
d
(x x
0
) =
b
d
(y
0
y)
Como mdc(a/d, b/d) = 1, temos b/d | x x
0
e assim podemos escrever x = x
0
+ bk/d.
Substituindo na equa cao original obtemos y = y
0
ak/d.
4
Exemplo 11. Encontre todas as solu c oes inteiras da equa c ao 21x + 48y = 6
O sitema e equivalente `a 7x + 16y = 2. Uma solucao e (x, y) = (2, 1). Pelo teorema
anterior, todas as solucoes s ao da forma:
(x
k
, y
k
) = (2 + 16k, 1 7k).
Exemplo 12. Resolva nos inteiros a equa c ao 2x + 3y + 5z = 11
Podemos transformar esse problema isolando qualquer uma das variaveis no problema que
ja sabemos resolver. Por exemplo, podemos resolver 2x + 3y = 11 5z. Supondo z xo,
podemos encontrar a solucao particular (x, y) = (4 z, 1 z). Assim, todas as solucoes
s ao da forma:
(x, y) = (4 z + 3k, 1 z 2k),
ou seja, as solucoes da equa cao original s ao da forma (x, y, z) = (4 z + 3k, 1 z 2k, z)
com k e z inteiros.
Vamos estudar agora alguns outros exemplos de equa coes diofantinas nao lineares:
Exemplo 13. Prove que a equa c ao 2
n
+ 1 = q
3
n ao admite solu c oes (n, q) em inteiros
positivos.

E facil ver que a equa cao nao admite solucoes se n = 1, 2, 3. Assim, podemos supor que
n > 3. Fatorando, temos:
(q 1)(q
2
+ q + 1) = 2
n
,
e consequentemente q = 2 ou q = 2k +1, para algum k N. Claramente, q = 2 nao produz
solucao. Entao q = 2k + 1 e q
3
1 = 8k
3
+ 12k
2
+ 6k e uma potencia de 2, maior ou igual
a 16. Entretanto:
8k
3
+ 12k
2
+ 6k = 2k(4k
2
+ 6k + 3),
nao e uma potencia de 2, pois 4k
2
+ 6k + 3 e mpar. Assim, a equa cao 2
n
+ 1 = q
3
nao
admite solucoes inteiras positivas.
Exemplo 14. (URSS 1991) Encontre todas as solu c oes inteiras do sistema

xz 2yt = 3
xt + yz = 1.
Uma boa estrategia sera aplicar alguma manipulacao algebrica, como somar as equa coes,
multiplic a-las, somar um fator de corre cao, entre outras para obtermos alguma fatora cao
envolvendo esses n umeros. Nesse problema, vamos elevar ambas as equa coes ao quadrado.

x
2
z
2
4xyzt + 4y
2
t
2
= 9
x
2
t
2
+ 2xytz + y
2
z
2
= 1.
Multiplicando a segunda por dois e somando com a primeira, temos:
x
2
(z
2
+ 2t
2
) + 2y
2
(z
2
+ 2t
2
) = 11
(x
2
+ 2y
2
)(z
2
+ 2t
2
) = 11.
5
Como cada uma das parcelas acima e um inteiro nao-negativo, temos dois casos:

x
2
+ 2y
2
= 11
z
2
+ 2t
2
= 1
(x, y, z, t) = (3, 1, 1, 0).
ou

x
2
+ 2y
2
= 1
z
2
+ 2t
2
= 11
(x, y, z, t) = (1, 0, 3, 1).
Logo, as unicas solucoes possveis s ao as quadruplas (1, 0, 3, 1) e (3, 1, 1, 0).
Problemas Propostos
Problema 15. Encontre todas as solu c oes de 999x 49y = 5000.
Problema 16. Encontre todos os inteiros x e y tais que 147x + 258y = 369.
Problema 17. Encontre todas as solu c oes inteiras de 2x + 3y + 4z = 5.
Problema 18. Encontre todas as solu c oes inteiras do sistema de equa c oes:
20x + 44y + 50z = 10
17x + 13y + 11z = 19.
Problema 19. (Torneio das Cidades 1997) Sejam a,b inteiros positivos tais que a
2
+ b
2
e
divisvel por ab. Mostre que a = b.
Problema 20. Encontre uma condic ao necessaria e suciente para que
x + b
1
y + c
1
z = d
1
e x + b
2
y + c
2
z = d
2
tenham pelo menos uma solu c ao simultanea em inteiros x, y, z, assumindo que os coeci-
entes sao inteiros com b
1
= b
2
.
Problema 21. (AMC 1989) Seja n um inteiro positivo. Se a equa c ao 2x+2y +n = 28 tem
28 solu c oes em inteiros positivos x, y e z, determine os possveis valores de n.
6
Polos Olmpicos de Treinamento
Curso de Teoria dos Nmeros - Nvel 2
Prof. Samuel Feitosa
Aula
9
O Teorema de Euler
Nesta aula, obteremos uma generaliza cao do teorema de Fermat.
Deni cao 1. Dado n N, denotaremos o n umero de naturais menores ou iguais a n e
relativamente primos com n por (n).
Segue imediatamente da denicao de (n) que (1) = 1, (2) = 1, (3) = 2, (5) = 4 e
(6) = 2. Se p e primo, (p) = p 1.
Lema 2. Se p e um n umero primo e k um n umero natural, entao:
(p
k
) = p
k1
(p 1).
Os unicos n umeros do conjunto {1, 2, . . . , p
k
} que nao s ao relativamente primos com p
k
s ao
aqueles que s ao divisveis por p. A quantidade de tais n umeros e
p
k
p
= p
k1
. Sendo assim,
(p
k
) = p
k
p
k1
= p
k1
(p 1).
Nosso proximo objetivo sera encontrar uma formula para calcular explicitamente (m) em
funcao da fatora cao em primos de m. Precisaremos relembrar um exemplo estudado na
aula 6:
Lema 3. Sejam m um n umero natural, l um n umero natural relativamente primo com m
e r um inteiro arbitrario. Ent ao, o conjunto:
r, l + r, 2l + r, . . . , (m1)l + r;
e um sistema completo de restos modulo m.
Suponha, por absurdo, que existem dois inteiros i e j com 0 i < j < m e para os quais
tenhamos r + il r + jl (mod m). Assim, (j i)l 0 (mod m). Como l e relativamente
primo com m, devemos ter j i 0 (mod m). Obtemos um absurdo pois 0 < j i < m.
Consequentemente, temos um conjunto de m inteiros todos incongruentes m odulo m e,
portanto, tal conjunto e um sistema completo de restos.
POT 2012 - Teoria dos N umeros - Nvel 2 - Aula 9 - Samuel Feitosa
Teorema 4. Se l e m sao n umeros naturais primos entre si, entao:
(ml) = (m)(l).
Demonstrac ao. Como (1) = 1, o teorema anterior e valido quando m = 1 ou n = 1.
Suponha entao que m, l > 1. Fa camos uma contagem dupla. Primeiramente, usando a
denicao, (mn) e o n umero de inteiros da tabela abaixo que s ao relativamente primos
com ml.
1, 2, . . . , r, . . . , l,
1 + l, l + 2, . . . , l + r, . . . , 2l,
21 + l, 2l + 2, . . . , 2l + r, . . . , 3l,
. . . , . . . , . . . , . . . , . . . , . . . ,
(m1)1 + l, (m1)l + 2, . . . , (m1)l + r, . . . , ml,
Seja r m um n umero natural qualquer. Considerando a r-esima coluna da tabela, se
mdc(r, l) > 1, nenhum de seus elementos e relativamente primo com l. Entao, se buscamos
os elementos que nao possuem nenhum fator em comum com ml, devemos nos ater `as
colunas com mdc(r, l) = 1. O n umero de tais colunas e (l). Considerando agora a r-esima
coluna e supondo que mdc(r, l) = 1, em virtude do lema anterior, sabemos que os restos de
seus elementos na divisao por m formam exatamente o conjunto {0, 1, . . . , m} e dentre eles
existem exatamente (m) n umeros relativamente primos com m. Sendo assim, podemos
contar os n umeros relativamente primos com ml atr aves do n umero de colunas boase do
n umero de bonselementos em cada uma delas, obtendo: (m)(l).
Corolario 5. Se n = p

1
1
p

2
2
. . . p

k
k
e a fatorac ao em primos de n, entao:
(n) = n

1
1
p
1

1
1
p
2

. . .

1
1
p
k

De fato, pelo teorema anterior,


(n) = (p

1
1
p

2
2
. . . p

k
k
)
= (p

1
1
)(p

2
2
) . . . (p

k
k
)
= p

1
1
1
(p
1
1)p

2
1
2
(p
2
1) . . . p

k
1
k
(p
k
1)
= p

1
1
1
p

2
1
2
. . . p

k
1
k
(p
1
1)(p
2
1) . . . (p
k
1)
= n

1
1
p
1

1
1
p
2

. . .

1
1
p
k

Exemplo 6. Mostre que qualquer n 7 pode ser escrito na forma a+b, com a e b naturais
primos entre si, ambos maiores que 1.
Podemos escrever b = n a e nosso objetivo e encontrar a com 1 < a < n 1 tal que
mdc(a, n a) = 1. Para isso, basta que mdc(a, n) = 1. Pelo corolario anterior,
(n) = p

1
1
1
p

2
1
2
. . . p

k
1
k
(p
1
1)(p
2
1) . . . (p
k
1)
2
POT 2012 - Teoria dos N umeros - Nvel 2 - Aula 9 - Samuel Feitosa
Se a expressao anterior e igual `a 2, necessariamente devemos ter
i
= 1 e p
i
= 2 ou 3 para
todo i. Sendo assim, n 6. Logo, (n) > 2 e existe pelo menos outro n umero natural
diferente de 1 e n 1 que e relativamente primo com n.
Exemplo 7. Prove que existem innitos inteiros positivos n tais que
(n) =
n
3
.
Basta tomar n = 2 3
m
, onde m e um inteiro positivo. Entao:
(n) = (2 3
m
) = (2)(3
m
) = 2 3
m1
=
n
3
.
Exemplo 8. Se n e um inteiro positivo composto, entao
(n) n

n
Se n = p

1
1
p

2
2
. . . p

k
k
, usando que n e composto, podemos garantir que existe um fator
primo p
i
tal que p
i


n. Assim,
(n) = n

1
1
p
1

1
1
p
2

. . .

1
1
p
k

1
1
p
i

1
1

= n

n
Teorema 9. (Teorema de Euler) Se mdc(a, m) = 1, entao
a
(m)
1 (mod m)
Demonstrac ao. A prova deste teorema sera muito similar `a prova do teorema de Fermat.
Sejam r
1
, r
2
, . . . , r
(m)
os restos em {0, 1, 2, . . . , m 1} que s ao relativamente primos com
m. Considere o conjunto {ar
1
, ar
2
, . . . , ar
(m)
}. Se dois de seus membros deixam o mesmo
resto por m, digamos:
ar
i
ar
j
(mod m);
temos r
i
r
j
(mod m) pois mdc(a, m) = 1. Claramente isso e uma contradi cao. Alem
disso, mdc(ar
i
, m) = mdc(m, r
i
) = 1. Analisando os restos na divisao por m dos membros
desse novo conjunto, podemos concluir que tal conjunto coincide com o conjunto dos restos
iniciais. Assim,
r
1
r
2
. . . r
(m)
ar
1
ar
2
. . . ar
(m)
a
(m)
r
1
r
2
. . . r
(m)
Como mdc(r
1
r
2
. . . r
(m)
, m) = 1, podemos cancelar esse termo em ambos os membros
da congruencia anterior obtendo assim o teorema de Euler.
3
POT 2012 - Teoria dos N umeros - Nvel 2 - Aula 9 - Samuel Feitosa
Exemplo 10. Encontre os ultimos tres dgitos de 7
9999
Como (1000) = 400, usando o Teorema de Euler, obtemos:
7
10000
= (7
400
)
25
1 (mod 1000)
Note que 7 143 = 1001 1 (mod 1000). Assim,
7
9999
7
9999
7 143
7
10000
143
143 (mod 1000)
Logo, 7
9999
termina em 143.
Exemplo 11. (Putnam 1972) Prove que n ao existe um inteiro n > 1 tal que n|2
n
1.
Se existem tais inteiros positivos, denotemos por m o menor deles. Claramente m e mpar,
pelo teorema de Euler, podemos garantir que:
m | 2
(m)
1.
Seja d = mdc(m, (m)). Pelo problema 27 da aula 3, temos 2
d
1 = mdc(2
m
1, 2
(m)
1).
Como m | mdc(2
m
1, 2
(m)
1), d > 1. Alem disso, d (m) < m e d | 2
d
1. Isso e
um absurdo pois m e o menor inteiro maior que 1 com tal propriedade.
Exemplo 12. (Olimpada de Matem atica Argentina) Demostre que para cada n umero na-
tural n, existe uma potencia de 2 cuja expansao decimal tem entre seus ultimos n dgitos
(da direita) mais de
2n
3
dgitos que sao iguais a 0.
Se 2
k
tiver um resto muito pequeno m odulo 10
n
, poderemos garantir que existir ao muitos ze-
ros consecutivos entre seus ultimos dgitos. Para obtermos a equa cao 2
k
r (mod 10
n
) com
r pequeno, e interessante comecarmos analisando 2
k
(mod 5
n
) uma vez que mdc(2, 5
n
) = 1.
Fa camos isso. Pelo teorema de Euler, temos:
2
(n)
1 (mod 5
n
)
2
(n)+n
2
n
(mod 10
n
).
Como 2
n
= 8
n/3
< 10
n/3
, podemos concluir que 2
n
possui menos que
n
3
dgitos e, conse-
quentemente, entre os ultimos n dgitos de 2
(n)+n
existem pelo menos n
n
3
=
2n
3
dgitos
consecutivos iguais `a zero.
Exemplo 13. (IMO 1971) Prove que a sequencia 2
n
3(n > 1) contem uma subsequencia
de n umeros primos entre si dois a dois.
4
POT 2012 - Teoria dos N umeros - Nvel 2 - Aula 9 - Samuel Feitosa
Uma boa estrategia e construir uma sequencia recursivamente. Suponha que ja tenhamos
escolhido os termos a
1
, a
2
, . . . , a
k
na sequencia de modo que mdc(a
i
, a
j
) = 1. Como pode-
remos escolher o proximo termo a
k+1
da forma 2
n
3? Claramente mdc(2, a
i
) = 1. Desde
que (a
i
) | n, poderemos usar o teorema de Euler para obter:
2
n
3 1 3
0 (mod a
i
)
Sendo assim, pelo teorema 4, basta escolhermos:
n = (a
1
a
2
. . . a
k
) = (a
1
)(a
2
) . . . (a
k
);
que naturalmente sera um m ultiplo de cada (a
i
). Logo, podemos denir
a
k+1
= 2
(a
1
a
2
...a
k
)
3
e assim temos uma sequencia de termos inta satisfazendo as condi coes do enunciado.
Problemas Propostos
Problema 14. Encontre todos os n umeros naturais n para os quais (n) n ao e divisvel por
4.
Problema 15. Prove que se p > 2 e 2p + 1 sao ambos n umeros primos, entao para n = 4p
vale que
(n + 2) = (n) + 2.
Problema 16. Encontre todas as solu c oes nos n umeros naturais da equa c ao (n) = (2n).
Problema 17. Encontre todas as solu c oes nos n umeros naturais da equa c ao (2n) = (3n).
Problema 18. Se n possui k fatores primos distintos, prove que 2
k
| (n).
Problema 19. Prove que para qualquer n umero natural k, existe pelo menos um n umero
natural n tal que
(n + k) = (n).
Dica: Considere o menor divisor primo p que n ao e um divisor de k e estude o n umero
n = (p 1)k.
Problema 20. Mostre que se a e b sao inteiros primos entre si, entao existem inteiros m e
n tais que a
m
+ b
n
1 (mod ab).
Problema 21. (Alemanha) Se n e um n umero natural tal que 4
n
+ 2
n
+ 1 e primo, prove
que n e potencia de 3.
5
Problema 22. (USAMO 1991) Mostre que para qualquer inteiro xo n 1, a sequencia
2, 2
2
, 2
2
2
, 2
2
2
2
, . . . (mod n);
e eventualmente constante, isto e, a partir de um certo termo da sequencia todos os restos
obtidos na divisao por n serao iguais.
Dica: Tente considerar os casos em que n e par ou n e mpar em separado e use induc ao.
Problema 23. Encontre os ultimos 8 dgitos da expansao bin aria de 27
1986
Problema 24. Mostre que, para qualquer inteiro positivo n com n = 2 e n = 6 temos:
(n)

n.
Referencias
[1] F. E. Brochero Martinez, C. G. Moreira, N. C. Saldanha, E. Tengan - Teoria dos
N umeros ? um passeio com primos e outros n umeros familiares pelo mundo inteiro,
Projeto Euclides, IMPA, 2010.
[2] E. Carneiro, O. Campos and F. Paiva, Olimpadas Cearenses de Matem atica 1981-2005
(Nveis J unior e Senior), Ed. Realce, 2005.
[3] S. B. Feitosa, B. Holanda, Y. Lima and C. T. Magalhaes, Treinamento Cone Sul 2008.
Fortaleza, Ed. Realce, 2010.
[4] D. Fomin, A. Kirichenko, Leningrad Mathematical Olympiads 1987-1991, MathPro
Press, Westford, MA, 1994.
[5] D. Fomin, S. Genkin and I. Itenberg, Mathematical Circles, Mathematical Words, Vol.
7, American Mathematical Society, Boston, MA, 1966.
[6] I. Niven, H. S. Zuckerman, and H. L. Montgomery, An Introduction to the Theory of
Numbers.
Polos Olmpicos de Treinamento
Curso de Teoria dos Nmeros - Nvel 2
Prof. Samuel Feitosa
Aula
10
Divisores
Suponha que n = p

1
1
p

2
2
. . . p

k
k
e a fatora cao em primos do inteiro n. Todos os divisores
de n s ao da forma m = p

1
1
p

2
2
. . . p

k
k
, onde 0
i

i
. Cada um desses n umeros, aparece
exatamente uma vez no produto:
(1 + p
1
+ p
2
1
+ . . . + p

1
1
)(1 + p
2
+ p
2
2
+ . . . + p

2
2
) . . . (1 + p
n
+ p
2
n
+ . . . + p

k
k
),
quando o mesmo e expandido usando a distributividade. Como existem
i
+ 1 termos em
cada parenteses, O n umero de termos dessa expans ao e:
(
1
+ 1)(
2
+ 1) . . . (
k
+ 1).
Alem disso, sabemos que:
1 +p
i
+p
2
i
+. . . +p

i
i
=
p

i
+1
i
1
p
i
1
.
Sendo assim, podemos concluir que:
Teorema 1. Se n = p

1
1
p

2
2
. . . p

k
k
e a fatorac ao em primos de n, entao:
a) O n umero de divisores de n, denotado por d(n), e: (
1
+ 1)(
2
+ 1) . . . (
n
+ 1).
b) A soma dos divisores de n, denotada por (n), e:
(1 +p
1
+p
2
1
+. . . +p

1
1
)(1 +p
2
+p
2
2
+. . . +p

2
2
) . . . (1 +p
n
+p
2
n
+. . . +p

n
n
)
ou, de forma mais sucinta,

1
+1
1
1
p
1
1

2
+1
2
1
p
2
1

. . .

n
+1
n
1
p
n
1

POT 2012 - Teoria dos N umeros - Nvel 2 - Aula 10 - Samuel Feitosa


Observacao 2. (Pareamento de divisores) Se d e um divisor de n, entao
n
d
tambem e um
divisor de n.
Portanto, pelo menos um dentre {d,
n
d
} e um divisor de n menor ou igual a

n.
Exemplo 3. Determine o n umero de divisores positivos de 2008
8
que sao menores que
2008
4
.
O n umero de divisores de 2008
8
= 2
24
251
8
e 225. Como n e um quadrado perfeito e em
virtude da observa cao anterior, 112 desses divisores s ao menores que

2008
8
= 2008
4
e 112
s ao maiores.
Exemplo 4. Encontre a soma dos inversos dos divisores postivos de 496.
Sejam d
1
, d
2
, . . . , d
n
os divisores de 496 e K a soma de seus inversos. Usando a observa cao
anterior, o conjunto {
496
d
1
+
496
d
2
+. . . +
496
d
n
} coincide com o conjunto {d
n
+d
n1
+. . . +d
1
}
e da:
1
d
1
+
1
d
2
+. . . +
1
d
n
= K
496
d
1
+
496
d
2
+. . . +
496
d
n
= 496K
d
n
+d
n1
+. . . +d
1
= 496K
2
5
1
2 1

31
2
1
31 1
= 496K
960
496
= K.
Portanto, k =
60
31
.
Exemplo 5. Um n umero natural n possui exatamente dois divisores e n + 1 possui exata-
mente 3 divisores. Encontre o n umero de divisores de n + 2.
Se n possui exatamente dois divisores, entao n = p e um n umero primo. Se n + 1 possui
um n umero mpar de divisores, entao n + 1 = x
2
e um quadrado perfeito, para algum x
inteiro positivo. Logo, x
2
1 = (x 1)(x + 1) = p. Como p e primo, a unica possibilidade
e x 1 = 1 e consequentemente n = 3. O n umero de divisores de n + 2 = 5 e 2.
Exemplo 6. Encontre todos os inteiros n que possuem exatamente

n divisores positivos.
Para

n ser inteiro, n deve ser um quadrado perfeito e assim podemos escrever:
n = p
2
1
1
p
2
2
2
. . . p
2
k
k
.
A condi cao do problema e equivalente ` a:
p

1
1
p

2
2
. . . p

k
k
= (2
1
+ 1)(2
2
+ 1) . . . (2
k
+ 1).
2
POT 2012 - Teoria dos N umeros - Nvel 2 - Aula 10 - Samuel Feitosa
Analisando o lado direito, podemos concluir que cada p
i
e mpar e consequentemente
p

i
i
3

i
2
i
+ 1.
Como devemos ter a igualdade, p
1
= 3 e 3

1
= 2
1
+ 1. Se
1
> 1, vale a desigualdade
estrita(veja o problema 13). Logo, a unica solucao e n = 9.
Exemplo 7. (Sui ca 2011) Encontre todos os inteiros positivos n para o qual n
3
e o produto
de todos os divores de n
Claramente n = 1 e solucao. Suponha que n > 1 e sejam d
1
< d
2
< . . . < d
k
os divisores
de n. Pela observa cao 2, podemos agrupar os divisores em pares cujo produto e n, assim:
n
6
= (d
1
d
2
. . . d
k
)(d
1
d
2
. . . d
k
)
= (d
1
d
k
)(d
2
d
k1
) . . . (d
k
d
1
)
= n
d(n)
Consequentemente, 6 = d(n) e n = p
5
ou n = pq
2
com p e q primos distintos. Fica a cargo
do leitor vericar que essas solucoes satisfazem o enunciado.
Exemplo 8. (Irlanda 1995) Para cada inteiro positivo n tal que n = p
1
p
2
p
3
p
4
, onde p
1
, p
2
,
p
3
e p
4
sao primos distintos, sejam:
d
1
= 1 < d
2
< d
3
< . . . < d
16
= n,
os 16 inteiros positivos que dividem n. Prove que se n < 1995, entao d
9
d
8
= 22.
Suponha que n < 1995 e d
9
d
8
= 22. Note inicialmente que d
8
nao pode ser par pois n
seria divisvel por 4 contradizendo o fato de que n possui quatro fatores primos distintos.
Consequentemente d
8
, d
9
e n s ao mpares. Tambem temos a fatora cao: 35 57 = 1995 =
3 5 7 19. Entao, usando a observa cao 2, d
8
d
9
= n. Se d
8
35 teramos d
9
< d
8
para
manter n < 1995 e isso seria um absurdo. Logo, d
8
< 35. Os divisores d
1
, d
2
, . . . , d
8
s ao
produtos de primos mpares distintos. Como 3 5 7 > 35, nenhum dentre d
1
, d
2
, . . . , d
8
e grande o suciente para possuir tres fatores primos distintos. Como n possui somente
quatro fatores primos distintos, quatro desses d
i
s devem ser o produto de dois primos
mpares. Os menores n umeros que s ao o produto de dois primos s ao:
15, 21, 33, 35, . . .
e consequentemente devemos ter d
8
35, uma contradi cao.
Exemplo 9. Prove que n ao existe inteiro positivo n tal que (n) = n
k
para algum inteiro
positivo k.
Armamos que n = 1 e a unica solucao. Suponha que n > 1 seja solucao e sejam
d
1
= 1 < d
2
< . . . < d
k
= n,
3
POT 2012 - Teoria dos N umeros - Nvel 2 - Aula 10 - Samuel Feitosa
os divisores de n. Entao
(n) = d
1
+d
2
+. . . +d
k
< 1 + 2 +. . . +n =
n(n + 1)
2
< n
2
.
Alem disso,
n < n + 1 d
1
+d
2
+. . . +d
k
= (n).
Da,
n < n
k
< n
2
,
e obtemos um absurdo.
Exemplo 10. (Olimpada de Leningrado 1989) Duas pessoas jogam um jogo. O n umero 2
est a inicialmente escrito no quadro. Cada jogador, na sua vez, muda o n umero atual N
no quadro negro pelo n umero N +d, onde d e um divisor de N com d < N. O jogador que
escrever um n umero maior que 19891988 perde o jogo. Qual deles ir a vencer se ambos os
jogadores sao perfeitos.
Nesse problema, basta determinarmos apenas aquele que possui a estrategia vencedora.
Note que o incio do jogo e estritamente determinado: 2 3 4. Suponha que o segundo
jogador vence o jogo. Ap os o movimento 4 5 do primeiro jogador, o segundo s o pode
jogar 5 6. Isto signica que 6 e uma posicao vencedora. Entretanto, o primeiro jogador
pode obter a posicao 6 jogando 4 6, uma contradi cao. Logo, o primeiro jogador possui
a estrategia vencendora.
Exemplo 11. (Olimpada de Leningrado) Duas pilhas de palitos sobre uma mesa contem
100 e 252 palitos, respectivamente. Dois jogadores jogam o seguinte jogo: Cada jogador em
sua vez pode remover alguns palitos de uma das pilhas de modo que o n umero de palitos
retirados seja um divisor do n umero de palitos da outra pilha. O jogador que zer o ultimo
movimento vence. Qual dos dois jogadores ir a vencer se ambos sao perfeitos?
O primeiro jogador perde. Em cada momento do jogo, podemos registrar o expoente
da maior potencia de 2 que divide os n umeros de palitos em cada pilha. Por exemplo,
no incio os n umeros s ao (2, 2). A estrategia do segundo jogador e manter esse n umeros
sempre iguais. Suponha que, em um dado momento, as pilhas possuem 2
m
a e 2
m
b
palitos com a e b mpares. O par registrado sera (m, m). Vejamos o que acontece quando
retiramos um divisor d da segunda pilha do n umero de palitos da primeira. Se 2
m
e a
maior potencia de 2 que divide d, entao 2
m+1
dividira o n umero de palitos da primeira
pilha e consequentemente o par registrado ter a n umeros diferentes. Se 2
k
, com k < m,
e a maior potencia de 2 que divide d, entao 2
k
sera a maior potencia de 2 que divide o
n umero de palitos da primeira pilha e novamente o par registrado ter a n umeros diferentes.
Assim, sempre que um jogador receber um par registrado com n umeros iguais, ele ir a passar
um par registrado com n umeros diferentes para o outro jogador. Suponha agora que, na
sua vez, as pilhas possuem 2
m
a e 2
n
b palitos, com m < n e a b 1 (mod 2).
Basta o jogador retirar 2
m
palitos da segunda pilha para passar um par registrado com
n umeros iguais a (m, m). Como inicialmente as pilhas possuem n umeros registrados iguais,
o segundo jogador pode sempre manter essa propriedade e consequentemente o unico que
pode passar uma pilha com zero palitos pela primeira vez e o primeiro jogador.
4
POT 2012 - Teoria dos N umeros - Nvel 2 - Aula 10 - Samuel Feitosa REFER

ENCIAS
Problemas Propostos
Problema 12. Mostre que se k e um inteiro positivo entao 3
k
2k+1 e vale a desigualdade
estrita quando k > 1.
Problema 13. (R ussia 2001) Encontre todos os n tais que quaisquer divisores primos dis-
tintos a e b de n o n umero a +b 1 tambem e um divisor de n
Problema 14. O n umero 3
32
1 tem exatamente dois divisores que sao maiores que 75 e
menores que 85. Qual o produto desses dois divisores?
Problema 15. (Ira 2012) Sejam a e b inteiros positivos de modo que o n umero de divisores
positivos de a,b, ab e 3,4 e 8, respectivamente. Encontre o n umero de divisores positivos
de b
2
.
Problema 16. (Olimpada de S ao Petesburgo) Enconte todos os inteiros positivos n tais
que 3
n1
+ 5
n1
divide 3
n
+ 5
n
.
Problema 17. Sejam 1 = d
1
< d
2
< .... < d
k
= n o conjunto de todos os divisores de um
inteiro positivo n. Determine todos os n tais que:
d
2
6
+d
2
7
1 = n.
Problema 18. Um divisor d > 0 de um inteiro positivo n e dito ser um divisor unit ario
se mdc(d,
n
d
) = 1. Suponha que n e um inteiro positivo tal que a soma de seus divisores
unit arios e 2n. Prove que n n ao pode ser mpar.
Referencias
[1] F. E. Brochero Martinez, C. G. Moreira, N. C. Saldanha, E. Tengan - Teoria dos
N umeros ? um passeio com primos e outros n umeros familiares pelo mundo inteiro,
Projeto Euclides, IMPA, 2010.
[2] E. Carneiro, O. Campos and F. Paiva, Olimpadas Cearenses de Matem atica 1981-2005
(Nveis J unior e Senior), Ed. Realce, 2005.
[3] S. B. Feitosa, B. Holanda, Y. Lima and C. T. Magalhaes, Treinamento Cone Sul 2008.
Fortaleza, Ed. Realce, 2010.
[4] D. Fomin, A. Kirichenko, Leningrad Mathematical Olympiads 1987-1991, MathPro
Press, Westford, MA, 1994.
[5] D. Fomin, S. Genkin and I. Itenberg, Mathematical Circles, Mathematical Words, Vol.
7, American Mathematical Society, Boston, MA, 1966.
[6] I. Niven, H. S. Zuckerman, and H. L. Montgomery, An Introduction to the Theory of
Numbers.
5
Polos Olmpicos de Treinamento
Curso de Teoria dos Nmeros - Nvel 2
Prof. Samuel Feitosa
Aula
11
O Teorema Chines dos Restos
Iremos estudar um antigo teorema descoberto pelos chineses no incio seculo XIII. Come-
cemos recordando um lema da aula 06:
Lema 1. Se mdc(a, m) = 1, entao existe um inteiro x tal que:
ax 1 (mod m).
Tal inteiro e unico modulo m. Se mdc(a, m) > 1, n ao existe x satisfazendo tal equa c ao.
Demonstra c ao. Pelo teorema de Bachet-Bezout, existem inteiros x e y tais que ax+my = 1.
Analisando essa congruencia m odulo m, obtemos ax 1 (mod m). Se y e outro inteiro
que satisfaz a mesma congruencia, temos ax ay (mod m). Pelo primeiro lema, x y
(mod m). Se d = mdc(a, m) > 1, n ao podemos ter d | m e m | ax 1 pois d ax 1.
Exemplo 2. Encontre x inteiro tal que:
x 1 (mod 11);
x 2 (mod 7).
A primeira congruencia nos diz que x = 11k +1 para algum k Z. Sejam q e r o quociente
e o resto da divisao de k por 7, respectivamente. Assim, k = 7q + r e x = 77q + 11r + 1.
Para x satisfazer a segunda congruencia, devemos encontrar r {0, 1, 2, 3, 4, 5, 6} tal que
11r + 1 2 (mod 7), ou seja, 4r 1 (mod 7). Como o inverso de 4 (mod 7) e 2, obtemos
r = 2 e x = 77q + 23. Veja que para qualquer q inteiro, tal x e solu cao do sistema de
congruencias.
Exemplo 3. Encontre x inteiro tal que:
x 1 (mod 11)
x 2 (mod 7)
x 4 (mod 5)
POT 2012 - Teoria dos N umeros - Nvel 2 - Aula 11 - Samuel Feitosa
Pelo exemplo anterior, para x satisfazer as duas primeiras equacoes, devemos ter
x = 77q+23. Dividindo q por 5, obtemos q = 5l+s com 0 s < 5. Da, x = 385l+77s+23.
Para satisfazer a ultima congruencia, devemos ter 77s + 23 4 (mod 5), ou seja, 2s 1
(mod 5). Como 3 e o inverso de 2 (mod 5), s = 3 e consequentemente x = 385l + 254.
Perceba que nos dois exemplos anteriores, o problema foi reduzido `a encontrarmos o inverso
de um inteiro. No ultimo exemplo, a solu cao geral possui a forma: x = 1175l+231+22+1.
Essencialmente, o trabalho de encontrar esses inversos foi possvel pois os inteiros 5, 7 e 11
s ao primos entre si dois a dois.
Veremos agora um mecanismo levemente diferente para resolver tais sistemas equacoes.
Teorema 4. (Teorema Chines dos Restos) Sejam m
1
, m
2
, . . . , m
r
, inteiros positivos
primos entre si, dois a dois, e sejam a
1
, a
2
, . . . , a
r
; r inteiros quaisquer. Entao, o sistema
de conguencias:
x a
1
(mod m
1
)
x a
2
(mod m
2
)
.
.
.
x a
r
(mod m
r
)
admite uma soluc ao x. Alem disso, as soluc oes s ao unicas modulo m = m
1
m
2
. . . m
r
.
Demonstra c ao. Escrevendo m = m
1
m
2
. . . m
r
, vemos que
m
m
j
e um inteiro e
mdc

m
m
j
, m
j

= 1. Entao, pelo lema inicial, para cada j, existe um inteiro b


j
tal que

m
m
j

b
j
1 (mod m
j
). Claramente

m
m
j

b
j
0 (mod m
i
) para i = j. Denamos
x
0
=
m
m
1
b
1
a
1
+
m
m
2
b
2
a
2
+ . . . +
m
m
r
b
r
a
r
Consideremos x
0
m odulo m
i
: x
0

m
m
i
b
j
a
j
a
i
(mod m
i
). Logo, x
0
e uma solu cao do
nosso sistema. Se x
0
e x
1
tambem o s ao, podemos escrever x
0
x
1
(mod m
i
) para cada i.
Como mdc(m
i
, m
j
) = 1, para i = j, pbtemos x
0
x
1
(mod m).
Observacao 5. Se cada uma das equa c oes do sistema anterior fosse do tipo
b
i
x a
i
(mod m)
i
, com mdc(b
i
, m) = 1, ainda poderamos us a-lo. Bastaria reescrever
b
i
x a
i
(mod m)
i
como x b
i
a
i
(mod m)
i
, onde b
i
e o inverso de b
i
(mod m)
i
.
Exemplo 6. Encontre o menor inteiro positivo x tal que x 5 (mod 7), x 7 (mod 11) e
x 3 (mod 13).
Usando o teorema anterior com m
1
= 5, m
2
= 7, m
3
= 11, a
1
= 5, a
2
= 7 e a
3
= 3 podemos
achar x 887 (mod 1001) = 7.11.13. Como a solu cao e unica m odulo m, isso signica
que, dentre os n umeros 1, 2, , 1001 a menor solu cao positiva e 887.
2
POT 2012 - Teoria dos N umeros - Nvel 2 - Aula 11 - Samuel Feitosa
Exemplo 7. (OBM 2009) Sejam m e n dois inteiros positivos primos entre si. O Teorema
Chines dos Restos arma que, dados inteiros i e j com 0 i < m e o j < n, existe
exatamente um inteiro a, com 0 a < mn, tal que o resto da divis ao de a por m e igual
a i e o resto da divis ao de a por n e igual a j. Por exemplo, para m = 3 e n = 7, temos
que 19 e o unico n umero que deixa restos 1 e 5 quando dividido por 3 e 7, respectivamente.
Assim, na tabela a seguir, cada n umero de 0 a 20 aparecera exatamente uma vez.
0 1 2 3 4 5 6
0 A B
1 C D
2 E F
Qual a soma dos n umeros das casas com as letras A, B, C, D, E e F?
Usando o teorema chines dos restos, podemos encontrar A = 15, B = 12, C = 10, D =
13, E = 8 e F = 11. Assim, A + B + C + D + E + F = 69.
Exemplo 8. (Est onia 2000) Determine todos os restos possveis da divis ao do quadrado de
um n umero primo com 120 por 120.
Seja n tal que mdc(n, 120) = 1. Como 120 = 3 5 8, temos que n 0 (mod 3), (mod 5)
(mod 2). Da, n
2
1 (mod 3), n
2
1 (mod 8) e n
2
1 ou 4 (mod 5). Sendo assim, n
2
satisfaz o sistema:
x 1 (mod 3)
x 1 (mod 8)
x 1 (mod 5)
cujas solu coes s ao x 1 (mod 120) e x 49 (mod 120).
Aconselhamos ao leitor a resolucao de alguns exemplos numericos ate adquirir pr atica com o
algoritmo usado para encontrar x
0
. Provamos, no teorema passado, que todas as solu coes
daquele sistema de congruencias s ao os termos de uma P.A de raz ao m. Geralmente
usaremos aquele teorema apenas para garantir que um sistema de congruencias admite
uma solu cao. Os pr oximos exemplos podem deixar isso mais claro.
Exemplo 9. Para cada n umero natural n, existe uma sequencia arbitrariamente longa de
n umeros natu rais consecutivos, cada um deles sendo divisvel por uma s-esima potencia
de um n umero natural maior que 1.
Demonstra c ao. Dado m N, considere o conjunto {p
1
, p
2
, . . . , p
m
} de primos distintos.
Como mdc(p
s
i
, p
s
j
) = 1, entao pelo teorema 3, existe x tal que x i (mod p
s
i
) para
i = 1, 2, . . . m. Cada um dos n umeros do conjunto {x + 1, x + 2, . . . , x + m} e divisvel por
um n umero da forma p
s
i
.
3
POT 2012 - Teoria dos N umeros - Nvel 2 - Aula 11 - Samuel Feitosa
Exemplo 10. (USAMO 1986)
(a) Existem 14 inteiros positivos consecutivos tais que, cada um e divisvel por um ou
mais primos p do intervalo 2 p 11?
(b) Existem 21 inteiros positivos consecutivos tais que, cada um e divisvel por um ou
mais primos p do intervalo 2 p 13?
Demonstra c ao. (a) Nao. Suponha que existam tais inteiros. Da nossa lista de 14 inteiros
consecutivos, 7 s ao n umeros pares. Vamos observar os mpares: a, a + 2, a + 4, a + 6, a +
8, a + 10 e a + 12. Podemos ter no m aximo tres deles divisveis por 3, dois por 5, um por
7 e um por 11. Veja que 3 + 2 + 1 + 1 = 7. Pelo Princpio da Casa dos Pombos, cada um
desses mpares e divisvel por exatamente um primo do conjunto {3, 5, 7, 11}. Alem disso,
note que os m ultiplos de 3 s o podem ser {a, a + 6, a +12}. Dois dos n umeros restantes em
(a + 2, a + 4, a + 8, e a + 10) s ao divisveis por 5. Mas isso e impossvel. (b) Sim. Como
os n umeros {210, 11, 13} s ao primos entre si, dois a dois, pelo teorema 3 existe um inteiro
positivo n > 10 tal que:
n 0( mod 210 = 2 3 5 7)
n 1( mod 11)
n 1( mod 13)
Veja que o conjunto {n 10, n 9, . . . , n + 9, n + 10} satisfaz as condi coes do item (b).
Exemplo 11. Sejam a e b inteiros positivos tais que, para qualquer n natural, a
n
+n | b
n
+n.
Prove que a = b.
Seja p um primo maior que a e b. Entao mdc(p, a) = mdc(p, b) = 1. Como mdc(p, p1) = 1,
existe um inteiro positivo n tal que n 1 (mod p1) e n a (mod p). Pelo teorema de
Fermat, a
n
+n 0 (mod ) e b
n
+n b a (mod p). Assim, p | |b a|. Como |b a| < p,
segue que |b a| = 0 e a = b.
Exemplo 12. (Olimpada Nordica 1998)
(a) Para quais inteiros positivos n existe um sequencia x
1
, x
2
, . . . , x
n
contendo cada um
dos inteiros 1, 2, . . . , n exatamente uma vez, e tal que k divide x
1
+x
2
+ +x
k
para
k = 1, 2, , n?
(b) Existe uma sequencia innita x
1
, x
2
, . . . contendo todo inteiro positivo exatamente
uma vez, e tal que para cada inteiro positivo k, k divide x
1
+ x
2
+ + x
k
?
a) Suponha que n e um inteiro que satisfaz o enunciado. Naturalmente n divide a soma:
x
1
+ x
2
+ . . . x
n
=
n(n + 1)
2
.
Da,
n + 1
2
e um inteiro e n deve ser mpar. Seja m =
n + 1
2
. Usando que
(n 1) | x
1
+ x
2
+ . . . x
n1
= mn x
n
,
4
POT 2012 - Teoria dos N umeros - Nvel 2 - Aula 11 - Samuel Feitosa
temos x
n
m (mod n 1) se n 3 e, consequentemente, x
n
= m. Repetindo a mesma
an alise para n 2 no lugar de n 1, obtemos x
n1
= m para n 5. Como n ao podem
existir dois termos iguais, temos um absurdo. Analisando os casos quando n 4, encon-
tramos n = 1 e n = 3 como unicas solu coes.
b) Iremos construir a sequencia indutivamente. Suponha que ja tenhamos denido os termos
x
1
, x
2
, . . . , x
n
satisfazendo a condi cao k | x
1
+x
2
. . . x
k
para todo k n. Seja m o menor in-
teiro positivo que ainda n ao apareceu na sequencia. Pelo Teorema Chines dos Restos, existe
x tal que x (x
1
+x
2
+. . . +x
n
) (mod n+1) e x (x
1
+x
2
+. . . +x
n
)m (mod n+2).
Escolha l, inteiro positivo, tal que l > x
1
, x
2
, . . . , x
n
, m e l x (mod (n+1)(n+2)). Dena
x
n+1
= l e x
n+2
= m. Veja que a condi cao k | x
1
+ x
2
. . . x
k
agora e verdadeira para todo
k n + 2. Para o incio, basta denir x
1
= 1.
Exemplo 13. (Olimpada de S ao Petesburgo 1990) Dado um polinomio F(x) com coe-
cientes inteiros, tal que, para cada inteiro n, o valor de F(n) e divisvel por pelo menos
um dos inteiros a
1
, a
2
, , a
m
. Prove que podemos encontrar um ndice k tal que F(n) e
divisvel por a
k
para cada inteiro positivo n.
Demonstra c ao. Suponha que n ao exista tal ndice. Para cada ndice k (k = 1, 2, . . . , m),
existe um inteiro x
k
tal que F(x
k
) n ao e divisvel por a
k
. Assim, existem n umeros
d
k
= p

k
k
(onde p
k
s ao n umeros primos), tais que d
k
divide a
k
mas n ao divide F(x
k
). Se
existem potencias do mesmo primo entre esses n umeros, podemos apagar aquelas repeti-
das deixando apenas uma que tem expoente mnimo. Caso F(x) n ao seja divisvel por
uma potencia apagada, n ao ser a pela potencia que tem expoente mnimo. Essas dele coes
garatem que nossa nova colecao d
1
, d
2
, . . . , d
j
de potencias de primos contenham apenas
inteiros primos entre si, dois a dois. Pelo teorema chines dos restos, exite um inteiro N
tal que N x
k
(mod d)
k
, para k {1, 2, . . . , j}. Suponhamos que d
k
| F(N). Sabemos
que x y | F(x) F(y) e consequentemente N x
k
| F(N) F(x
k
). Como d
k
| N x
k
,
devemos ter d
k
| F(x
k
). Uma contradi cao! Logo, F(N) n ao e divisvel por nenhum d
k
e
isso contradiz a hip otese sobre os a
i
.
Problemas Propostos
Problema 14. Encontre o menor inteiro positivo (com a exce c ao de x = 1) que satisfa ca o
seguinte sistema de congruencias:
x 1 (mod 3)
x 1 (mod 5)
x 1 (mod 7)
5
POT 2012 - Teoria dos N umeros - Nvel 2 - Aula 11 - Samuel Feitosa
Problema 15. Encontre todas as solu c oes do sistema:
x 2 (mod 3)
x 3 (mod 5)
x 5 (mod 2)
Problema 16. Encontre todos os inteiros que deixam restos 1, 2 e 3 quando divididos por
3, 4 e 5, respectivamente.
Problema 17. Encontre todas as solu c oes do sistema:
3x 1 (mod 4)
2x 1 (mod 3)
4x 5 (mod 7)
Problema 18. Encontre todas as solu c oes das congruencias:
a) 20x 4 (mod 30).
b) 20x 30 (mod 4).
c) 353x 254 (mod 400).
Problema 19. Se a e escolhido ao acaso no conjunto {1, 2, 3, . . . , 14} e b e escolhido ao
acaso no conjunto {1, 2, . . . , 15}, qual a probabilidade de que a equa c ao ax b (mod 15)
possua pelo menos uma solu c ao?
Problema 20. Sejam a e b inteiros tais que mdc(a, b) = 1 e c > 0. Prove que existe um
inteiro x tal que mdc(a + bx, c) = 1.
Problema 21. Existem n inteiros consecutivos tal que cada um contem um fator primo
repetido k vezes?
Problema 22. Seja n um n umero natural arbitr ario. Prove que existe um par de naturais
(a, b) tais que mdc(a + r, b + s) > 1 r, s = 1, 2, . . . , n.
Problema 23. Um ponto (x, y) Z
2
e legal se mdc(x, y) = 1. Prove ou disprove: Dado
um inteiro positivo n, existe um ponto (a, b) Z
2
cuja dist ancia a todo ponto legal e pelo
menos n?
Problema 24. Sejam m
o
, m
1
, ..., m
r
inteiros positivos que s ao primos entre si, dois a dois.
Mostre que existem r+1 inteiros consecutivos s, s+1, ..., s+r tal que m
i
divide s+i para i =
0, 1, ..., r.
Problema 25. (Romenia 1995) Seja f : N{0, 1} N denida por f(n) = mmc[1, 2, ..., n].
Prove que para todo n 2, existem n n umeros consecutivos para os quais f e constante.
6
Problema 26. (OBM 2005) Dados os inteiros positivos a, c e o inteiro b, prove que existe
um inteiro positivo x tal que a
x
+ x b (mod c).
Problema 27. (Cone Sul 2003) Demonstrar que existe uma sequencia de inteiros positivos
x
1
, x
2
, . . . que satisfaz as duas condi c oes seguintes:
(a) contem exatamente uma vez cada um dos inteiros positivos,
(b) a soma parcial x
1
+ x
2
+ . . . x
n
e divisvel por n
n
.
Problema 28. (Rep ublica Tcheca e Eslovaca 1997) Mostre que existe uma sequencia cres-
cente {a
n
}

n=1
de n umeros naturais tais que para k 0 , a sequencia {a
n
+ k} contem um
n umero nito de primos.
Problema 29. Considere o inteiro c 1 e a sequencia denida por a
1
= c e a
i+1
= c
a
i
.
Mostre que esta sequencia se torna eventualmente constante quando a reduzimos modulo n
para algum inteiro positivo n (isto signica que a
m
a
j
(mod n) se m j).
Problema 30. (Coreia 1999) Encontre todos os inteiros n tais que 2
n
1 e um m ultiplo de
3 e
2
n
1
3
e um divisor de 4m
2
+ 1 para algum inteiro m.
Problema 31. (OBM 2006) Prove que, para todo inteiro n 2, o n umero de matrizes
quadradas 2 2 com entradas inteiras e pertencentes ao conjunto {0, 1, 2, . . . , n 1} que
tem determinante da forma kn + 1 para algum k inteiro e dado por

p primo
p | n

1
1
p
2

.
Problema 32. Encontre todos os subconjuntos S Z
+
tais que todas as somas de uma
quantidade nita de elementos de S(com possveis repetic oes de elementos) s ao n umeros
compostos.
Problema 33. Existe algum natural n para o qual existem n 1 progress oes aritmeticas
com razoes 2, 3, . . . , n tais que qualquer natural esta em pelo menos uma das progress oes?
Problema 34. Seja P(X) um polinomio com coecientes inteiros e k e um inteior qualquer.
Prove que existe um inteiro m tal que P(m) tem pelo menos k fatores primos distintos.
Acompanhe as dicussoes dos problemas propostos no f orum do POTI:
www.poti.impa.br/forum/
Referencias
[1] F. E. Brochero Martinez, C. G. Moreira, N. C. Saldanha, E. Tengan - Teoria dos
N umeros ? um passeio com primos e outros n umeros familiares pelo mundo inteiro,
Projeto Euclides, IMPA, 2010.
[2] E. Carneiro, O. Campos and F. Paiva, Olimpadas Cearenses de Matematica 1981-2005
(Nveis J unior e Senior), Ed. Realce, 2005.
[3] S. B. Feitosa, B. Holanda, Y. Lima and C. T. Magalhaes, Treinamento Cone Sul 2008.
Fortaleza, Ed. Realce, 2010.
[4] D. Fomin, A. Kirichenko, Leningrad Mathematical Olympiads 1987-1991, MathPro
Press, Westford, MA, 1994.
[5] D. Fomin, S. Genkin and I. Itenberg, Mathematical Circles, Mathematical Words, Vol.
7, American Mathematical Society, Boston, MA, 1966.
[6] I. Niven, H. S. Zuckerman, and H. L. Montgomery, An Introduction to the Theory of
Numbers.
Polos Olmpicos de Treinamento
Curso de Teoria dos Nmeros - Nvel 2
Prof. Samuel Feitosa
Aula
12
Equacoes Diofantinas II
Continuaremos nosso estudo das equac oes diofantinas abordando agora algumas equacoes
quadr aticas. Come caremos peloo classico problema das ternas pitag oricas.
Desejamos encontrar todas as solu coes (x, y, z) da equacao:
x
2
+ y
2
= z
2
,
em inteiros positivos. Seja d = mdc(x, y). Como d
2
| z
2
, segue que d | z e que

x
d
,
y
d
,
z
d

tambem e solu cao. Alem disso, podemos concluir que:


mdc(x/d, y/d) = mdc(x/d, z/d) = mdc(y/d, z/d) = 1.
Uma terna que e solu cao e possui a propriedade de que quaisquer dois de seus termos
s ao primos entre si, ser a chamada de solu cao primitiva. Assim, toda solu cao (x, y, z) e da
forma (dx
1
, dy
1
, dz
1
) onde (x
1
, y
1
, z
1
) e uma solu cao primitiva. Para cumprimirmos nosso
objetivo, bastara nos concentrarmos em encontrar todas as solu coes primitivas. Analisando
a equacao m odulo 4 e lembrando que todo quadrado perfeito pode deixar apenas os restos 0
ou 1, conclumos que exatamente um dentre x e y e par. Suponha sem perda de generalidade
que y seja par. Fatorando a equacao, obtemos:
z + x
2

z x
2
=

y
2

2
Como mdc((z +x)/2, (z x)/2) = 1, conclumos que (z +x)/2 e (z x)/2 devem ser ambos
quadrados perfeitos, i.e., existem inteiros positivos r e s, com r > s e mdc(r, s) = 1, tais que
(z + x)/2 = r
2
e (z x)/2 = s
2
(veja o primeiro problema proposto). Consequentemente,
x = r
2
s
2
, y = 2rs e z = r
2
+ s
2
. Reciprocamente, se (x, y, z) = (r
2
s
2
, 2rs, r
2
+ s
2
),
temos:
x
2
+ y
2
= (r
2
s
2
)
2
+ (2rs)
2
= (r
2
+ s
2
)
2
= z
2
.
O pr oximo teorema resume nossa discussao original:
POT 2012 - Teoria dos N umeros - Nvel 2 - Aula 12 - Samuel Feitosa
Teorema 1. Todas as soluc oes primitivas de x
2
+y
2
= z
2
com y par s ao da forma x = r
2
s
2
,
y = 2rs e z = r
2
+ s
2
, onde r e s s ao inteiros de paridade oposta com r > s > 0 e
mdc(r, s) = 1.
Exemplo 2. Encontre todas as ternas pitag oricas (a, b, c) tais que a + b + c = 1000.
Seja k = mdc(a, b, c) e suponha sem perda de generalidade que b/k e par. Pelo teorema
anterior, (a, b, c) = (k(x
2
y
2
), k(2xy), k(x
2
+y
2
)), onde x > y, mdc(x, y) = 1 e pelo menos
um dentre x e y par. Assim, (x
2
y
2
) +2xy +(x
2
+y
2
) = 2x(x +y) e um divisor de 1000.
Com mais raz ao, x(x +y) | 500. Usando que mdc(x, x +y) = 1 e a fatoracao em primos de
500, podemos concluir que um deles e uma potencia de 5 e o outro uma potencia de 2. Veja
que x n ao pode ser uma potencia de 5 pois nesse caso y deveria ser mpar para garantir
que x+y seja uma potencia de 2. Assim, x | 500 e x = 2
k
, produzindo como possibilidades
x = 1, 2 ou 4. Analisando cada um desses casos e levando em conta que y < x, e facil
encontrar que x = 4 e y = 1 s ao as unicas op coes possveis. Nesse caso, x = 15, y = 8 e
z = 17. Consequentemente, (a, b, c) = (20 15, 20 8, 20 17).
Exemplo 3. Mostre que se a, b e c s ao inteiros positivos tais que a
2
+ b
2
= c
2
, entao
(ab)
4
+ (bc)
4
+ (ca)
4
e um quadrado perfeito.
Veja que:
(ab)
4
+ (bc)
4
+ (ca)
4
= (a
2
b
2
+ b
4
)
2
+ (a
2
b
2
)
2
+ (a
2
b
2
+ a
4
)
2
= (a
4
+ a
2
b
2
+ b
4
)
2
.
Exemplo 4. Encontre todas as soluc oes de x
2
+ 2y
2
= z
2
em inteiros positivos com
mdc(x, y, z) = 1.
Como 2y
2
0 (mod 2), devemos ter x z (mod 2). Alem disso, se fosse x z 0
(mod 2) teramos 4 | z
2
x
2
= 2y
2
e consequentemente 2 | y, contradizendo a hip ostese
mdc(x, y, z) = 1. Fatorando a express ao, temos:
2y
2
= (z x)(z + x).
Como mdc(x, z) = 1 e ambos s ao mpares; mdc(z x, z + x) = 2 e apenas um deles e
congruo `a 2 (mod 4). Temos dois casos a considerar: 1) z + x 0 (mod 4) e z x 2
(mod 4). Nesse caso, y
2
= (z x)/2 (z + x) com mdc((z x)/2, (z + x)) = 1. Da,
existem inteiros positivos r e s tais que (z x)/2 = r
2
e (z + x)/2 = s
2
, produzindo a
solu cao (x, y, z) = ((s
2
2r
2
)/2, rs, (2r
2
+ s
2
)/2) com mdc(r, s) = 1 e s 0 (mod 2). Um
raciocnio an alogo para o caso z + x 2 (mod 4) e z x 0 (mod 4) produz (x, y, z) =
((2s
2
r
2
)/2, rs, (r
2
+ 2s
2
)/2) com mdc(r, s) = 1 e r 0 (mod 2).
Problema 5. (USAMO 1976) Encontre todas as soluc oes naturais da equa c ao
a
2
+ b
2
+ c
2
= a
2
b
2
.
A equacao pode ser reescrita como:
c
2
= (a
2
1)(b
2
1) 1.
2
POT 2012 - Teoria dos N umeros - Nvel 2 - Aula 12 - Samuel Feitosa
Se pelo menos um dentre a ou b e mpar, teremos c
2
3 (mod 4). Como os quadrados
perfeitos s o podem deixar resto 0 ou 1 (mod 4), temos um absurdo. Portanto, a, b e
consequentemente c s ao n umeros pares. Seja k o maior inteiro tal que 2
k
divida esses tres
n umeros. Assim, a = 2
x
, b = 2
k
y, c = 2
k
z onde pelo menos um dentre x, y e z mpar.
Assim,
x
2
+ y
2
+ z
2
= 2
2r
x
2
y
2
.
Como r > 0, x
2
+ y
2
+ z
2
0 (mod 4). Entretanto, isso n ao e possvel se um dentre os
x, y, z e mpar pois a soma s o poderia ser congruente `a 1, 2, 3 (mod 4).
Exemplo 6. (Extrado de [1]) Determine todas as ternas (a, b, c) de inteiros positivos tais
que a
2
= 2
b
+ c
4
.
Como a
2
= 2
b
+ c
4
(a c
2
)(a + c
2
) = 2
b
, pelo Teorema Fundamental da Aritmetica
existem dois naturais m > n tais que m + n = b, a c
2
= 2
n
e a + c
2
= 2
m
. Subtraindo as
duas ultimas equacoes, obtemos que 2c
2
= 2
m
2
n
, assim c
2
= 2
n1
(2
mn
1). Como 2
n1
e 2
mn
1 s ao primos entre si e o seu produto e um quadrado perfeito (i.e. os expoentes
das potencias de primos distintos s ao pares), novamente pelo Teorema Fundamental da
Aritmetica 2
n1
e 2
mn
1 devem ser ambos quadrados perfeitos, logo n 1 e par e
2
mn
1 = (2k1)
2
para algum inteiro positivo k. Como 2
mn
= (2k1)
2
+1 = 4k(k1)+2
e divisvel por 2 mas n ao por 4, temos m n = 1. Assim, fazendo n 1 = 2t, temos que
todas as solu coes s ao da forma (a, b, c) = (3 2
2t
, 4t +3, 2
t
) com t N e e facil vericar que
todos os n umeros desta forma s ao solu coes.
O pr oximo exemplo ilustrara o metodo da descida de Fermat que faz uso do princpio
da boa ordenacao: todo subconjunto n ao vazio de inteiros positivos possui um elemento
mnimo.
Exemplo 7. Determine todas as solu c oes da equa c ao x
4
+ y
4
= z
2
em inteiros positivos
com mdc(x, y) = 1.
Como (x
2
)
2
+(y
2
)
2
= z
2
e mdc(x
2
, y
2
) = 1, podemos usar o primeiro teorema para concluir
que existem u e v tais que x
2
= u
2
v
2
, y
2
= 2uv, z = u
2
+ v
2
, u > v > 0 e mdc(u, v) = 1
(Estamos assumindo sem perda de generalidade que x e mpar). Se u e par, entao v ser a
mpar e teremos x
2
3 (mod 4). Como isso e um absurdo, u deve ser mpar e v deve ser
par. Sendo assim, (y/2)
2
= u v/2 com mdc(u, v/2) = 1. Devemos ter u = r
2
, v/2 = s
2
,
com mdc(r, s) = 1, r, s > 0, r mpar e y = 2rs. Alem disso, como x
2
+ v
2
= u
2
, obtemos
x
2
+4s
2
= r
4
. Como mdc(r, 2s) = 1, novamente pelo primeiro teorema, existem m e n tais
que x = m
2
n
2
, 2s
2
= 2mn e r
2
= m
2
+ n
2
com mdc(m, n) = 1 e m > n > 0. Como
mn = s
2
, podemos escrever m = f
2
e n = g
2
com f, g > 0 e mdc(f, g) = 1. Portanto,
r
2
= f
4
+ g
4
. Note que dada a solu cao em inteiros positivos (x, y, z), obtivemos outra
solu cao (f, g, r), tambem nos inteiros positivos, com 0 < r < z. Isso nos diz que existe
uma innidade decrescente de possveis valores para o inteiro positivo z e naturalmente
obtemos uma contradi cao do princpio da boa ordenacao. Sendo assim, a equacao anterior
n ao possui solu cao nos inteiros positivos.
Observacao 8. Outra maneira de formalizar o argumento anterior e escolher dentre as
ternas nos inteiros positivos que s ao soluc oes, aquela com z mnimo. A nova terna (f, g, r)
caracterizaria um absurdo.
3
POT 2012 - Teoria dos N umeros - Nvel 2 - Aula 12 - Samuel Feitosa
Exemplo 9. Prove que para todo inteiro n > 2, existem inteiros positivos p e q tais que
n
2
+ q
2
= p
2
.
Fatorando a express ao, obtemos n
2
= (p q)(p + q). Se n e mpar, podemos encontrar p e
q tais que p + q = n
2
e p q = 1, bastando para isso resolver o sistema originado, obtendo
(n, q, p) = (n,
n
2
1
2
,
n
2
+1
2
). Se n e par, podemos fazer algo semelhante e encontrar p e q tais
que p + q = n
2
/2 e p q = 2, cuja solu cao e (n, q, p) = (n,
n
2
4
1,
n
2
4
+ 1).
Exemplo 10. (Extrado de [3]) Prove que a equa c ao
x
2
+ y
2
+ z
2
+ w
2
= 2xyzw (1)
n ao possui solu c oes inteiras positivas.
Por contradi cao, suponha que (1) possua pelo menos uma solu cao n ao-trivial, digamos
(x
0
, y
0
, z
0
, w
0
). Se x
0
, y
0
, z
0
, w
0
forem todos mpares, o lado esquerdo e um m ultiplo de 4
e o lado direito n ao. Se apenas um ou tres deles forem pares, o lado esquerdo e mpar e o
direito e par. Se dois deles forem pares e dois forem mpares, o lado direito e um m ultiplo
de quatro e o esquerdo n ao. Desse modo, x
0
, y
0
, z
0
, w
0
s ao todos pares, ou seja, x
0
= 2x
1
,
y
0
= 2y
1
, z
0
= 2z
1
e w
0
= 2w
1
. Substituindo em (1) e dividindo por quatro, conclumos
que x
1
, y
1
, z
1
, w
1
satisfazem a igualdade
x
2
1
+ y
2
1
+ z
2
1
+ w
2
1
= 8x
1
y
1
z
1
w
1
.
Com uma an alise de paridades an aloga `a acima, obtemos x
1
= 2x
2
, y
1
= 2y
2
, z
1
= 2z
2
e
w
1
= 2w
2
, e da
x
2
2
+ y
2
2
+ z
2
2
+ w
2
2
= 32x
2
y
2
z
2
w
2
.
Procedendo dessa maneira, x
0
, y
0
, z
0
, w
0
devem ser todos m ultiplos de 2
n
, qualquer que seja
n 1. Entao x
0
= y
0
= z
0
= w
0
= 0, absurdo.
Exemplo 11. (Extrado de [3]) Encontre todas as quadr uplas (x, y, z, k) de n umeros inteiros,
com x, y, z > 0 e k 0, tais que
x
6
+ y
6
+ z
6
= 4826 7
k
.
Vamos mostrar o seguinte fato:
(x, y, z, k) e solu cao, com k 1 (x/7, y/7, z/7, k 6) e solu cao,
e nesse caso k 6.
(=) Temos x
6
+ y
6
+ z
6
0 (mod 7). Como x
6
, y
6
, z
6
0 ou 1 (mod 7), devemos ter
x, y, z m ultiplos de 7. Da, 7
6
|4826 7
k
7
6
|7
k
k 6. Ademais, vale a igualdade

x
7

6
+

y
7

6
+

z
7

6
= 4826 7
k6
,
4
ou seja, (x/7, y/7, z/7, k 6) tambem e solu cao.
(=) Claro.
O fato acima garante que podemos ir subtraindo 6 de k e retirando um fator 7 de x, y,
z enquanto k 1, ate que o expoente de 7 no lado direito da igualdade seja 0. Em outras
palavras, existe n 0 tal que k = 6n, com x = 7
n
x
0
, y = 7
n
y
0
, z = 7
n
z
0
, e
x
0
6
+ y
0
6
+ z
0
6
= 4826.
A equacao acima s o tem a solu cao (1, 3, 4) e suas permuta coes. Assim, as solu coes da
equacao original s ao (7
n
, 3 7
n
, 4 7
n
, 6n), n 0, e suas permuta coes nas tres primeiras
coordenadas.
Problemas Propostos
Problema 12. Mostre que se a b = x
2
e mdc(a, b) = 1 entao existem r e s tais que a = r
2
e b = s
2
.
Problema 13. Prove que todas as soluc oes positivas da equa c ao
1
x
2
+
1
y
2
=
1
z
2
com
mdc(x, y, z) = 1 s ao dadas por
(x, y, z) = (r
4
s
4
, 2rs(r
2
+ s
2
), rs(r
2
s
2
))
ou
(x, y, z) = (2rs(r
2
+ s
2
), r
4
s
4
, rs(r
2
s
2
)),
onde r > s > 0, mdc(r, s) = 1 e r e s de paridades opostas.
Problema 14. Encontre todos os pares de racionais (x, y) tais que x
2
+ y
2
= 1.
Problema 15. Resolva simultaneamente em inteiros positivos:
a
2
+ b
2
= c
2
a
2
+ c
2
= d
2
onde a, b, c e d s ao inteiros positivos relativamente primos entre si dois dois.
Problema 16. (Torneio das Cidades 1997) Prove que a equa c ao
x
2
+ y
2
z
2
= 1997
tem innitas solu c oes inteiras (x, y, z).
Problema 17. Encontre todas as solu c oes inteiras de x
2
+ y
2
+ z
2
= t
2
.
Problema 18. Encontre todas as solu c oes de 5m
2
+ n
2
= 5
2011
Problema 19. Encontre todas as solu c oes em n umeros naturais m e n da equa c ao:
m
2
= 1 + 2 + . . . + n.
Referencias
[1] F. E. Brochero Martinez, C. G. Moreira, N. C. Saldanha, E. Tengan - Teoria dos
N umeros: um passeio com primos e outros n umeros familiares pelo mundo inteiro,
Projeto Euclides, IMPA, 2010.
[2] E. Carneiro, O. Campos and F. Paiva, Olimpadas Cearenses de Matematica 1981-2005
(Nveis J unior e Senior), Ed. Realce, 2005.
[3] S. B. Feitosa, B. Holanda, Y. Lima and C. T. Magalhaes, Treinamento Cone Sul 2008.
Fortaleza, Ed. Realce, 2010.
[4] D. Fomin, A. Kirichenko, Leningrad Mathematical Olympiads 1987-1991, MathPro
Press, Westford, MA, 1994.
[5] D. Fomin, S. Genkin and I. Itenberg, Mathematical Circles, Mathematical Words, Vol.
7, American Mathematical Society, Boston, MA, 1966.
[6] I. Niven, H. S. Zuckerman, and H. L. Montgomery, An Introduction to the Theory of
Numbers.
Polos Olmpicos de Treinamento
Curso de Teoria dos Nmeros - Nvel 2
Prof. Samuel Feitosa
Aula
13
Equac oes Diofantinas III
Ja estudamos as equa coes diofantinas lineares e equa c oes em que alguma fatora cao conveni-
ente poderia facilitar a busca por solucoes. Nesta aula, estaremos interessados em encontrar
m odulos convenientes para analisar os termos de uma equa cao.
Exemplo 1. Encontre todas as solu c oes em inteiros da equa c ao x
2
7y = 1004.
Analisando os restos na divisao por 7, obtemos x
2
3 (mod 7). Entretando, os unicos
inteiros que s ao restos de quadrados perfeitos na divisao por 7 s ao 0, 1, 2 e 4. Como
3 1004 (mod 7) nao faz parte dessa lista, nao existem soulcoes inteiras para a equa cao.
Exemplo 2. Encontre todas as solu c oes em inteiros da equa c ao x
3
+ 98y
2
+ 5 = 0.
Analisemos os possveis restos de x
3
(mod 7) fazendo uma tabela dos restos correspondentes
de x e x
3
:
x 0 1 2 3 4 5 6
x
3
0 1 1 6 1 6 6
Como os unicos restos possveis s ao 0, 1, 1 (mod 7), o lado esquerdo da equa cao s o pode
deixar resto 5, 6, 4 (mod 7). Como o resto do lado direito nao faz parte dessa lista, nao
existem solucoes em inteiros.
Exemplo 3. Prove que a equa c ao x
2
= 3y
2
+ 8 n ao possui solu c oes em inteiros x e y.
Analisando o resto na divisao por 3, obtemos x
2
2 (mod 3). Como os unicos restos de
um quadrado por 3 s ao 0 e 1, nao existem solucoes em inteiros.
Nos proximo problema, usaremos congruencias para encontrarmos informa coes sobre as
incognitas envolvidas nos expoentes e buscaremos alguma fatora cao apropriada para reduzir
o problema `a resolu cao de um sistema de equa coes.
Exemplo 4. Encontre todas as solu c oes em inteiros positivos da equa c ao 3
m
+ 7 = 2
n
POT 2012 - Teoria dos N umeros - Nvel 2 - Aula 13 - Samuel Feitosa
Analisando o resto m odulo 3 do lado esquerdo, podemos concluir que 2
n
1 (mod 3).
Como 2
n
(1)
n
(mod 3), conclumos que n e par, ou seja, n = 2k, para algum k N.
Assim, como o lado direito e m ultiplo de 4, podemos concluir que:
3
m
7 (mod 4)
(1)
m
1 (mod 4)
Logo, m e par, ou seja, m = 2t, para algum t N. Usando diferen ca de quadrados,
podemos escrever:
7 = (2
k
3
t
)(2
k
+ 3
t
).
Como 7 e primo, temos as seguintes op coes:
7 = 2
k
+ 3
t
2
k
3
t
= 1
1 = 2
k
+ 3
t
2
k
3
t
= 7
Em ambos os casos, 8 = 2
k+1
e da k = 2. Substituindo nas equa coes, obtemos solucao
apenas no primeiro caso com t = 1. Assim, (m, n) = (2, 4).
Exemplo 5. Encontre todas as solu c oes em inteiros positivos da equa c ao 3 2
m
+ 1 = n
2
.
Analisandoa equa cao m odulo 3, n
2
1 (mod 3) e assim, n 1 (mod 3). No primeiro
caso, se n = 3k + 2, temos 3 2
m
+ 1 = n
2
= 9k
2
+ 12k + 4 e da 2
m
= (3k + 1)(k + 1).
Como o lado esquerdo possui apenas um fatores 2, temos 3k + 1 = 2
i
, k + 1 = 2
j
,com
j i. Da, 3 2
j
2
i
= 2. Se j = i, temos 2
i+1
= 2 e consequentemente i = 0 produzindo
k = 0 e (m, n) = (0, 2). Se j < i, temos j = 1 pois o lado esquerdo possui um unico fator
2 e por conseguinte, i = 2, (m, n) = (3, 5). No segundo caso, quando n = 3k +1, e tratado
analogamente produzindo apenas a nova solucao (m, n) = (4, 7).
Exemplo 6. Encontre todas as solu c oes da equa c ao x
2
xy + y = 3 em inteiros x, y.
Fixado o valor de y, podemos encontrar os valores de x usando a formula de B askara. Como
x e inteiro, o discriminante y
2
4(y 3) = (y 2)
2
+ 8 deve ser um quadrado perfeito,
digamos z
2
. Assim,
z
2
(y 2)
2
= (z y + 2)(z + y 2) = 8.
Como z y +2 e z +y 2 possuem a mesma paridade, o produto anterior dever (2) (4).
Em qualquer caso, somando ambos os termos, obtemos 2z = 6 e z = 3. Logo, y2 = 1.
Substituindo os valores de y na equa cao original, obtemos os valores correspondentes para
x. As solucoes s ao: (x, y) = (2, 1), (1, 1), (0, 3), (3, 3).
Exemplo 7. (Hungria 1969) Seja n um inteiro positivo. Prove que se 2+2

28n
2
+ 1 e um
inteiro, entao e um quadrado perfeito.
2
POT 2012 - Teoria dos N umeros - Nvel 2 - Aula 13 - Samuel Feitosa
Necessariamente

28n
2
+ 1 deve ser racional e para isso 28n
2
+ 1 deve ser um quadrado
perfeito. Assim,
28n
2
+ 1 = t
2
7n
2
=

t 1
2

t + 1
2

Como 7 e primo, 7 |
t + 1
2
ou que 7 |
t 1
2
. No primeiro caso,
n
2
+ 1 =

t + 1
14

t 1
2

Alem disso, como mdc((t 1)/2, (t + 1)/2) = 1, existem a e b tais que


a
2
=
t + 1
14
b
2
=
t 1
2
Da, 7a
2
b
2
= 1 e b
2
1 (mod 7). Como quadrados perfeitos s o podem deixar restos
0, 1, 2, 4 (mod 7), esse caso nao gera solucoes. No segundo caso,
a
2
=
t + 1
2
b
2
=
t 1
14
.
Logo, 2 + 2

28n
2
+ 1 = 2 + 2t = 2 + 2(2a
2
1) = (2a)
2
.
Exemplo 8. (Reino Unido 1996) Encontre todas as solu c oes em inteiros n ao negativos
x, y, z da equa c ao:
2
x
+ 3
y
= z
2
.
Se y = 0, entao 2
x
= z
2
1 = (z 1)(z + 1). Analisando a fatora cao em primos, existem
i, j, com i > j, tais que z + 1 = 2
i
e z 1 = 2
j
. A diferen ca das duas equa coes produz
2 = 2
i
2
j
= 2
j
(2
ij
1). Como o lado esquerdo possui apenas um fator 2, j = 1 e
i j = 1. Nossa primeira solucao encontrada e (x, y, z) = (3, 0, 3). Se y > 0, 2
x
z
2
(mod 3). Como 2
x
1 (mod 3) e z
2
0, 1 (mod 3) temos, 2
x
z
2
1. Isso implica
que x e par, ou seja, x = 2m. Fatorando, obtemos:
3
y
= z
2
2
2m
= (z 2
m
)(z + 2
m
)
Novamente, analisando a fatora cao em primos, existem l e k, com l < k, tais que z 2
m
=
3
l
, z + 2
m
= 3
k
. A diferen ca das duas equa coes produz 2
m+1
= 3
l
(3
kl
1). Novamente
3
POT 2012 - Teoria dos N umeros - Nvel 2 - Aula 13 - Samuel Feitosa
analisando a fatora cao em primos, l = 0 e 2
m+1
= 3
k
1. Se m = 0, temos k = 1 e
(x, y, z) = (0, 1, 2). Se m > 0,
3
k
= 1 (mod 4)
(1)
k
= 1 (mod 4).
e devemos ter k par, ou seja, existe t tal que k = 2t. Fatorando novamente, 2
m+1
=
(3
k
1)(3
k
+ 1). Escrevendo 3
k
+ 1 = 2
p
e 3
k
1 = 2
q
, temos 2 = 2
q
(2
pq
1). Veja
que ja tratamos essa equa cao no incio e assim podemos concluir que q = 1 e p q = 1.
Produzindo a solucao (x, y, z) = (4, 2, 5).
Nos proximos dois problemas, contruiremos solucoes indutivamente.
Exemplo 9. (Bulgaria) Prove que para qualquer n umero natural n 3, existem n umeros
naturais mpares x
n
e y
n
tais que 7x
2
n
+ y
2
n
= 2
n
.
Para n = 3, basta tomar x
1
= y
1
= 1. Suponha que tenhamos encontrado x
k
e y
k
mpares,
satisfazendo
7x
2
k
+ y
2
k
= 2
k
.
Um dos n umeros (x
k
+ y
k
)/2, (x
k
y
k
)/2 e mpar e assim podemos escolher um deles de
modo a satisfazer o enunciado para k + 1:
7

x
k
y
k
2

2
+

x
k
y
k
2

2
= 2(7x
2
k
+ y
2
k
) = 2
k+1
.
Exemplo 10. Mostre que existe uma sequencia innita de inteiros positivos a
1
, a
2
, . . . tais
que a
2
1
+ a
2
2
+ . . . + a
2
n
e um quadrado perfeito para todo n natural.
Denamos a
1
= 3. Suponha que a sequencia ja esteja denida para a
1
, a
2
, . . . , a
k
com
a
2
1
+ a
2
2
+ . . . + a
2
k
= (2t + 1)
2
.
Vejamos que podemos denir o proximo termo de modo que a soma de todos os primeiros
k + 1 termos ao quadrado ainda seja um quadrado perfeito de um inteiro mpar. Basta
fazer a
k+1
= 2t
2
+ 2t. Veja que:
a
2
1
+ a
2
2
+ . . . + a
2
k
+ a
2
k+1
= (2t + 1)
2
+ (2t
2
+ 2t)
2
= (2t
2
+ 2t + 1)
2
.
que e novamente o quadrado de um mpar.
Problemas Propostos
Problema 11. Encontre todas as solu c oes em inteiros x, y, z, t da equa c ao:
x
2
+ y
2
+ z
2
= 8t 1.
4
POT 2012 - Teoria dos N umeros - Nvel 2 - Aula 13 - Samuel Feitosa
Problema 12. Encontre todas as solu c oes em inteiros positivos da equa c ao
1
a
+
1
b
+
1
c
= 1.
Problema 13. Encontre todas as solu c oes em inteiros de x
2
y
2
1988
Problema 14. Mostre que para todo inteiro z, existem inteiros x e y satisfazendo x
2
y
2
=
z
3
Problema 15. Encontre todas as solu c oes de 1 + x + x
2
+ x
3
= 2
y
em inteiros positivos x
e y.
Problema 16. Mostre que a equa c ao diofantina 5m
2
6mn + 7n
2
= 1988, n ao possui
solu c ao nos inteiros.
Problema 17. (R ussia 1996) Sejam x, y, p, n, k n umeros naturais tais que
x
n
y
n
= p
k
.
Prove que se n > 1 e mpar, e p e um primo mpar, entao n e uma potencia de p.
Problema 18. (R ussia 1997) Encontre todas as solu c oes inteiras da equa c ao
(x
2
y
2
)
2
= 1 + 16y.
Problema 19. (OBM 2009) Prove que n ao existem inteiros positivos x e y tais que x
3
+y
3
=
2
2009
.
Apendice: A conjectura de Catalan
Em alguns dos problemas anteriores, nos deparamos com a quest ao de encontrarmos duas
potencias perfeitas consecutivas nao triviais. As unicas solucoes que apareceram foram
2
3
= 8 e 3
2
= 9. Em 1844, Eug`ene Catalan conjecturou que essa seria a unica solucao.
Recentemente, tal conjectura se mostrou verdadeira atr aves do:
Teorema 20. (Mihalnescu - 2002)Existe uma unica solu c ao nos n umeros naturais de
x
a
y
b
= 1,
com x, a, y, b > 1 que e (x, y, a, b) = (3, 2, 2, 3).
Problema 21. Encontre toda as solu c oes em inteiros positivos da seguinte equa c ao diofan-
tina:
2y
2
= x
4
+ x.
5
Polos Olmpicos de Treinamento
Curso de Teoria dos Nmeros - Nvel 2
Prof. Samuel Feitosa
Aula
14
A Funcao Parte Inteira - I
1 O jogo de Wytho
O objetivo da aula de hoje e resolver o seguinte problema:
Exemplo 1. Dois jogadores jogam alternadamente removendo pedras de duas pilhas sobre
uma mesa. Na sua vez, cada jogador pode remover qualquer quantidade de pedras de uma
pilha ou igual n umero de pedras de ambas as pilhas. O ganhador e aquele que retirar a
ultima pedra. Determine todas posic oes perdedoras.
Uma boa estrategia para identicar as posicoes perdedoras nesse jogo, e associar o mo-
vimento dos jogadores ao movimento de uma peca em um tabuleiro. Suponha que ini-
cialmente as duas pilhas possuem 5 e 7 pedras. Colocaremos uma peca no canto direito
superior em um tabuleiro 8 6. O movimento de retirar x pedras da coluna de 5 sera
traduzido como um deslocamento vertical de x casas para baixo, enquanto que a mesma
retirada da outra coluna sera traduzido como um movimento horizontal para a esquerda
de mesmo deslocamento. Um movimento de retirada de x pedras de ambas as colunas
sera traduzido como um deslocamento diagonal da direita para a esquerda dessa mesma
quantidade de casas. O jogo terminar a quando a peca chegar na casa do canto extremo
esquerdo simbolizando que ambas as colunas estao com 0 pedras.
A posicao (0, 0) e perdedora porque uma vez que um jogador a receba, ele ter a perdido
o jogo. Qualquer posicao do tipo (x, 0), (0, x) ou (x, x), com x > 0, sera uma posicao
vencedora. Marquemos essas posicoes no tabuleiro:
POT 2012 - Teoria dos N umeros - Nvel 2 - Aula 14 - Samuel Feitosa 2 DEFINIC

AO E PROPRIEDADES

+
+
+
+
+
+
+
+
+
+
+ + + + + + +
As proximas posicoes perdedoras que encontramos s ao (1, 2),(2, 1). A partir dessa nova
posicao, podemos preencher o tabuleiro com as novas posi coes vencedoras.

+
+
+
+
+
+
+
+
+
+
+ + + + + + +

+
+
+

+
+
+
+
+
+
+
+
+
+
+
+ + + + + +
+ + + + +
As proximas posicoes perdedoras que encontramos s ao (3, 5) e (5, 3). Como existe simetria
entre as duas pilhas, basta procurarmos as posicoes perdedoras (x, y) com x < y. Repetindo
o processo anterior, podemos listar as primeiras posicoes perdedoras ordenadas (x
n
, y
n
) com
x
n
< y
n
.
n 0 1 2 3 4 5 6 7 8 9 10 11 12
x
n
0 1 3 4 6 8 9 11 12 14 16 17 19
y
n
0 2 5 7 10 13 15 18 20 23 26 28 31
As proximas secoes nos ajudarao a estabelecer alguma padr ao entre os valores de (x
n
, y
n
)
em funcao de n.
2 Denicao e Propriedades
Deni cao 2. A parte inteira de um n umero real x e o maior inteiro x que n ao e maior
que x. Denimos a parte fracion aria {x} de x por {x} = x x. (exemplos: 3 =
3 , 3, 5 = 3 e 4, 7 = 5)
Teorema 3. Sejam x e y n umeros reais. Ent ao:
1. x x < x + 1 e 0 {x} < 1.
2. x + m = x + m se m e um inteiro.
3. x +y x + y x +y + 1.
4.
_
x
m
_
=
_
x
m
_
se m e um inteiro positivo.
5. Se n e a sao inteiros positivos,
_
n
a
_
e o n umero de inteiros entre 1, 2, ..., n que sao
divisveis por a.
2
POT 2012 - Teoria dos N umeros - Nvel 2 - Aula 14 - Samuel Feitosa 2 DEFINIC

AO E PROPRIEDADES
Demonstrac ao. Os primeiros dois itens decorrem facilmente da denicao e serao deixados
a cargo do leitor.
Para provar (3), veja que:
x +y x +y +{x} +{y}
= x +y +{x} +{y}
= x + y.
Como {x} +{y} < 2, {x} +{y} 1 e da:
x + y = x +y +{x} +{y}
x +y + 1.
Para provar (4), seja x = qm + r com 0 r < m1, entao:
_
x
m
_
=
_
q +
r
m
_
= q.
Como 0 {x} < 1,
q = q +
_
r +{x}
m
_
=
_
qm + r +{x}
m
_
=
_
x
m
_
.
Finalmente, para provar (5), sejam a, 2a, ..., ja todos os inteiros positivos n que s ao
divisveis por a. Entao,
ja n < (j + 1)a j
n
a
< j + 1
j =
_
n
a
_
.
Observacao 4. Em alguns dos problemas desta se c ao, sera usada a notac ao de somat orio.
Recomenda-se que o professor escrevar por extenso os primeiros somat orios ate que os
alunos se sintam confort aveis com a manipulac ao dos ndices.
Teorema 5 (Formula de Polignac). Seja p um primo. Ent ao o maior expoente p na fa-
torac ao em primos de n! e:
t
p
(n) =

i=1
_
n
p
i
_
.
Demonstrac ao. O que signica
_
n
p
i
_
? Ele conta o n umero de inteiros positivos menores ou
iguais a n divisveis por p
i
. Cada m ultiplo de p contribui com um expoente 1 para p em n!,
cada m ultiplo de p
2
contribui com expoente 2 para p em n! e assim sucessivamente. Entao,

i=1
_
n
p
i
_
e a soma de todas essas contribuicoes (veja que um m ultiplo de p
i
e contado i
vezes em
_
n
p
_
,
_
n
p
2
_
, ...,
_
n
p
i
_
).
3
POT 2012 - Teoria dos N umeros - Nvel 2 - Aula 14 - Samuel Feitosa 2 DEFINIC

AO E PROPRIEDADES
Observacao 6. Se p e primo e p

e a maior potencia de p que divide n, usaremos a notac ao


p

n.
Exemplo 7. Em quantos zeros termina a representac ao decimal de 1000!?
Para determinarmos o n umero de zeros, basta determinarmos a maior potencia de 10 que
divide 1000!. Existem mais fatores 2 do que fatores 5 e assim bastar a encontrarmos o
expoente de 5 na fatorac ao de 1000!. Pelo teorema anterior, tal n umero e:

i=1
_
1000
5
i
_
= 200 + 40 + 8 + 1 = 249.
Exemplo 8. Mostre que
k1

i=0
_
x +
i
k
_
= kx.
Sejam a = x, b = {x} e j {0, 1, . . . , k 1} tal que
j
k
b <
j + 1
k
. Ent ao kb = j.
Alem disso, b + i/k = 0 se i < k j e b + j/k = 1 se k j i k 1. Da,
k1

i=0
b + i/k =
k1

i=kj
b + i/k
= j
= kb
Exemplo 9. Mostre que x + y +x +y 2x +2y.
Sejam a = {x} e b = {y}. A desigualdade e equivalente `a
2x + 2y +a +b +a + b 2x + 2y +2a +2b.
que pode ser reescrita como:
a +b +a + b 2a +2b.
Temos que 0 a + b < 2 e a = b = 0. Se a + b = 1 segue que pelo menos um
dentre a, b e maior ou igual `a 1/2. Da,
a +b +a + b = 1
2a +2b.
Caso contrario, a +b +a + b = 0 e a desigualdade segue.
Exemplo 10. Mostre que se m e n sao inteiros positivos, entao
(2m)!(2n)!
(m)!(n)!(m + n)!
e um
inteiro.
4
POT 2012 - Teoria dos N umeros - Nvel 2 - Aula 14 - Samuel Feitosa 2 DEFINIC

AO E PROPRIEDADES
Pelo teorema anterior, basta mostrarmos que:
_
2m
p
k
_
+
_
2n
p
k
_

_
m
p
k
_
+
_
n
p
k
_
+
_
m + n
p
k
_
,
para todo primo p e todo inteiro k. A desigualdade segue do exemplo anterior.
Exemplo 11. Prove que
_
2n
n
_
divide MMC{1, 2, ..., 2n}.
Seja p um primo. Se p

_
2n
n
_
e p

2n < p
+1
, pelo teorema anterior, temos:
=

j=1
_
2n
p
j
_
2
_
n
p
j
_
=

j=1
_
2n
p
j
_
2
_
n
p
j
_
,
pois 2x 2x = 0 ou 1. Como p

MMC{1, 2, ..., 2n} e , segue o resultado.


Exemplo 12. Mostre que

k=1
_
n
2
i
+
1
2
_
= n.
O n umero de inteiros que sao m ultiplos de 2
k
, mas n ao de 2
k+1
e n/2
k
n/2
k+1
. Se
n = 2
k+1
q +r, esse n umero e q +r/2
k
. Para x [0, 1), e verdade que 2x = x +1/2.
Assim,
n/2
k
n/2
k+1
= q +r/2
k

= r/2
k
+ 1/2 + q
= r/2
k+1
+ 1/2
Somando a quantidade de n umeros que sao m ultiplos de 2
k
mas n ao de 2
k+1
entre 1 e n
para k = 0, 1, . . . obtemos a quantidade total de n umeros, ou seja,

k=1
_
n
2
i
+
1
2
_
= n.
Teorema 13. Seja v
p
(n) a soma dos dgitos da representac ao de n na base p. Mostre que
o expoente de p na fatorac ao em primos de n! e
n v
p
n
p 1
.
Demonstrac ao. Seja k
p
(n!) o maior expoente de p que divide n!. Considere a representa cao
de n na base p: n = d
0
+ d
1
p + ... + d
r
p
r
onde 0 d
i
< p. Entao,
5
POT 2012 - Teoria dos N umeros - Nvel 2 - Aula 14 - Samuel Feitosa 2 DEFINIC

AO E PROPRIEDADES
_
n
p
_
= d
1
+ d
2
p + . . . + d
r
p
r1
,
_
n
p
2
_
= d
2
+ d
3
p + . . . + d
r
p
r2
,
. . .
_
n
p
_
= d
r
.
Somando tudo, obtemos:
k
p
(n!) = d
1
+ d
2
(p + 1) + d
3
(p
2
+ p + 1) + + d
r
(p
r1
+ + 1)
=
1
p 1
_
d
1
(p 1) + d
2
(p
2
1) + + d
r
(p
r
1)
_
=
n v
p
(n)
p 1
Exemplo 14. Seja B(m) o conjunto dos inteiros r tais que 2
r
e um termo na representac ao
na base 2 de n. Por exemplo, B(100) = {2, 5, 6} pois 100 = 2
2
+ 2
5
+ 2
6
. Prove que
_
n
k
_
e
mpar se, e somente se, B(k) B(n).
Aproveitando a nota cao dos teoremas anteriores, t
2
(n) = n v
2
(n). Assim,
_
n
k
_
1 (mod 2) [n v
2
(n)] [(k v
2
(k)) ((n k) v
2
(n k))] = 0
v
2
(k) + v
2
(n k) v
2
(n) = 0
v
2
(k) + v
2
(n k) = v
2
(n).
A ultima equa cao nos diz que na soma das representa coes na base 2 de nk e k nao ocorre
a operacao de vai um, ou seja, B(k) B(n).
Observacao 15. Esse exemplo tambem mostra que
_
n
k
_
0 (mod 2) para todo k {1, 2, . . . , n
1} se, e somente se, n e uma potencia de 2.
Exemplo 16. (Olimpada Rioplatense) Seja r um real tal que
_
r +
19
100
_
+
_
r +
20
100
_
+ +
_
r +
92
100
_
= 554.
Calcule 100r .
Sejam r = a e {x} = b. Podemos reescrever a equa c ao como:
74a +
73

i=0
_
b +
19 + i
100
_
= 554.
6
POT 2012 - Teoria dos N umeros - Nvel 2 - Aula 14 - Samuel Feitosa 2 DEFINIC

AO E PROPRIEDADES
Como 0
_
b +
19 + i
100
_
< 2 para todo i {0, 1, . . . , 73}, segue que 74a 554 74a + 74.
N ao podemos ter 554 = 74(a+1) pois 74 554. Logo, a < 554/74 < a+1 e por conseguinte
a = 554/74 = 7. Assim,
73

i=0
_
b +
19 + i
100
_
= 554 74 7 = 36.
Pelo primeiro exemplo,
99

i=0
_
b +
i
100
_
= j.
onde
j
100
b <
j + 1
100
. Nao podemos ter
_
r +
19
100
_
= 1 pois nesse caso teramos
73

i=0
_
r +
19 + i
100
_
73 = 36.
Logo,
_
r +
i
100
_
= 0 para todo i 19. Como
_
r +
i
100
_
= 1 se 100 j i 99, temos:
100b = j
=
99

i=0
_
r +
i
100
_
=
92

i=19
_
r +
i
100
_
+
__
r +
93
100
_
. . .
_
r +
99
100
__
= 36 + 7
= 43.
Finalmente,
100r = 100a +100b
= 700 + 43
= 743.
Exemplo 17. Prove que existe um natural n tal que a representac ao decimal de n
2
comeca
( da esquerda para a direita ) com o n umero 201120112011. . . 2011 ( 2011 vezes).
Podemos encontrar n tal que n
2
comece com qualquer sequencia de dgitos (c
1
c
2
...c
r
) = m.
Tome k sucientemente grande tal que 2

m < 10
k1
. Seja n = 10
k

m + 1. Ent ao,
0 < 10
k

m < n 10
k

m + 1
10
2k
m < n
2
10
2k
m + 2.10
k

m + 1
10
2k
m < n
2
10
2k
m + 10
2k1
+ 1
10
2k
m < n
2
< 10
2k
(m + 1).
Assim, n
2
comeca com a sequencia de dgitos m.
7
POT 2012 - Teoria dos N umeros - Nvel 2 - Aula 14 - Samuel Feitosa 2 DEFINIC

AO E PROPRIEDADES
Exemplo 18. (OBM 1999) Prove que ha pelo menos um algarismo diferente de zero entre
a 1000000
a
e a 3000000
a
casa decimal de

2 apos a vrgula.
Suponha que n ao, entao 10
210
6
10
10
6

2 = 10
310
6

2. Se k = 10
10
6

2, temos:
10
210
6
k 10
310
6

2 < 10
210
6
k + 1
k
10
10
6
<

2 <
k
10
10
6
+
1
10
310
6

k
2
10
210
6
< 2 <
k
2
10
210
6
+
2k
10
410
6
+
1
10
610
6
Como
2k
10
210
6
<
2

210
10
6
10
210
6

1
2
,
k
2
< 2 10
210
6
< k
2
+ 1
0 < 2 10
210
6
k
2
< 1.
Isso e um absurdo pois 2 10
210
6
k
2
Z.
Exemplo 19. ( Sequencia de Beatty) Se e sao irracionais satisfazendo
1

+
1

= 1 entao,
as sequencias
, 2, 3, . . . ;
e
, 2, 3, . . . ;
incluem todos os n umeros naturais exatamente uma vez.
Primeiramente, provemos a unicidade. Suponha que k = l = n, como e sao
irracionais, n < k < n + 1 e n < l < n + 1. Assim,
k
n + 1
+
l
n + 1
<
1

+
1

<
k
n
+
l
n
Isso nos diz que
k + l
n + 1
< 1 <
k + l
n
. Temos um absurdo pois a desigualdade anterior diz
que o inteiro k + l est a entre dois inteiros consecutivos.
Mostremos agora que todo natural aparece nas sequencias. Dado n N, existe k Z
+
tal
que
k 1
n
<
1

<
k
n
.
8
POT 2012 - Teoria dos N umeros - Nvel 2 - Aula 14 - Samuel Feitosa 3 A SOLUC

AO DO PROBLEMA INICIAL
Dividamos o intervalo [k/(n+1), k/n] em duas partes. Se
k 1
n
<
1

<
k
n
, temos k = n.
Se por outro lado,
k 1
n
<
1

<
k
n + 1
, temos:
k 1
n
< 1
1

<
k
n + 1

n + 1 k
n + 1
<
1

<
n + 1 k
n

(n + 1 k) = n
Em qualquer caso, n faz parte da sequencia.
3 A solucao do problema inicial
Voltemos ao exemplo inicial. Veja que toda linha ou coluna do tabuleiro deve possuir no
maximo uma posic ao perdedora. Se a k-esima coluna n ao possuir nenhuma posic ao per-
dedora, para cada uma de suas casinhas, poderemos encontrar um posic ao perdedora na
mesma linha ou diagonal e isso implicaria na existencia de uma innidade de posic oes
perdedoras entre as k 1 primeiras colunas. O mesmo argumeto se aplica para as linhas.
Consequentemente, cada natural aparece exatamente uma vez dentre os termos da sequencia
x
0
, x
1
, . . . ; y
0
, y
1
, . . .. Alem disso, tambem e facil concluir que as sequencias (x
n
) e (y
n
) sao
crescentes. Induzidos do exemplo anterior, isso nos leva a conjecturar a existencia de dois
irracionais e que possam de alguma forma gerar os termos das posic oes perdedoras.
A inserc ao dos pontos (x
n
, y
n
) em um graco sugere que esses pontos est ao pr oximos a
alguma reta. Isso poderia ser traduzido dizendo que o quociente y
n
/x
n
e pr oximo a algum
valor. De fato, se =
1 +

5
2
, x
n
n e y
n
( + 1)n. Como as posic oes sao n umeros
inteiros, podemos conjecturar que:
Conjectura 20. Se =
1 +

5
2
, entao (x
n
, y
n
) = (n , n ( + 1))
Veja que e irracional e que:
1

+
1
+ 1
=
2 + 1

2
+
=
2 + 1
2 + 1
= 1.
Provemos a armac ao anterior por induc ao. Ela e facilmente vericavel para os casos
iniciais apresentados na primeira tabela. Suponha sua validade para todos os inteiros no
conjunto {0, 1, 2, . . . , k}. Provemos que o mesmo tambem pe v alido para k + 1. Seja t o
menor natural que n ao est a no conjunto {x
0
, x
1
, . . . , x
k
, y
0
, y
1
, . . . , y
k
}. Como as sequencias
9
POT 2012 - Teoria dos N umeros - Nvel 2 - Aula 14 - Samuel Feitosa 3 A SOLUC

AO DO PROBLEMA INICIAL
x
n
e y
n
sao crescentes e x
n
< y
n
, se x
k+1
= t, o inteiro t n ao aprecera entre os termos
das sequencias e isso contradiz nossa observac ao inicial. Em virtude da unicidade de rep-
senta c ao da sequencia de Beatty, o inteiro (k +1) ainda n ao apareceu dentre os termos
das k + 1 primeiras posic oes perdedoras. Se t < (k + 1), como x
n
e crescente, deve
existir j tal que ( + 1)j = t com j > k. Nesse caso,
t = ( + 1)j
( + 1)(k + 1)
= (k + 1) + k + 1
> (k + 1)
Contrariando a suposic ao inicial sobre t. Logo, devemos ter x
k+1
= (k + 1). Seja
l = y
k+1
x
k+1
. Se l < k + 1, O movimento diagonal
(x
k+1
, y
k+1
) (x
l
, y
k+1
x
k+1
+ x
l
) = (x
l
, y
l
),
passa uma posic ao perdedora para contradizendo o fato de que (x
k+1
, y
k+1
) era uma posic ao
perdedora. Se l > k +1, y
k+1
> (+1)(k +1) e o jogador naquela posic ao pode remover
pedras de apenas uma pilha obtendo:
(x
k+1
, y
k+1
) ((k + 1), ( + 1)(k + 1)).
Usando novamente hip otese de induc ao e lembrando que y
i
x
i
= k + 1 para todo i
{0, 1, . . . , k}, qualquer movimento do pr oximo jogador, conduzir a a uma posic ao em que exa-
tamente uma das pilhas possui um n umero de pedras igual a um dos n umeros x
0
, y
0
, . . . , x
k
, y
k
.
Assim, o oponente podera passar uma posic ao perdedora. Novamente temos um absurdo
pois (x
k+1
, y
k+1
). Logo, y
k+1
x
k+1
= k + 1 e consequentemente y
k+1
= ( + 1)(k + 1)
concluindo a prova da conjectura para k + 1.
Problemas Propostos
Problema 21. Mostre que a parte fracion aria do n umero

4n
2
+ n n ao e maior que 0, 25.
Problema 22. Sejam {a
i
}
0ir
, inteiros n ao negativos com n = a
1
+a
2
+ +a
r
. Mostre
que
n!
a
1
!a
2
!...a
r
!
e um inteiro.
Problema 23. Prove que, para qualquer n natural,

i=1
_
n + 2
i1
2
i
_
= n.
Problema 24. Seja p um divisor primo do n umero
_
2n
n
_
, com p

2n. Ent ao o expoente


de p na fatorac ao em primos do do n umero
_
2n
n
_
e igual a 1.
10
POT 2012 - Teoria dos N umeros - Nvel 2 - Aula 14 - Samuel Feitosa 3 A SOLUC

AO DO PROBLEMA INICIAL
Problema 25. (Coreia 1997) Expresse

n
k=1

k em termos de n e

n.
Problema 26. (Canad a 1998) Determine o n umero de solu c oes reais da equa c ao
_
a
2
_
+
_
a
3
_
+
_
a
5
_
= a.
Problema 27. Encontre todos o reais tais que a igualdade

n+

n + =

4n + 1
e verdadeira para todos os naturais n.
Problema 28. Se a, b, c sao reais e na + nb = nc para todo n natural, entao a Z
ou b Z.
Problema 29. Sejam a, b, c e d n umeros reais. Suponha que an + bn = cn + dn
para todos os inteiros positivos n. Mostre que pelo menos um dentre a + b, a c, a d e
inteiro.
Problema 30. Seja n 3 um inteiro positivo. Mostre que e possvel eliminar no maximo
dois elementos do conjunto {1, 2, . . . , n} de modo que a soma dos n umeros restantes seja
um quadrado perfeito.
Problema 31. Sejam a, m, b inteiros dados, com mdc(a, m) = 1. Calcule
m1

x=0
_
ax + b
m
_
.
Problema 32. Encontre todos os naturais n tais que 2
n1
| n!.
Problema 33. Determine os pares (a, b) de reais tais que abn = ban para todo inteiro
positivo n.
Problema 34. Se p e primo , entao
_
p
k
i
_
0 (mod p)( para 1 i p
k
1).
Problema 35. Prove que
_
(
3

n +
3

n + 2)
3
_
e divisvel por 8.
Problema 36. Prove que, t
1
+ t
2
+ . . . + t
n
=
_
n
1
_
+
_
n
2
_
+ ... +
_
n
n
_
, onde t
n
e o n umero
de divisores do natural n.
Problema 37. Prove que, se p e um n umero primo, entao a diferenca
_
n
p
_

_
n
p
_
e divisvel
por p.
11
Polos Olmpicos de Treinamento
Curso de Teoria dos Nmeros - Nvel 2
Prof. Samuel Feitosa
Aula
16
Ordem
Denicao 1. O menor inteiro positivo k para o qual a
k
1 (mod m), onde mdc(a, m) = 1,
e chamado ordem de a modulo me ser a denotado por ord
m
a.
Teorema 2. Se a e um inteiro relativamente primo com m, entao a
n
1 (mod m) se, e
somente se, ord
m
a|n. Ademais, a
n
0
a
n
1
(mod m) se , e somente se, n
0
n
1
(mod ord
m
a)
Demonstra c ao. Sejam b = ord
m
a e n = qb + r com 0 r < b. Como a
b
1 (mod m),
a
n
1 (mod m) a
qb+r
1 (mod m)
a
r
1 (mod m)
Como 0 r < b, devemos ter r = 0. Usando que mdc(a, m) = 1 e supondo que n
0
> n
1
,
a
n
0
a
n
1
(mod m) a
n
0
n
1
1 (mod m)
n
0
n
1
0 (mod b)
Teorema 3. Se mdc(a, m) = 1, ord
m
a|(m)
Demonstra c ao. Pelo teorema de Euler, a
(m)
1 (mod m). O resultado segue do teorema
anterior.
Problema 4. (Putnam 1972) Prove que n ao existe inteiro positivo n > 1 tal que n|2
n
1.
Suponha, por absurdo, que existe um inteiro positivo n > 1 com essa propriedade e que k
e o menor dentre eles. Se d = ord
k
2, entao d | k. Como 2
d
1 (mod k), temos 2
d
1
(mod d). Em virtude da minimalidade de k, temos d = 1 ou d = k. No primeiro caso,
teramos k = 1 produzindo uma contradi cao. No segundo caso, em decorrencia do teorema
anterior, k | (k). Entretanto, se k > 1, (k) k 1 e obtemos assim um absurdo.
Problema 5. (Leningrado 1990) Prove que para todos os inteiros a > 1 e n , n|(a
n
1).
Se k = ord
a
n
1
a, como a
n
1 (mod a
n
1), temos k | n e consequentemente k n. Nao
podemos ter k < n porque a
n
1 | a
k
1 a
n
1 a
k
1. Assim, k = n e usando o
teorema anterior podemos concluir que k | (a
n
1).
Problema 6. Mostre que
a) ord
3
n2 = 2 3
n1
b) Se 2
m
1 (mod 3
n
), entao 3
n1
| m.
Provaremos por inducao que 2
3
k
+ 1 = 3
k+1
m
k
com 3 m
k
. Suponha que a armacao vale
para k. Provemos para k + 1:
2
3
k+1
= (3
k+1
m
k
1)
3
= 3
3k+3
m
3
k
3
2k+3
m
2
k
+ 3
k+2
m
k
1
= 3
k+2
(3
2k+1
m
3
k
3
k+1
+ m
k
) 1
= 3
k+2
m
k+1
1
Claramente 3 m
k+1
. Voltemos ao problema. Seja b = ord
3
n2, entao b | (3
n
) = 2 3
n1
.
Temos duas possibilidades: ou b = 2 3
j
ou b = 3
j
. Como 2
3
n1
1 (mod 3
n
) e 3
j
| 3
n1
se j n 1, devemos ter b = 2 3
j
. Assim, (2
3
j
1)(2
3
j
+ 1) 1 (mod 3)
n
. Usando que
2
3
j
1 1 (mod 3), temos 2
3
j
1 (mod 3
n
). Novamente pelo lema provado no incio,
3
j
3
n1
e assim b = 2 3
n1
. Para o item b), de 2
m
1 (mod 3
n
), podemos concluir
que 2
2m
1 (mod 3)
n
. Da, 2 3
n1
| 2m e o resultado segue.
Problema 7. (Bulgaria 1997) Encontre todos os n umeros inteiros m, n 2 tais que
1 + m
3
n
+ m
23
n
n
e um inteiro
Claramente n e mpar, mdc(m, n) = 1 e n > 2. Se n = 3, como mdc(m, n) = 1 devemos
ter que m 1 (mod 3) pois caso contr ario 1 + m
3
n
+ m
23
n
1 1 + 1 1 (mod 3).

E facil ver que todo par (m, n) = (3k + 1, 3) e solu cao. Suponha agora n > 3 e seja
k = ord
n
m. Se n > 3 m
3
n
1 (mod n). Como 1 + m
3
n
+ m
23
n
=
m
3
n+1
1
m
3
n
1
segue
que n | m
3
n+1
1 k | 3
n+1
. Logo, k = 3
n+1
. Pelo teorema de Euler, m
(n)
1 (mod n)
entao k (n) e 3
n+1
(n) n 1, uma contradi cao.
Problema 8. Prove que que se p e primo, entao p
p
1 tem um fator primo congruente a
1 modulo p
Seja q um primo que divide
p
p
1
p 1
. Como q | p
p
1 segue que ord
q
p | p. Se ord
q
p = 1
entao q | p
p
1 e 0 p
p1
+p
p2
+. . . p +1 1 + 1 +. . . + 1 +1 p (mod q). Mas isso e
um absurdo pois p = q. Logo ord
q
p = p e obtemos p | (q) = q 1. Da, todos os divisores
primos de
p
p
1
p 1
s ao congruentes a 1 m odulo p.
Problemas Propostos
Problema 9. Se ord
a
m = h, ord
m
b = k e mdc(h, k) = 1 mostre que ord
m
ab = hk.
2
Problema 10. Prove que se a, b s ao n umeros naturais tais que a > b , n > 1, entao cada
divisor primo do n umero a
n
b
n
e ou da forma nk +1, onde k e um inteiro, ou um divisor
de um n umero a
n
1
b
n
1
, onde n
1
|n e n
1
< n.
Problema 11. Prove que se a, b s ao n umeros naturais tais que a > b , n > 1, entao cada
divisor primo do n umero a
n
+ b
n
e ou da forma 2nk + 1, onde k e um inteiro, ou um
divisor de um n umero a
n
1
+ b
n
1
, onde n
1
e o quociente obtido por dividir o n umero n por
um n umero mpar mairo que 1.
Problema 12. Seja p um primo que n ao divide 10, e seja n um inteiro, 0 < n < p. Seja d
a ordem de 10 modulo p.
1. Mostre que o comprimento do perodo da representac ao decimal de n/p e d.
2. Prove que que se d e par, entao o perodo da representac ao decimal de n/p pode ser
dividido em duas partes cuja soma e 10
d/2
1. Por exmeplo, 1/7 = 0, 142857, entao
d = 6, e 142 + 857 = 999 = 10
3
1.
3. Se ord
m
a = h ord
m
a
k
=
h
mdc(h, k)
Problema 13. Se p e um primo maior que 3, entao qualquer divisor maior que 1 do n umero
2
p
+ 1
3
e da forma 2kp + 1, onde k e um n umero natural.
Teorema 14. Se p e um primo maior que 2, entao qualquer n umero natural que divida o
n umero 2
p
1 e da forma 2kp + 1, onde k e um inteiro.
Problema 15. (Bulgaria 1995) Encontre todos os primos p e q tais que o n umero 2
p
+ 2
q
seja divisvel por pq.
Problema 16. Mostre que se k > 1 entao 2
k1
1 (mod k)
Problema 17. Mostre que se 3 d 2
n1
, entao d (a
2
n
+1) para qualquer inteiro positivo
a.
Problema 18. (Eureka) Prove que se p e um primo da forma 4k +3, entao 2p +1 tambem
e primo se e somente se 2p + 1 divide 2
p
1.
Problema 19. Prove que todos os divisores dos n umeros de Fermat 2
2
n
+ 1, n > 1, s ao da
forma 2
n+2
k + 1.
Problema 20. (IMO 1990) Encontre todos os inteiros positivos n > 1 tais que
2
n
+ 1
n
2
e um inteiro.
Problema 21. (Teste Cone Sul 2002) Encontre o perodo na representac ao decimal de
1
3
2002
.
3
Problema 22. (Teste de Selec ao do Ira para a IMO) Seja a um natural xo. Mostre que o
conjunto dos divisores primos de 2
2
n
+ a, para n N, e innito.
Problema 23. (Col ombia 2009) Encontre todas as triplas de inteiros positivos (a, b, n) que
satisfazem a equa c ao:
a
b
= 1 + b + . . . + b
n
.
Problema 24. (IMO 2003) Seja p um n umero primo. Demonstre que existe um n umero
primo q tal que, para todo inteiro n, o n umero n
p
p n ao e divisvel por q.
4
Polos Olmpicos de Treinamento
Curso de Teoria dos Nmeros - Nvel 2
Prof. Samuel Feitosa
Aula
17
Aula de Revisao e Aprofundamento
Exemplo 1. O mnimo multiplo comum dos inteiros a, b, c, d e d e igual ` a a + b + c + d.
Prove que abcd e divisvel por 3 ou por 5.
Solu cao: Suponha inicialmente que mdc(a, b, c, d) = 1 e seja L = a +b +c +d. Como L e
o mnimo m ultiplo comum, existem x, y, z, w tais que aw = bx = cy = dz = L.

E facil ver
que L tambem e o mnimo m ultiplo comum de x, y, z, w e que
1
x
+
1
y
+
1
z
+
1
w
=
a
L
+
b
L
+
c
L
+
d
L
= 1.
Suponha sem perda de generalidade que w x y z. Da equacao anterior, o maior
valor de w e 4 e ocorrendo igualdade deveramos ter a = b = c = d = 1 que n ao satisfaz
o enunciado. Para w = 3, o leitor poder a facilmente vericar que a = b = 4, c = 3
e d = 1 e a unica solu cao. Para w = 2, temos as seguintes solu coes: (a, b, c, d) =
(5, 2, 2, 1), (10, 5, 4, 1), (6, 4, 1, 1) , (9, 6, 2, 1), (12, 8, 3, 1), (21, 14, 6, 1). Em todos os casos, L
e divisvel por 3 ou 5.
Exemplo 2. Seja A = {a
1
< a
2
< a
3
< } uma seq uencia crescente de inteiros positivos
em que o n umero de fatores primos de cada termo, contando fatores repetidos, nunca e
maior que 2007. Prove que e sempre possvel extrair do conjunto A um subconjunto innito
B = {b
1
< b
2
< b
3
< }
tal que o maximo divisor comum entre b
i
e b
j
e sempre o mesmo para quaisquer naturais
i = j.
O n umero de primos usados como fatores dos a
i
n ao pode ser nito, pois se essa quantidade
e n, teramos apenas n
2007
possveis elementos a
i
. Podem ocorrer duas situa coes:
a) Nenhum primo divide innitos a
i
. Seja b
1
= a
1
. Como cada fator primo de a
1
aparece um n umero nito de vezes como fator dos a
i
, existira um termo a
l
> a
1
tal que a
1
e a
l
s ao primos entre si. Seja b
2
= a
l
. Esse argumento pode ser repetido para gerar um
subconjunto innito B = {b
1
< b
2
< b
3
< } de modo que mdc(b
i
, b
j
) = 1, quaisquer que
sejam os naturais i, j.
b) Existe um primo p que divide innitos a
i
. Como o expoente de p
1
= p em cada
a
i
que e m ultiplo de p
1
e um elemento do conjunto {1, . . . , 2007}, pelo menos um deles,
digamos r
1
> 0, dever a ocorrer innitas vezes. Seja A
1
o conjunto de todos os termos
a
i
para os quais o expoente de p
1
em a
i
e r
1
. Se nenhum primo p
2
= p
1
divide innitos
elementos de A
1
, o caso anterior mostra que A
1
possui um subconjunto tal que o m aximo
divisor comum de quaisquer dois elementos e p
1
r
1
. Sen ao, seja p
2
= p
1
um primo que divide
innitos elementos de A
1
. O expoente de p
2
em cada elemento de A
1
que e m ultiplo de p
2
pertence ao conjunto {1, . . . , 2007 r
1
}. Sejam r
2
> 0 um expoente que ocorre innitas
vezes e A
2
o conjunto dos elementos de A
1
para os quais p
1
e p
2
tem expoentes r
1
e r
2
,
respectivamente. Esse processo deve terminar em i passos, i 2007, e nessa situa cao A
i
e um subconjunto de A para o qual todo elemento e um m ultiplo de P = p
r
1
1
p
r
2
2
p
r
i
i
,
digamos
A
i
= {Pc
1
< Pc
2
< Pc
3
< },
onde C={c
1
< c
2
< c
3
< } e um conjunto em que cada termo e o produto de n ao mais
que 2007 (r
1
+r
2
+ + r
i
) fatores primos, nenhum dos quais ocorrendo innitas vezes.
Pelo caso a), C possui um subconjunto B
1
tal que quaisquer dois elementos s ao primos
entre si. O conjunto
B = P B
1
= {Px| x B
1
}
satisfaz as condi coes do problema, pois o m aximo divisor comum entre quaisquer dois de
seus elementos e igual a P.
Exemplo 3. (Seletiva Rioplatense 2001) Encontre todos os pares (m, n) de n umeros naturais
com m < n tais que m
2
+ 1 e um m ultiplo de n e n
2
+ 1 e um m ultiplo de m.
Armamos que todas as solu coes s ao da forma (F
2k1
, F
2k+1
), k 0 (F
n
e o n-esimo
termo da sequencia de Fibonacci).

E facil ver que F
2k1
F
2k+3
= F
2
2k+1
+ 1 e portanto
os pares anteriores s ao solu coes. Seja P o conjunto das solu coes que n ao s ao da forma
(F
2k1
, F
2k+1
). O conjunto P contem um par (a, b) tal que a + b e mnimo. Suponhamos
a < b(se a = b (a, b) = (1, 1) = (F
1
, F
1
) P). Como b | a
2
+ 1, a
2
+ 1 = bb

e b

< a.

E facil ver que a | b

2
+ 1 e b

| a
2
+ 1. Logo (b

, a) e uma solu cao com


b

+ a < a + b Entretanto, (b

, a) P e da (b

, a) = (F
2k1
, F
2k+1
). COnsequentemente,
F
2k1
b = b

b = a
2
+ 1 = F
2
2k+1
b = F
2k+3
(a, b) = (F
2k+1
, F
2k+3
) P. Logo P deve
ser vazio.
Exemplo 4. (URSS 1988) A sequencia de inteiros a
n
e dada por a
0
= 0, a
n
= P(a
n1
),
onde P(x) e um polinomio cujos coecientes s ao inteiros positivos. Mostre que para quais-
quer inteiros positivos m, k com maximo divisor comum d, o maximo divisor comum de a
m
e a
k
e a
d
.
Quando temos um polin omio com coecientes inteiros e sempre bom lembrar que a b |
P(a) P(b). Essa ser a nossa principal ferramenta nesta solu cao.
2
1. a
m
| a
mr
. Provaremos por induc ao. Se a
m(r1)
0 (mod a
m
) a
m(r1)+1
P(0)
(mod a
m
) a
m(r1)+2
P(P(0)) (mod a
m
) a
mr
= a
m(r1)+m
P(P(. . . (P(0))

m vezes
=
a
m
0 (mod a
m
).
2. Se l | a
t
e l | a
f
l | a
tf
(Supondo t > f). (Deixaremos a prova dessa armacao
para o leitor).
Pelo teorema de Bezout, existem inteiros positivos x, y tais que mx ky = d. Seja n =
mdc(a
m
, a
k
). Como n | a
m
| a
mx
e n | a
k
| a
ky
, pelo item 2, n | a
mxky
= a
d
. Mas a
d
| a
m
e a
d
| a
k
, entao a
d
| n. Portanto a
d
= n.
Exemplo 5. Prove que existem innitos n umeros compostos n para os quais
n | 3
n1
2
n1
.
Lema: 2
t
| 3
2
t
1 t N
Vamos provar o lema por inducao. Para t = 1 e trivial. Suponha que a armacao
seja valida para t = r, provemos que tambem e valida para t = r + 1. Fatorando
2
2
t+1
3
3
t+1
= (2
2
t
3
3
t
)(2
2
t
+ 3
3
t
), como o primeiro parentesis e m ultiplo de 2
t
e o
segundo e m ultiplo de 2, o produto deles e m ultiplo de 2
t+1
.
Seja n = 3
2
t
2
2
t
com t > 1. Como n 1 (3
2
t
1) 2
2
t
0 (mod 2
t
), pelo lema,
obtemos n = 3
2
t
2
2
t
| 3
n1
2
n1
. Aqui estamos usando o fato que x
k
y
k
| x
mk
y
mk
.
Exemplo 6. Consideramos todas as sequencias (x
n
)
n1
de inteiros positivos satisfazendo
x
n+2
= mdc(x
n
, x
n+1
) + 2008, n 1.
Alguma dessas seq uencias contem exatamente 10
2008
n umeros distintos?
A ideia e construir a sequencia de tr as para frente. Mostraremos que para qualquer inteiro
positivo k > 1, existe uma tal sequencia contendo exatamente k n umeros distintos. Basta
encontrarmos uma sequencia que satisfaca:
x
n+2
= mdc(x
n
, x
n+1
) + 2, n 1. (1)
pois a multiplica cao de todos os termos por 1004 produz a sequencia do problema. De-
namos a
1
= 4, a
2
= 6, a
3
= 8, a
4
= 2(a
3
2) e a
n
=
(a
n1
2)(a
n2
2)
2
para n 5.

E facil ver por inducao que todos os a


i
ser ao pares e que a sequencia e crescente. Con-
sequentemente, todos os a
i
com i 1 s ao inteiros e distintos. Alem disso, denamos os
termos com ndices n ao positivos por a
3k
= 4, a
3k1
= 6, a
3k2
= 4 para k 0.
. . . a
6
, a
5
, a
4
, a
3
= 8, a
2
= 6, a
1
= 4, a
0
= 4, a
1
= 6, a
2
= 4, a
3
= 4, a
4
= 6, . . .
Armamos que a sequencia anterior satisfaz mdc(a
n+2
, a
n+1
)+2 = a
n
n Z.

E imediato
vericar isso para n 1 Para n = 2, mdc(a
4
, a
3
) + 2 = mdc(12, 8) + 2 = 6 = a
2
. Para
n > 2,
mdc(a
n+2
, a
n+1
)+2 = mdc(
(a
n+1
2)(a
n
2)
2
, a
n+1
)+2 = mdc((a
n+1
)
a
n
2
2
a
n
+2, a
n+1
)+2
3
Portanto,
mdc(a
n+2
, a
n+1
) + 2 = mdc(a
n
2, (a
n
2)
a
n1
2
2
) + 2 = a
n
2 + 2 = a
n
.
Para encontrarmos uma sequencia satisfazendo (1) com exatamente k > 1 termos distintos
basta escolhermos x
1
= a
k
e x
2
= a
k1
e denirmos o resto da sequencia usando a rela cao
de recorrencia x
n+2
= mdc(x
n
, x
n+1
) + 2
Problemas Propostos
Problema 7. Resolva em inteiros a equa c ao:
xy
z
+
xz
y
+
yz
x
= 3
Exemplo 8. Alguns inteiros positivos estao escritos no quadro. Podemos apagar quaisquer
dois inteiros distintos e substitu-los pelo maximo divisor comum e o mnimo divisor comum
dos dois n umeros. Prove que eventualmente a opera c ao de alterar os n umeros n ao ser a mais
executada.
Exemplo 9. Mostre que se k > 1 entao 2
k1
1 (mod k)
Exemplo 10. (Est onia 2005) Sejam a, b inteiros positivos primos entre si tais que (a +
b)/(ab) e um inteiro positivo. Prove que ao menos um dentre os n umeros ab+1 e 4ab+1
e um quadrado perfeito.
Exemplo 11. (Ibero 1999) Seja n um inteiro maior que 10 tal que cada um de seus dgitos
pertence ao conjunto S = {1, 3, 7, 9}. Prove que n tem algum divisor primo maior ou igual
a 11.
4

Você também pode gostar